Go Math Grade 6 Answer Key Chapter 4 Model Ratios

go-math-grade-6-chapter-4-model-ratios-answer-key

Model Ratios are an interesting topic in Grade 6.  The students who are in search of Go Math Grade 6 Answer Key Chapter 4 Model Ratios can get them on this page. The best and quick way of learning is possible with Go Math Grade 6 Chapter 4 Model Ratios Answer Key. We have provided the solutions for all the questions in an easy manner in our Go Math Grade 6 Chapter 4 Model Ratios Solution Key. Get Free Access to Download Go Math Grade 6 Chapter 4 Model Ratios Solution Key PDF @ ccssmathanswers.com Thus make use of the links and Download HMH Go math Grade 6 answer key chapter 4 model ratios and get the best results.

Go Math Grade 6 Chapter 4 Model Ratios Answer Key

The easy-solving of math problems will help you to understand all the difficult problems. So, begin your practice now and be on the top list to score good marks in the exam. Use Go Math 6 Standard Answer Key Chapter 4 Model Ratios handy solutions to learn deep maths online or offline. Come and fall in love with maths by practicing the problems from HMH Go Math 6th Grade Chapter 4 Model Ratios Answer Key. Hit the links and start your preparation now.

Lesson 1: Investigate • Model Ratios

Lesson 2: Ratios and Rates

Lesson 3: Equivalent Ratios and Multiplication Tables

Lesson 4: Problem Solving • Use Tables to Compare Ratios

Lesson 5: Algebra • Use Equivalent Ratios

Mid-Chapter Checkpoint

Lesson 6: Find Unit Rates

Lesson 7: Algebra • Use Unit Rates

Lesson 8: Algebra • Equivalent Ratios and Graphs

Chapter 4 Review/Test

Share and Show – Page No. 213

Write the ratio of yellow counters to red counters.

Question 1.
Go Math Grade 6 Answer Key Chapter 4 Model Ratios 1
Type below:
___________

Answer:
1: 2

Explanation:
There are one yellow counter and two red counters.
So, the ratio is 1:2

Question 2.
Go Math Grade 6 Answer Key Chapter 4 Model Ratios 2
Type below:
___________

Answer:
5: 3

Explanation:
There are 5 yellow counter and 3 red counters.
So, the ratio is 5:3

Draw a model of the ratio.

Question 3.
3 : 2
Type below:
___________

Answer:
Grade 6 Chapter 4 image 1

Explanation:
As the ratio is 3:2, we can draw three yellow counters and 2 red counters.

Question 4.
1 : 5
Type below:
___________

Answer:
Grade 6 Chapter 4 image 2

Explanation:
As the ratio is 1:5, we can draw 1 yellow counter and 5 red counters.

Use the ratio to complete the table.

Question 5.
Wen is arranging flowers in vases. For every 1 rose she uses, she uses 6 tulips. Complete the table to show the ratio of roses to tulips.
Go Math Grade 6 Answer Key Chapter 4 Model Ratios 3
Type below:
___________

Answer:
Grade 6 Chapter 4 image 3

Explanation:
There is 1 box for every 6 Tulips.
The ratio is 1 : 6.
Each time the number of boxes increases by 1, the number of Tulips increases by 6
So, for 2 boxes, 6 + 6 = 12 Tulips
For 3 boxes, 12 + 6 = 18 Tulips
For 4 boxes, 18 + 6 = 24 Tulips

Question 6.
On the sixth-grade field trip, there are 8 students for every 1 adult. Complete the table to show the ratio of students to adults.
Go Math Grade 6 Answer Key Chapter 4 Model Ratios 4
Type below:
___________

Answer:
Grade 6 Chapter 4 image 4

Explanation:
There is 1 adult out of 8 Students.
The ratio is 8:1.
Each time the number of students increases double, the number of adults becomes double.
So, for 8 + 8 = 16 students, 2 Adults available
For 16 + 8 = 24 students, 2 + 1 = 3 Adults available
For 24 + 8 = 32 students, 3 + 1 = 4 Adults available

Question 7.
Zena adds 4 cups of flour for every 3 cups of sugar in her recipe. Draw a model that compares cups of flour to cups of sugar.
Type below:
___________

Answer:
Grade 6 Chapter 4 image 5

Explanation:
Zena adds 4 cups of flour for every 3 cups of sugar in her recipe.
For every 3 cups of sugar, she adds 4 cups of flour.
For 6 cups of sugar, she adds 8 cups of flour
For 9 cups of sugar, she adds 12 cups of flour
For 12 cups of sugar, she adds 16 cups of flour

Draw Conclusions – Page No. 214

The reading skill draw conclusions can help you analyze and make sense of information.

Hikers take trail mix as a snack on long hikes because it is tasty, nutritious, and easy to carry. There are many different recipes for trail mix, but it is usually made from different combinations of dried fruit, raisins, seeds, and nuts. Tanner and his dad make trail mix that has 1 cup of raisins for every 3 cups of sunflower seeds.

Question 8.
Model Mathematics Explain how you could model the ratio that compares cups of raisins to cups of sunflower seeds when Tanner uses 2 cups of raisins.
Type below:
___________

Answer:
Grade 6 Chapter 4 image 6

Explanation:
Hikers take trail mix as a snack on long hikes because it is tasty, nutritious, and easy to carry. There are many different recipes for trail mix, but it is usually made from different combinations of dried fruit, raisins, seeds, and nuts. Tanner and his dad make trail mix that has 1 cup of raisins for every 3 cups of sunflower seeds.
For 2 cups of raisins, he needs 3 + 3 = 6 cups of sunflower seeds

The table shows the ratio of cups of raisins to cups of sunflower seeds for different amounts of trail mix. Model each ratio as you complete the table.
Go Math Grade 6 Answer Key Chapter 4 Model Ratios 5
Go Math Grade 6 Answer Key Chapter 4 Model Ratios 6

Question 9.
Describe the pattern you see in the table.
Type below:
___________

Answer:
Multiply Raisins by 3 to get number of Sunflower Seeds.

Question 10.
Draw Conclusions What conclusion can Tanner draw from this pattern?
Type below:
___________

Answer:
He needs 3 times as many seeds as raisins

Question 11.
What is the ratio of cups of sunflower seeds to cups of trail mix when Tanner uses 4 cups of raisins?
Type below:
___________

Answer:
4:12

Explanation:
If tanner uses 4 cups of raisins, he needs 12 cups of sunflower seeds.

Model Ratios – Page No. 215

Write the ratio of gray counters to white counters.

Question 1.
Go Math Grade 6 Answer Key Chapter 4 Model Ratios 7
Type below:
___________

Answer:
3:4

Explanation:
There are 3 gray counter and 4 white counters.
So, the ratio is 3:4

Question 2.
Go Math Grade 6 Answer Key Chapter 4 Model Ratios 8
Type below:
___________

Answer:
4:1

Explanation:
There are 4 gray counter and 1 white counter.
So, the ratio is 4:1

Question 3.
Go Math Grade 6 Answer Key Chapter 4 Model Ratios 9
Type below:
___________

Answer:
2:3

Explanation:
There are 2 gray counter and 3 white counters.
So, the ratio is 2:3

Draw a model of the ratio.

Question 4.
5 : 1
Type below:
___________

Answer:
Grade 6 Chapter 4 image 7

Explanation:
As the ratio is 5:1, we can draw 5 yellow counters and 1 red counter.

Question 5.
6 : 3
Type below:
___________

Answer:
Grade 6 Chapter 4 image 8

Explanation:
As the ratio is 6:3, we can draw 6 yellow counters and 3 red counters.

Use the ratio to complete the table.

Question 6.
Marc is assembling gift bags. For every 2 pencils he places in the bag, he uses 3 stickers. Complete the table to show the ratio of pencils to stickers.
Go Math Grade 6 Answer Key Chapter 4 Model Ratios 10
Type below:
___________

Answer:
Grade 6 Chapter 4 image 9

Explanation:
Marc is assembling gift bags. For every 2 pencils he places in the bag, he uses 3 stickers.
For 4 pencils, he uses 3 + 3 = 6 stickers
For 6 pencils, he uses 6 + 3 = 9 stickers
For 8 pencils, he uses 9 + 3 = 12 stickers

Question 7.
Singh is making a bracelet. She uses 5 blue beads for every 1 silver bead. Complete the table to show the ratio of blue beads to silver beads
Go Math Grade 6 Answer Key Chapter 4 Model Ratios 11
Type below:
___________

Answer:
Grade 6 Chapter 4 image 10

Explanation:
Singh is making a bracelet. She uses 5 blue beads for every 1 silver bead.
For 2 silver bead, she uses 5 + 5 = 10 blue beads.
For 3 silver bead, she uses 10 + 5 = 15 blue beads.
For 4 silver bead, she uses 15 + 5 = 20 blue beads.

Problem Solving

Question 8.
There are 4 quarts in 1 gallon. How many quarts are in 3 gallons?
______ quarts

Answer:
12 quarts

Explanation:
There are 4 quarts in 1 gallon. If there are 3 gallons, he uses 3 × 4 = 12 quarts

Question 9.
Martin mixes 1 cup lemonade with 4 cups cranberry juice to make his favorite drink. How much cranberry juice does he need if he uses 5 cups of lemonade?
______ cups

Answer:
20 cups

Explanation:
Martin mixes 1 cup lemonade with 4 cups cranberry juice to make his favorite drink. If he uses 5 cups of lemonade, 5 × 4 = 20 cups

Question 10.
Suppose there was 1 centerpiece for every 5 tables. Use counters to show the ratio of centerpieces to tables. Then make a table to find the number of tables if there are 3 centerpieces.
Type below:
___________

Answer:
Grade 6 Chapter 4 image 7

Grade 6 Chapter 4 image 11

Explanation:
Suppose there was 1 centerpiece for every 5 tables.
If there are 3 centerpieces, 5 × 3 = 15 tables

Lesson Check – Page No. 216

Question 1.
Francine is making a necklace that has 1 blue bead for every 6 white beads. How many white beads will she use if she uses 11 blue beads?
______ white beads

Answer:
66 white beads

Explanation:
Francine is making a necklace that has 1 blue bead for every 6 white beads.
11 × 6 = 66 white beads

Question 2.
A basketball league assigns 8 players to each team. How many players can sign up for the league if there are 24 teams?
______ players

Answer:
192 players

Explanation:
A basketball league assigns 8 players to each team.
If there are 24 teams, 24 × 8 = 192 players to each team

Spiral Review

Question 3.
Louis has 45 pencils and 75 pens to divide into gift bags at the fair. He does not want to mix the pens and pencils. He wants to place an equal amount in each bag. What is the greatest number of pens or pencils he can place in each bag?
______

Answer:
Louis can form at most 15 bags, each of them will contain 3 pencils and 5 pens.

Explanation:
Louis has 45 pencils and 75 pens to divide into gift bags at the fair. He does not want to mix the pens and pencils. He wants to place an equal amount in each bag.
Factor both these numbers:
45 = 3·3·5;
75 = 3·5·5.
The greatest common factor (write all common factors and multiply them) is 3·5=15. Then:
45=15·3;
75=15·5.
Louis can form at most 15 bags, each of them will contain 3 pencils and 5 pens.

Question 4.
Of the 24 students in Greg’s class, \(\frac{3}{8}\) ride the bus to school. How many students ride the bus?
______ students

Answer:
9 students

Explanation:
Of the 24 students in Greg’s class, \(\frac{3}{8}\) ride the bus to school.
3/8 x 24= 9

Question 5.
Elisa made 0.44 of the free throws she attempted. What is that amount written as a fraction in simplest form?
\(\frac{□}{□}\)

Answer:
\(\frac{11}{25}\)

Explanation:
Elisa made 0.44 of the free throws she attempted.
0.44 = 44/100
44/100 = 22/50 = 11/25
11/25

Question 6.
On a coordinate plane, the vertices of a rectangle are (–1, 1), (3, 1), (–1, –4), and (3, –4). What is the perimeter of the rectangle?
______ units

Answer:
18 units

Explanation:
On a coordinate plane, the vertices of a rectangle are (–1, 1), (3, 1), (–1, –4), and (3, –4).
|-1| = 1
The distance from (–1, 1), (3, 1) is 1 + 0 + 0 + 3 = 4
|-4| = 4
The distance from (3, 1), (3, –4) is 1 + 0 + 0 + 4 = 5
perimeter of the rectangle = 4 + 5 + 5 + 4 = 18

Share and Show – Page No. 219

Question 1.
Write the ratio of the number of red bars to blue stars.
Go Math Grade 6 Answer Key Chapter 4 Model Ratios 12
\(\frac{□}{□}\)

Answer:
\(\frac{8}{3}\)

Explanation:
There are 8 stars and 3 red boxes.
So, the ratio is 8:3

Write the ratio in two different ways.

Question 2.
8 to 16
Type below:
___________

Answer:
\(\frac{8}{16}\)
8:16

Explanation:
8 to 16 as a fraction 8/16
8 to 16 with a colon 8:16

Question 3.
\(\frac{4}{24}\)
Type below:
___________

Answer:
4 to 24
4:24

Explanation:
\(\frac{4}{24}\) using words 4 to 24
\(\frac{4}{24}\) with a colon 4:24

Question 4.
1 : 3
Type below:
___________

Answer:
1 to 3
\(\frac{1}{3}\)

Explanation:
1 : 3 using words 1 to 3
1 : 3 as a fraction 1/3

Question 5.
7 to 9
Type below:
___________

Answer:
\(\frac{7}{9}\)
7:9

Explanation:
7 to 9 as a fraction 7/9
7 to 9 with a colon 7:9

Question 6.
Marilyn saves $15 per week. Complete the table to find the rate that gives the amount saved in 4 weeks. Write the rate in three different ways.
Go Math Grade 6 Answer Key Chapter 4 Model Ratios 13
Type below:
___________

Answer:
Grade 6 Chapter 4 image 15

Explanation:
Marilyn saves $15 per week.
for 4 weeks, $15 × 4 = $60

On Your Own

Write the ratio in two different ways.

Question 7.
\(\frac{16}{40}\)
Type below:
___________

Answer:
16 to 40
16:40

Explanation:
\(\frac{16}{40}\) using words 16 to 40
\(\frac{16}{40}\) with a colon 16:40

Question 8.
8 : 12
Type below:
___________

Answer:
8 to 12
\(\frac{8}{12}\)

Explanation:
8 : 12 using words 8 to 12
8 : 12 as a fraction \(\frac{8}{12}\)

Question 9.
4 to 11
Type below:
___________

Answer:
\(\frac{4}{11}\)
4:11

Explanation:
4 to 11 as a fraction \(\frac{4}{11}\)
4 to 11 with a colon 4:11

Question 10.
2 : 13
Type below:
___________

Answer:
2 to 13
\(\frac{2}{13}\)

Explanation:
2 : 13 using words 2 to 13
2 : 13 as a fraction \(\frac{2}{13}\)

Question 11.
There are 24 baseball cards in 4 packs. Complete the table to find the rate that gives the number of cards in 2 packs. Write this rate in three different ways.
Go Math Grade 6 Answer Key Chapter 4 Model Ratios 14
Type below:
___________

Answer:
Grade 6 Chapter 4 image 16

Explanation:
There are 24 baseball cards in 4 packs.
For 2 packs, (2 × 24)/4 = 12
For 1 pack, (1× 24)/4 = 6

Question 12.
Make Connections Explain how the statement “There is \(\frac{3}{4}\) cup per serving” represents a rate.
Type below:
___________

Answer:
There is a 3/4 cup of whatever in one serving. If that serving amount changed to 2, then the 3/4 would be multiplied by 2. If there is half a serving, then it would be divided by 2. There is a constant change and not one that is always changing.

Problem Solving + Applications – Page No. 220

Use the diagram of a birdhouse for 13–15.
Go Math Grade 6 Answer Key Chapter 4 Model Ratios 15

Question 13.
Write the ratio of AB to BC in three different ways.
Type below:
___________

Answer:
28 : 12, 28 to 12, \(\frac{2}{13}\)

Explanation:
AB = 28 in
BC = 12 in
AB : BC = 28 : 12, 28 to 12, \(\frac{2}{13}\)

Question 14.
Write the ratio of the shortest side length of triangle ABC to the perimeter of the triangle in three different ways.
Type below:
___________

Answer:
12 : 64, 12 to 64, \(\frac{12}{64}\)

Explanation:
the shortest side length of triangle ABC = 12 in
the perimeter of the triangle 12 + 28 + 24 = 64
12 : 64, 12 to 64, \(\frac{12}{64}\)

Question 15.
Represent a Problem Write the ratio of the perimeter of triangle ABC to the longest side length of the triangle in three different ways.
Type below:
___________

Answer:
64 : 28, 64 to 28, \(\frac{64}{28}\)

Explanation:
the ratio of the perimeter of triangle ABC = 12 + 28 + 24 = 64
the longest side length of the triangle = 28 in
64 : 28, 64 to 28, \(\frac{64}{28}\)

Question 16.
Leandra places 6 photos on each page in a photo album. Find the rate that gives the number of photos on 2 pages. Write the rate in three different ways.
Type below:
___________

Answer:
6 : 12, 6 to 12, \(\frac{6}{12}\)

Explanation:
Leandra places 6 photos on each page in a photo album.
For 2 pages, 6 × 2 = 12 in
6 : 12, 6 to 12, \(\frac{6}{12}\)

Question 17.
What’s the Question? The ratio of total students in Ms. Murray’s class to students in the class who have an older brother is 3 to 1. The answer is 1:2. What is the question?
Type below:
___________

Answer:
What is the ratio of students in the class who don’t have an older brother to students in the class with an older brother.

Question 18.
What do all unit rates have in common?
Type below:
___________

Answer:
A rate is a ratio that is used to compare different kinds of quantities. A unit rate describes how many units of the first type of quantity corresponds to one unit of the second type of quantity.

Question 19.
Julia has 2 green reusable shopping bags and 5 purple reusable shopping bags. Select the ratios that compare the number of purple reusable shopping bags to the total number of reusable shopping bags. Mark all that apply.

  • 5 to 7
  • 5 : 7
  • 5 : 2
  • \(\frac{2}{5}\)
  • 2 to 7
  • \(\frac{5}{7}\)

Type below:
___________

Answer:
5 to 7, 5 : 7, \(\frac{5}{7}\)

Explanation:
the number of purple reusable shopping bags = 5
the total number of reusable shopping bags = 5 + 2 = 7
5 to 7, 5 : 7, \(\frac{5}{7}\)

Ratios and Rates – Page No. 221

Write the ratio in two different ways.

Question 1.
\(\frac{4}{5}\)
Type below:
___________

Answer:
4 to 5
4 : 5

Explanation:
\(\frac{4}{5}\) using words 4 to 5
\(\frac{4}{5}\) with a colon 4 : 5

Question 2.
16 to 3
Type below:
___________

Answer:
\(\frac{16}{3}\)
16 : 3

Explanation:
16 to 3 as a fraction \(\frac{16}{3}\)
16 to 3 with a colon 16 : 3

Question 3.
9 : 13
Type below:
___________

Answer:
9 to 13
\(\frac{9}{13}\)

Explanation:
9 : 13 using words 9 to 13
9 : 13 as a fraction \(\frac{9}{13}\)

Question 4.
\(\frac{15}{8}\)
Type below:
___________

Answer:
15 to 8
15 : 8

Explanation:
\(\frac{15}{8}\) using words 15 to 8
\(\frac{15}{8}\) with a colon 15 : 8

Question 5.
There are 20 light bulbs in 5 packages. Complete the table to find the rate that gives the number of light bulbs in 3 packages. Write this rate in three different ways.
Go Math Grade 6 Answer Key Chapter 4 Model Ratios 16
Type below:
___________

Answer:
Grade 6 Chapter 4 image 12

Explanation:
There are 20 light bulbs in 5 packages.
For 1 package, 4 light bulbs available
For 2 package, 8 light bulbs available
For 3 package, 12 light bulbs available
For 4 package, 16 light bulbs available

Problem Solving

Question 6.
Gemma spends 4 hours each week playing soccer and 3 hours each week practicing her clarinet. Write the ratio of hours spent practicing clarinet to hours spent playing soccer three different ways.
Type below:
___________

Answer:
\(\frac{3}{4}\), 3 : 4, 3 to 4

Explanation:
Gemma spends 4 hours each week playing soccer and 3 hours each week practicing her clarinet.
3/4, 3 : 4, 3 to 4

Question 7.
Randall bought 2 game controllers at Electronics Plus for $36. What is the unit rate for a game controller at Electronics Plus?
Type below:
___________

Answer:
\(\frac{$18}{1}\)

Explanation:
Randall bought 2 game controllers at Electronics Plus for $36. $36/2 = $18/1 is the unit rate for a game controller at Electronics Plus

Question 8.
Explain how to determine if a given rate is also a unit rate.
Type below:
___________

Answer:
when rates are expressed as a quantity of 1, such as 2 feet per second or 5 miles per hour, they are called unit rates. If you have a multiple-unit rate such as 120 student for every 3 buses, an want to find the single-unit rate, write a ratio equal to the multiple-unit rate with 1 as the second term

Lesson Check – Page No. 222

Question 1.
At the grocery store, Luis bought 10 bananas and 4 apples. What are three different ways to write the ratio of apples to bananas?
Type below:
___________

Answer:
4 : 10, 4 to 10, \(\frac{4}{10}\)

Explanation:
At the grocery store, Luis bought 10 bananas and 4 apples. 4/10, 4 : 10, 4 to 10

Question 2.
Rita checked out 7 books from the library. She had 2 non-fiction books. The rest were fiction. What are three different ways to write the ratio of non-fiction to fiction?
Type below:
___________

Answer:
2 to 5, 2 : 5, \(\frac{2}{5}\)

Explanation:
Rita checked out 7 books from the library. She had 2 non-fiction books. The rest were fiction.
fiction = 5
2 to 5, 2 : 5, \(\frac{2}{5}\)

Spiral Review

Question 3.
McKenzie bought 1.2 pounds of coffee for $11.82. What was the cost per pound?
$ ______

Answer:
$9.85

Explanation:
McKenzie bought 1.2 pounds of coffee for $11.82. $11.82/1.2 = $9.85

Question 4.
Pedro has a bag of flour that weighs \(\frac{9}{10}\) pound. He uses \(\frac{2}{3}\) of the bag to make gravy. How many pounds of flour does Pedro use to make gravy?
\(\frac{□}{□}\) pound

Answer:
\(\frac{3}{5}\) pound

Explanation:
Pedro has a bag of flour that weighs \(\frac{9}{10}\) pound. He uses \(\frac{2}{3}\) of the bag to make gravy.
\(\frac{9}{10}\) × \(\frac{2}{3}\) = 3/5

Question 5.
Gina draws a map of her town on a coordinate plane. The point that represents the town’s civic center is 1 unit to the right of the origin and 4 units above it. What are the coordinates of the point representing the civic center?
Type below:
___________

Answer:
(-1, 4)

Explanation:
Gina draws a map of her town on a coordinate plane. The point that represents the town’s civic center is 1 unit to the right of the origin and 4 units above it.
(-1, 4)

Question 6.
Stefan draws these shapes. What is the ratio of triangles to stars?
Go Math Grade 6 Answer Key Chapter 4 Model Ratios 17
Type below:
___________

Answer:
2 to 5

Explanation:
There are 2 triangles and 5 stars. So, the ratio is 2 : 5

Share and Show – Page No. 225

Write two equivalent ratios.

Question 1.
Use a multiplication table to write two ratios that are equivalent to \(\frac{4}{7}\).
Type below:
___________

Answer:
\(\frac{4}{7}\) = \(\frac{8}{14}\), \(\frac{12}{21}\)

Explanation:
The original ratio is 4/7. Shade the row for 4 and the row for 7 on the multiplication table.
The column for 2 shows there are 2 ∙ 4, when there are 2 ∙ 7. So, 4/7 equal to 8/14
The column for 3 shows there are 3 ∙ 4, when there are 3 ∙ 7. So, 4/7 equal to 12/21

Question 2.
Go Math Grade 6 Answer Key Chapter 4 Model Ratios 18
Type below:
___________

Answer:
Grade 6 Chapter 4 image 18

Explanation:
The original ratio is 3/7. Shade the row for 3 and the row for 7 on the multiplication table.
The column for 2 shows there are 2 ∙ 3, when there are 2 ∙ 7. So, 3/7 equal to 6/14
The column for 3 shows there are 3 ∙ 3, when there are 3 ∙ 7. So, 3/7 equal to 9/21

Question 3.
Go Math Grade 6 Answer Key Chapter 4 Model Ratios 19
Type below:
___________

Answer:
Grade 6 Chapter 4 image 19

Explanation:
The original ratio is 5/2. Shade the row for 5 and the row for 2 on the multiplication table.
The column for 2 shows there are 2 ∙ 5 when there are 2 ∙ 2. So, 5/2 equal to 10/4
The column for 3 shows there are 3 ∙ 5 when there are 3 ∙ 2. So, 5/2 equal to 15/6

Question 4.
Go Math Grade 6 Answer Key Chapter 4 Model Ratios 20
Type below:
___________

Answer:
Grade 6 Chapter 4 image 20

Explanation:
The original ratio is 2/10. Shade the row for 2 and the row for 10 on the multiplication table.
The column for 1 shows there are 1 ∙ 2 when there are 5 ∙ 2. So, 2/10 equal to 1/5
The column for 3 shows there are 1 ∙ 3 when there are 5 ∙ 3. So, 2/10 equal to 3/15

Question 5.
\(\frac{4}{5}\)
Type below:
___________

Answer:
\(\frac{4}{5}\) = \(\frac{8}{10}\), \(\frac{12}{15}\)

Explanation:
The original ratio is 4/5. Shade the row for 4 and the row for 5 on the multiplication table.
The column for 2 shows there are 2 ∙ 4, when there are 2 ∙ 5. So, 4/5 equal to 8/10
The column for 3 shows there are 3 ∙ 4, when there are 3 ∙ 5. So, 4/5 equal to 12/15

Question 6.
\(\frac{12}{30}\)
Type below:
___________

Answer:
\(\frac{12}{30}\) = \(\frac{24}{60}\), \(\frac{36}{90}\)

Explanation:
The original ratio is 12/30. Shade the row for 12 and the row for 30 on the multiplication table.
The column for 2 shows there are 2 ∙ 12 when there are 2 ∙ 30. So, 12/30 equal to 24/60
The column for 3 shows there are 3 ∙ 12 when there are 3 ∙ 30. So, 12/30 equal to 36/90

Question 7.
\(\frac{2}{9}\)
Type below:
___________

Answer:
\(\frac{2}{9}\) = \(\frac{4}{18}\), \(\frac{6}{27}\)

Explanation:
The original ratio is 2/9. Shade the row for 2 and the row for 9 on the multiplication table.
The column for 2 shows there are 2 ∙ 2, when there are 2 ∙ 9. So, 2/9 equal to 4/18
The column for 3 shows there are 3 ∙ 2, when there are 3 ∙ 9. So, 2/9 equal to 6/27

On Your Own

Write two equivalent ratios.

Question 8.
Go Math Grade 6 Answer Key Chapter 4 Model Ratios 21
Type below:
___________

Answer:
Grade 6 Chapter 4 image 21

Explanation:
The original ratio is 9/8. Shade the row for 9 and the row for 8 on the multiplication table.
The column for 2 shows there are 2 ∙ 9 when there are 2 ∙ 8. So, 9/8 equal to 18/16
The column for 3 shows there are 3 ∙ 9 when there are 3 ∙ 8. So, 9/8 equal to 27/24

Question 9.
Go Math Grade 6 Answer Key Chapter 4 Model Ratios 22
Type below:
___________

Answer:
Grade 6 Chapter 4 image 22

Explanation:
The original ratio is 5/4. Shade the row for 5 and the row for 4 on the multiplication table.
The column for 2 shows there are 2 ∙ 5 when there are 2 ∙ 4. So, 5/4 equal to 10/8
The column for 3 shows there are 3 ∙ 5 when there are 3 ∙ 4. So, 5/4 equal to 15/20

Question 10.
Go Math Grade 6 Answer Key Chapter 4 Model Ratios 23
Type below:
___________

Answer:
Grade 6 Chapter 4 image 23

Explanation:
The original ratio is 6/9. Shade the row for 6 and the row for 9 on the multiplication table.
The column for 1 shows there are 1 ∙ 3 when there are 1. 4.5. So, 5/4 equal to 3/4.5
The column for 3 shows there are 3 ∙ 3 when there are 3 ∙ 4.5. So, 5/4 equal to 9/13.5

Question 11.
\(\frac{8}{7}\)
Type below:
___________

Answer:
\(\frac{8}{7}\) = \(\frac{16}{14}\), \(\frac{24}{21}\)

Explanation:
The original ratio is 8/7. Shade the row for 8 and the row for 7 on the multiplication table.
The column for 2 shows there are 2 ∙ 8, when there are 2 ∙ 7. So, 8/7 equal to 16/14
The column for 3 shows there are 3 ∙ 8, when there are 3 ∙ 7. So, 8/7 equal to 24/21

Question 12.
\(\frac{2}{6}\)
Type below:
___________

Answer:
\(\frac{2}{6}\) = \(\frac{4}{12}\), \(\frac{6}{18}\)

Explanation:
The original ratio is 2/6. Shade the row for 2 and the row for 6 on the multiplication table.
The column for 2 shows there are 2 ∙ 2, when there are 2 ∙ 6. So, 2/6 equal to 4/12
The column for 3 shows there are 3 ∙ 2, when there are 3 ∙ 6. So, 2/6 equal to 6/18

Question 13.
\(\frac{4}{11}\)
Type below:
___________

Answer:
\(\frac{4}{11}\) = \(\frac{8}{22}\), \(\frac{12}{33}\)

Explanation:
The original ratio is 4/11. Shade the row for 4 and the row for 11 on the multiplication table.
The column for 2 shows there are 2 ∙ 4, when there are 2 ∙ 11. So, 4/11 equal to 8/22
The column for 3 shows there are 3 ∙ 4, when there are 3 ∙ 11. So, 4/11 equal to 12/33

Determine whether the ratios are equivalent.

Question 14.
\(\frac{2}{3} \text { and } \frac{8}{12}\)
___________

Answer:
Yes

Explanation:
2/3 × 4/4 = 8/12
So, 2/3 is equal to 8/12

Question 15.
\(\frac{8}{10} \text { and } \frac{6}{10}\)
___________

Answer:
No

Explanation:
8/10 ÷ 2/2 = 4/5
8/10 is not equal to 6/10

Question 16.
\(\frac{16}{60} \text { and } \frac{4}{15}\)
___________

Answer:
yes

Explanation:
16/60 ÷ 4/4 = 4/15
16/60 is equal to 4/15

Question 17.
\(\frac{3}{14} \text { and } \frac{8}{28}\)
___________

Answer:
No

Explanation:
3/14 is not equal to 8/28

Problem Solving + Applications – Page No. 226

Use the multiplication table for 18 and 19.
Go Math Grade 6 Answer Key Chapter 4 Model Ratios 24

Question 18.
In Keith’s baseball games this year, the ratio of times he has gotten on base to the times he has been at bat is \(\frac{4}{14}\). Write two ratios that are equivalent to \(\frac{4}{14}\).
Type below:
___________

Answer:
\(\frac{4}{14}\) = \(\frac{8}{28}\), \(\frac{2}{7}\)

Explanation:
4/14
multiply both numbers by 2
8/28
divide both numbers by 2
2/7

Question 19.
Pose a Problem Use the multiplication table to write a new problem involving equivalent ratios. Then solve the problem.
Type below:
___________

Answer:
The ratio of times he has gotten on base to the times he has been at bat is \(\frac{6}{9}\). Write two ratios that are equivalent to \(\frac{6}{9}\)
.multiply both numbers by 2 = 12/18
multiply both numbers by 3 = 18/ 27

Question 20.
Describe how to write an equivalent ratio for \(\frac{9}{27}\) without using a multiplication table.
Type below:
___________

Answer:
\(\frac{9}{27}\) = \(\frac{18}{54}\), \(\frac{3}{9}\)

Explanation:
\(\frac{9}{27}\)
multiply both numbers by 2, 18/54
divide both numbers by 3
3/9

Question 21.
Write a ratio that is equivalent to \(\frac{6}{9} \text { and } \frac{16}{24}\).
\(\frac{□}{□}\)

Answer:
\(\frac{2}{3}\)

Explanation:
\(\frac{6}{9} \text { and } \frac{16}{24}\)
\(\frac{2}{3}\) is the equivalent ratio to \(\frac{6}{9} \text { and } \frac{16}{24}\)

Question 22.
Determine whether each ratio is equivalent to \(\frac{1}{3}, \frac{5}{10}, \text { or } \frac{3}{5}\). Write the ratio in the correct box.
Go Math Grade 6 Answer Key Chapter 4 Model Ratios 25
Type below:
___________

Answer:
3/9, 7/21, 18/30, 10/30

Explanation:
2/4 = 1/2
3/9 = 1/3
7/21 = 1/3
18/30 = 3/5
10/30 = 1/3
6/10 = 2/5
8/16 = 4/8 = 1/2

Equivalent Ratios and Multiplication Tables – Page No. 227

Write two equivalent ratios.

Question 1.
Use a multiplication table to write two ratios that are equivalent to \(\frac{5}{3}\).
Type below:
___________

Answer:
\(\frac{5}{3}\) = \(\frac{10}{6}\), \(\frac{15}{9}\)

Explanation:
The original ratio is 5/3. Shade the row for 5 and the row for 3 on the multiplication table.
The column for 2 shows there are 2 ∙ 5, when there are 2 ∙ 3. So, 5/3 equal to 10/6
The column for 3 shows there are 3 ∙ 5, when there are 3 ∙ 3. So, 5/3 equal to 15/9

Question 2.
Go Math Grade 6 Answer Key Chapter 4 Model Ratios 26
Type below:
___________

Answer:
Grade 6 Chapter 4 image 24

Explanation:
The original ratio is 6/7. Shade the row for 6 and the row for 7 on the multiplication table.
The column for 2 shows there are 2 ∙ 6 when there are 2 ∙ 7. So, 6/7 equal to 12/14
The column for 3 shows there are 3 ∙ 6 when there are 3 ∙ 7. So, 6/7 equal to 18/21

Question 2.
Go Math Grade 6 Answer Key Chapter 4 Model Ratios 27
Type below:
___________

Answer:
Grade 6 Chapter 4 image 25

Explanation:
The original ratio is 3/2. Shade the row for 3 and the row for 2 on the multiplication table.
The column for 2 shows there are 2 ∙ 3 when there are 2 ∙ 2. So, 3/2 equal to 6/4
Multiply 3/2 with 4/4 = 12/8

Question 4.
\(\frac{6}{8}\)
Type below:
___________

Answer:
\(\frac{6}{8}\) =\(\frac{12}{16}\), \(\frac{18}{24}\)

Explanation:
The original ratio is 6/8. Shade the row for 6 and the row for 8 on the multiplication table.
The column for 2 shows there are 2 ∙ 6, when there are 2 ∙ 8. So, 6/8 equal to 12/16
The column for 3 shows there are 3 ∙ 6, when there are 3 ∙ 8. So, 6/8 equal to 18/24

Question 5.
\(\frac{11}{1}\)
Type below:
___________

Answer:
\(\frac{11}{1}\) = \(\frac{22}{2}\), \(\frac{33}{3}\)

Explanation:
The original ratio is 11/1. Shade the row for 11 and the row for 1 on the multiplication table.
The column for 2 shows there are 2 ∙ 11, when there are 2 ∙ 1. So, 11/1 equal to 22/2
The column for 3 shows there are 3 ∙ 11, when there are 3 ∙ 1. So, 11/1 equal to 33/3

Determine whether the ratios are equivalent.

Question 6.
\(\frac{2}{3} \text { and } \frac{5}{6}\).
___________

Answer:
No

Explanation:
2/3 is not equal to 5/6

Question 7.
\(\frac{5}{10} \text { and } \frac{1}{6}\).
___________

Answer:
No

Explanation:
5/10 is not equal to 1/6

Question 8.
\(\frac{8}{3} \text { and } \frac{32}{12}\).
___________

Answer:
Yes

Explanation:
8/3 × 4/4 = 32/12
8/3 is equal to 32/12

Question 9.
\(\frac{9}{12} \text { and } \frac{3}{4}\).
___________

Answer:
Yes

Explanation:
9/12 ÷ 3/3 = 3/4
9/12 is equal to 3/4

Problem Solving

Question 10.
Tristan uses 7 stars and 9 diamonds to make a design. Write two ratios that are equivalent to \(\frac{7}{9}\).
Type below:
___________

Answer:
\(\frac{7}{9}\)  = \(\frac{14}{18}\) , \(\frac{21}{27}\)

Explanation:
Tristan uses 7 stars and 9 diamonds to make a design.
\(\frac{7}{9}\)
The original ratio is 7/9. Shade the row for 7 and the row for 9 on the multiplication table.
The column for 2 shows there are 2 ∙ 7, when there are 2 ∙ 9. So, 7/9 equal to 14/18
The column for 3 shows there are 3 ∙ 7, when there are 3 ∙ 9. So, 7/9 equal to 21/27

Question 11.
There are 12 girls and 16 boys in Javier’s math class. There are 26 girls and 14 boys in Javier’s choir class. Are the ratios of girls to boys in the two classes equivalent? Explain.
Type below:
___________

Answer:
No, the ratio 26/14 is not equal to the ratio 12/16

Question 12.
Explain how to determine whether two ratios are equivalent.
Type below:
___________

Answer:
If any ratio is multiplied or divided by the same number, then the ratios are equivalent.

Lesson Check – Page No. 228

Question 1.
A pancake recipe calls for 4 cups of flour and 3 cups milk. Does a recipe calling for 2 cups flour and 1.5 cups milk use the same ratio of flour to milk?
___________

Answer:
A muffin recipe that calls for 2 cups flour and 1.5 cups milk

Explanation:
A pancake recipe calls for 4 cups of flour and 3 cups milk. A muffin recipe that calls for 2 cups flour and 1.5 cups milk.

Question 2.
A bracelet is made of 14 red beads and 19 gold beads. A necklace is made of 84 red beads and 133 gold beads. Do the two pieces of jewelry have the same ratio of red beads to gold beads?
___________

Answer:
The bracelet has 14 red and 19 gold, so the ratio between red and gold is 14/19. We cannot simplify this ratio as there are not common factors between 14 and 19, because 19 is a prime number.
As there are 84 red and 133 gold the ratio will be 84/133. For this ratio to be equal to 14/19 it should be that 84 is multiple of 14 and 133 multiple of 19, and both multiples must the same,
84/133 is not equal to 14/19

Spiral Review

Question 3.
Scissors come in packages of 3. Glue sticks come in packages of 10. Martha wants to buy the same number of each. What is the fewest glue sticks Martha can buy?
_____ glue sticks

Answer:
30 glue sticks

Explanation:
Scissors come in packages of 3. Glue sticks come in packages of 10. Martha wants to buy the same number of each.
3 × 10 = 30 glue sticks

Question 4.
Cole had \(\frac{3}{4}\) hour of free time before dinner. He spent \(\frac{2}{3}\) of the time playing the guitar. How long did he play the guitar?
\(\frac{□}{□}\) hour

Answer:
\(\frac{1}{2}\) hour

Explanation:
Cole had \(\frac{3}{4}\) hour of free time before dinner. He spent \(\frac{2}{3}\) of the time playing the guitar.
\(\frac{2}{3}\) × \(\frac{3}{4}\) = 1/2 hour

Question 5.
Delia has 3 \(\frac{5}{8}\) yards of ribbon. About how many \(\frac{1}{4}\)-yard-long pieces can she cut?
About _____ pieces

Answer:
About 14 pieces

Explanation:
Length of yards of ribbon is 3 5/8 = 29/8
Length of yards of ribbon pieces need to be cut is 1/4
Number of yards = 29/8 ÷ 1/4 = 14.5 = 14

Question 6.
Which point is located at –1.1?
Go Math Grade 6 Answer Key Chapter 4 Model Ratios 28
Type below:
___________

Answer:
B

Explanation:
-1.1 is in between -1 and -2
-1.1 is close to -1
So, the answer is point B

Share and Show – Page No. 231

Question 1.
In Jawan’s school, 4 out of 10 students chose basketball as a sport they like to watch, and 3 out of 5 students chose football. Is the ratio of students who chose basketball (4 to 10) equivalent to the ratio of students who chose football (3 to 5)?
Type below:
___________

Answer:
the ratio of students who chose basketball (4 to 10) is not equivalent to the ratio of students who chose football (3 to 5)

Explanation:
In Jawan’s school, 4 out of 10 students chose basketball as a sport they like to watch, and 3 out of 5 students chose football.
4/10 = 0.4
3/5 = 0.6
0.4 is not equal to 0.6
The ratio of students who chose basketball (4 to 10) is not equivalent to the ratio of students who chose football (3 to 5)

Question 2.
What if 20 out of 50 students chose baseball as a sport they like to watch? Is this ratio equivalent to the ratio for either basketball or football? Explain.
Type below:
___________

Answer:
The baseball ratio is equal to the basketball ratio

Explanation:
If 20 out of 50 students chose baseball, 20/50 = 2/5
2/5 × 2/2 = 4/10
The baseball ratio is equal to the basketball ratio.

Question 3.
Look for Structure The table shows the results of the quizzes Hannah took in one week. Did Hannah get the same score on her math and science quizzes? Explain.
Go Math Grade 6 Answer Key Chapter 4 Model Ratios 29
Type below:
___________

Answer:
Hannah didn’t get the same score on her math and science quizzes

Explanation:
Social Studies = 4/5
Math = 8/10 = 0.8
Science = 3/4 = 0.75
English = 10/12
Math = 8/10
Divide the 8/10 with 2/2 = 8/10 ÷ 2/2 = 4/5
Hannah didn’t get the same score on her math and science quizzes

Question 4.
Did Hannah get the same score on the quizzes in any of her classes? Explain.
Type below:
___________

Answer:
The ratio of Social Studies is equal to the ratio of Math

Explanation:
Social Studies = 4/5 = 0.8
Math = 8/10 = 0.8
Science = 3/4 = 0.75
English = 10/12 = 0.8333
The ratio of Social Studies is equal to the ratio of Math

On Your Own – Page No. 232

Question 5.
For every $10 that Julie makes, she saves $3. For every $15 Liam makes, he saves $6. Is Julie’s ratio of money saved to money earned equivalent to Liam’s ratio of money saved to money earned?
Type below:
___________

Answer:
Julie’s ratio of money saved to money earned is not equivalent to Liam’s ratio of money saved to money earned.

Explanation:
No. Julie’s ratio is 3:10 or 30 percent towards her savings while Lion’s is 6:15 which is 40 percent towards savings.

Question 6.
A florist offers three different bouquets of tulips and irises. The list shows the ratios of tulips to irises in each bouquet. Determine the bouquets that have equivalent ratios.
Go Math Grade 6 Answer Key Chapter 4 Model Ratios 30
Type below:
___________

Answer:
The ratio of Spring Mix is equal to the ratio of Splash of Sun

Explanation:
Spring Mix = 4/6 = 0.66
Morning Melody = 9/12 = 0.75
Splash of Sun = 10/15 =0.66
The ratio of Spring Mix is equal to the ratio of Splash of Sun

Question 7.
The ratio of boys to girls in a school’s soccer club is 3 to 5. The ratio of boys to girls in the school’s chess club is 13 to 15. Is the ratio of boys to girls in the soccer club equivalent to the ratio of boys to girls in the chess club? Explain
Type below:
___________

Answer:
No

Explanation:
They are not equivalent because you can not reduce 13 any further because it is a prime number and if you multiply 3 by 3 and 5 by 3 you would get 9:15 as the equivalent ratio.

Question 8.
Analyze Thad, Joey, and Mia ran in a race. The finishing times were 4.56 minutes, 3.33 minutes, and 4.75 minutes. Thad did not finish last. Mia had the fastest time. What was each runner’s time?
Type below:
___________

Answer:
Mia = 3.33 minutes
Joey = 4.75 minutes
Thad = 4.56 minutes

Explanation:
Mia had the fastest time. 3.33 minutes
Thad did not finish last. So, Joey = 4.75 minutes
Thad = 4.56 minutes

Question 9.
Fernando donates $2 to a local charity organization for every $15 he earns. Cleo donates $4 for every $17 she earns. Is Fernando’s ratio of money donated to money earned equivalent to Cleo’s ratio of money donated to money earned? Explain.
Type below:
___________

Answer:
Fernando’s ratio of money donated to money earned is not equivalent to Cleo’s ratio of money donated to money earned

Explanation:
Fernando donates $2 to a local charity organization for every $15 he earns.
$2/$15 = 0.1333
Cleo donates $4 for every $17 she earns. $4/$17 = 0.2359
Fernando’s ratio of money donated to money earned is not equivalent to Cleo’s ratio of money donated to money earned

Problem Solving Use Tables to Compare Ratios – Page No. 233

Read each problem and solve.

Question 1.
Sarah asked some friends about their favorite colors. She found that 4 out of 6 people prefer blue, and 8 out of 12 people prefer green. Is the ratio of friends who chose blue to the total asked equivalent to the ratio of friends who chose green to the total asked?
Type below:
___________

Answer:
Yes, 4/6 is equivalent to 8/12

Explanation:
Grade 6 Chapter 4 image 26
4/6 = 0.666
8/12 = 0.666

Question 2.
Lisa and Tim make necklaces. Lisa uses 5 red beads for every 3 yellow beads. Tim uses 9 red beads for every 6 yellow beads. Is the ratio of red beads to yellow beads in Lisa’s necklace equivalent to the ratio in Tim’s necklace?
Type below:
___________

Answer:
The ratio of red beads to yellow beads in Lisa’s necklace is not equivalent to the ratio in Tim’s necklace

Explanation:
Lisa and Tim make necklaces. Lisa uses 5 red beads for every 3 yellow beads.
5/3 = 1.666
Tim uses 9 red beads for every 6 yellow beads. 9/6 = 1.5
The ratio of red beads to yellow beads in Lisa’s necklace is not equivalent to the ratio in Tim’s necklace

Question 3.
Mitch scored 4 out of 5 on a quiz. Demetri scored 8 out of 10 on a quiz. Did Mitch and Demetri get equivalent scores?
Type below:
___________

Answer:
Mitch and Demetri get equivalent scores

Explanation:
Mitch scored 4 out of 5 on a quiz. 4/5 = 0.8
Demetri scored 8 out of 10 on a quiz. = 8/10 = 0.8
Mitch and Demetri get equivalent scores

Question 4.
Use tables to show which of these ratios are equivalent : \(\frac{4}{6}\), \(\frac{10}{25}, \text { and } \frac{6}{15}\).
Type below:
___________

Answer:
\(\frac{10}{25}, \text { and } \frac{6}{15}\) are equal

Explanation:
\(\frac{4}{6}\) = 0.6666
\(\frac{10}{25}\) = 0.4
\(\frac{6}{15}\) = 0.4
\(\frac{10}{25}, \text { and } \frac{6}{15}\) are equal

Page No. 234

Question 1.
Mrs. Sahd distributes pencils and paper to students in the ratio of 2 pencils to 10 sheets of paper. Three of these ratios are equivalent to \(\frac{2}{10}\). Which one is NOT equivalent?
\(\frac{1}{5} \frac{7}{15} \frac{4}{20} \frac{8}{40}\)
Type below:
___________

Answer:
\(\frac{7}{15}\) is not equal \(\frac{2}{10}\)

Explanation:
Mrs. Sahd distributes pencils and paper to students in the ratio of 2 pencils to 10 sheets of paper. Three of these ratios are equivalent to \(\frac{2}{10}\) = 0.2
\(\frac{1}{5}\) = 0.2
\(\frac{7}{15}\) = 0.4666
\(\frac{4}{20}\) = 0.2
\(\frac{8}{40}\) = 0.2
\(\frac{7}{15}\) is not equal \(\frac{2}{10}\)

Question 2.
Keith uses 18 cherries and 3 peaches to make a pie filling. Lena uses an equivalent ratio of cherries to peaches when she makes pie filling. Can Lena use a ratio of 21 cherries to 6 peaches? Explain.
Type below:
___________

Answer:
No, she cannot use a ratio of 21 cherries to 6 peaches

Explanation:
Keith uses 18 cherries and 3 peaches to make a pie filling. 18/3 = 6
Lena uses a ratio of 21 cherries to 6 peaches, 21/6 = 3.5
No, she cannot use a ratio of 21 cherries to 6 peaches

Spiral Review

Question 3.
What is the quotient \(\frac{3}{20} \div \frac{7}{10}\)?
Type below:
___________

Answer:
\(\frac{3}{14}\)

Explanation:
\(\frac{3}{20} \div \frac{7}{10}\)
3/20 × 10/7 = 3/14

Question 4.
Which of these numbers is greater than – 2.25 but less than –1?
1 -1.5 0 -2.5
Type below:
___________

Answer:

Explanation:
1 lies between 0 to 1
-1.5 lies between -1 and -2. It is greater than -2.25 and also less than -1
0 lies between -1 to 1
-2.5 lies between -2 and -3. -2.5 is less than -2.25

Question 5.
Alicia plots a point at (0, 5) and (0, –2). What is the distance between the points?
Type below:
___________

Answer:
7 units

Explanation:
Alicia plots a point at (0, 5) and (0, –2).
The given points have the same x-coordinates.
|-2| = 2
5 + 0 = 5
0 + 2 = 2
5 + 2 = 7
The distance is 7 units

Question 6.
Morton sees these stickers at a craft store. What is the ratio of clouds to suns?
Go Math Grade 6 Answer Key Chapter 4 Model Ratios 31
Type below:
___________

Answer:
3 : 2

Explanation:
there are 3 clouds and 2 suns. So, the ratio is 3 to 2.

Share and Show – Page No. 237

Use equivalent ratios to find the unknown value.

Question 1.
\(\frac{?}{10}=\frac{4}{5}\)
_____

Answer:
\(\frac{8}{10}\) = \(\frac{4}{5}\)

Explanation:
Use common denominators to write equivalent ratios.
10 is a multiple of 5, so 10 is a common denominator.
Multiply the 4 and denominator by 2 to write the ratios using a common denominator.
4/5 × 2/2 = 8/10
The denominators are the same, so the numerators are equal to each other.
So, the unknown value is 8/10 = 4/5
\(\frac{8}{10}\)

Question 2.
\(\frac{18}{24}=\frac{6}{?}\)
_____

Answer:
\(\frac{6}{8}\) = \(\frac{18}{24}\)

Explanation:
Write an equivalent ratio with 18 in the numerator.
Divide 18 by 6 to get 3
So, divide the denominator by 24 as well.
24/3 = 8
The numerators are the same, so the denominators are equal to each other.
So, the unknown value is 6/8 = 18/24
\(\frac{6}{8}\)

Question 3.
\(\frac{3}{6}=\frac{15}{?}\)
_____

Answer:
\(\frac{15}{30}\)

Explanation:
Write an equivalent ratio with 15 in the numerator.
Multiply 3 with 5 to get 15
So, Multiply 6 with 5 to get the denominator of unknown number.
6 × 5 = 30
The numerators are the same, so the denominators are equal to each other.
So, the unknown value is 3/6 = 15/30
\(\frac{15}{30}\)

Question 4.
\(\frac{?}{5}=\frac{8}{10}\)
_____

Answer:
\(\frac{4}{5}\)

Explanation:
Write an equivalent ratio with 10 in the denominator.
Divide 10 by 2 to get 5
So, divide the numerator 8 as well.
8/2 = 4
The denominators are the same, so the numerators are equal to each other.
So, the unknown value is 8/10 = 4/5
\(\frac{4}{5}\)

Question 5.
\(\frac{7}{4}=\frac{?}{12}\)
_____

Answer:
\(\frac{21}{12}\)

Explanation:
Write an equivalent ratio with 12 in the denominator.
Multiply 4 with 3 to get 12
So, Multiply 7 with 3 to get the numerator of unknown number.
7 × 3 = 21
The denominators are the same, so the numerators are equal to each other.
So, the unknown value is 21/12 = 7/4
\(\frac{21}{12}\)

Question 6.
\(\frac{10}{?}=\frac{40}{12}\)
_____

Answer:
\(\frac{10}{3}\)

Explanation:
Write an equivalent ratio with 40 in the numerator.
Divide 40 by 4 to get 10
So, divide the denominator 12 as well.
12/4 = 3
The numerators are the same, so the denominators are equal to each other.
So, the unknown value is 10/3 = 40/12
\(\frac{10}{3}\)

On Your Own

Use equivalent ratios to find the unknown value.

Question 7.
\(\frac{2}{6}=\frac{?}{30}\)
_____

Answer:
\(\frac{10}{30}\)

Explanation:
Use common denominators to write equivalent ratios.
30 is a multiple of 6, so 30 is a common denominator.
Multiply the 6 and denominator by 5 to write the ratios using a common denominator.
2/6 × 5/5 =10/30
The denominators are the same, so the numerators are equal to each other.
So, the unknown value is 10/30 = 2/6
\(\frac{10}{30}\)

Question 8.
\(\frac{5}{?}=\frac{55}{110}\)
_____

Answer:
\(\frac{5}{10}\)

Explanation:
Write an equivalent ratio with 55 in the numerator.
Divide 55 with 11 to get 5
So, Divide 110 with 11 to get the denominator of unknown number.
110/11 = 10
The numerators are the same, so the denominators are equal to each other.
So, the unknown value is 5/10 = 55/110
\(\frac{5}{10}\)

Question 9.
\(\frac{3}{9}=\frac{9}{?}\)
_____

Answer:
\(\frac{9}{27}\)

Explanation:
Write an equivalent ratio with 9 in the numerator.
Multiply 3 with 3 to get 9
So, Multiply 9 with 3 to get the denominator of unknown number.
9 × 3 = 27
The numerators are the same, so the denominators are equal to each other.
So, the unknown value is 9/27 = 3/9
\(\frac{9}{27}\)

Question 10.
\(\frac{?}{6}=\frac{16}{24}\)
_____

Answer:
\(\frac{4}{6}\)

Explanation:
Use common denominators to write equivalent ratios.
Divide 24 with 4 to get 6.
So, divide 16 with 4 to know the unknown number of numerator
16/4 = 4
The denominators are the same, so the numerators are equal to each other.
So, the unknown value is 4/6 = 16/24
\(\frac{4}{6}\)

Question 11.
Mavis walks 3 miles in 45 minutes. How many minutes will it take Mavis to walk 9 miles?
_____ minutes

Answer:
135 minutes

Explanation:
Mavis walks 3 miles in 45 minutes.
For 9 miles, (9 × 45)/3 = 135 minutes

Question 12.
The ratio of boys to girls in a choir is 3 to 8. There are 32 girls in the choir. How many members are in the choir?
_____ members

Answer:
12 members

Explanation:
The ratio of boys to girls in a choir is 3 to 8.
3/8 × 4/ 4 = 12/32
So, if there are 32 girls in the choir, there will be 12 boys present.

Question 13.
Use Reasoning Is the unknown value in \(\frac{2}{3}=\frac{?}{18}\) the same as the unknown value in \(\frac{3}{2}=\frac{18}{?}\)? Explain.
Type below:
___________

Answer:
12

Explanation:
\(\frac{2}{3}=\frac{?}{18}\)
2/3 × 6/6 = 12/18
the unknown value is 12
\(\frac{3}{2}=\frac{18}{?}\)
3/2 × 6/6 = 18/12
the unknown value is 12

Problem Solving + Applications – Page No. 238

Solve by finding an equivalent ratio.

Question 14.
It takes 8 minutes for Sue to make 2 laps around the go-kart track. How many laps can Sue complete in 24 minutes?
_____ laps

Answer:
6 laps

Explanation:
It takes 8 minutes for Sue to make 2 laps around the go-kart track.
For 24 minutes, (24 × 2)/8 = 48/8 =6

Question 15.
The width of Jay’s original photo is 8 inches. The length of the original photo is 10 inches. He prints a smaller version that has an equivalent ratio of width to length. The width of the smaller version is 4 inches less than the width of the original. What is the length of the smaller version?
_____ inches

Answer:
5 inches

Explanation:
The width of Jay’s original photo is 8 inches. The length of the original photo is 10 inches.
8/10
He prints a smaller version that has an equivalent ratio of width to length. The width of the smaller version is 4 inches less than the width of the original.
4/s
8/10 ÷ 2/2 = 4/5
5 inches

Question 16.
Ariel bought 3 raffle tickets for $5. How many tickets could Ariel buy for $15?
_____ tickets

Answer:
9 tickets

Explanation:
Ariel bought 3 raffle tickets for $5.
For $15, ($15 × 3)/ $5 = 45/5 = 9

Question 17.
What’s the Error? Greg used the steps shown to find the unknown value. Describe his error and give the correct solution.
\(\frac{2}{6}=\frac{?}{12}\)
\(\frac{2+6}{6+6}=\frac{?}{12}\)
\(\frac{8}{12}=\frac{?}{12}\)
The unknown value is 8.
Type below:
___________

Answer:
Greg added 6 to the numerator and denominator which is not correct to find the unknown value.
\(\frac{2}{6}=\frac{?}{12}\)
2/6 × 2/2 = 4/12
4 is the unknown value.

Question 18.
Courtney bought 3 maps for $10. Use the table of equivalent ratios to find how many maps she can buy for $30.
Go Math Grade 6 Answer Key Chapter 4 Model Ratios 32
Type below:
___________

Answer:
Grade 6 Chapter 4 image 28

Explanation:
3/10 × 3/3 = 9/30

Use Equivalent Ratios – Page No. 239

Use equivalent ratios to find the unknown value.

Question 1.
\(\frac{4}{10}=\frac{?}{40}\)
_____

Answer:
\(\frac{16}{40}\)

Explanation:
4/10 × 4/4 = 16/40

Question 2.
\(\frac{3}{24}=\frac{33}{?}\)
_____

Answer:
\(\frac{33}{264}\)

Explanation:
3/24 × 11/11 = 33/264

Question 3.
\(\frac{7}{?}=\frac{21}{27}\)
_____

Answer:
\(\frac{7}{9}\)

Explanation:
21/27 ÷ 3/3 = 7/9

Question 4.
\(\frac{?}{9}=\frac{12}{54}\)
_____

Answer:
\(\frac{2}{9}\)

Explanation:
12/54 ÷ 6/6 = 2/9

Question 5.
\(\frac{3}{2}=\frac{12}{?}\)
_____

Answer:
\(\frac{12}{8}\)

Explanation:
3/2 × 4/4 = 12/8

Question 6.
\(\frac{4}{5}=\frac{?}{40}\)
_____

Answer:
\(\frac{32}{40}\)

Explanation:
4/5 × 8/8 = 32/40

Question 7.
\(\frac{?}{2}=\frac{45}{30}\)
_____

Answer:
\(\frac{3}{2}\)

Explanation:
45/30 ÷ 15/15 = 3/2

Question 8.
\(\frac{45}{?}=\frac{5}{6}\)
_____

Answer:
\(\frac{45}{54}\)

Explanation:
5/6 × 9/9 = 45/54

Problem Solving

Question 9.
Honeybees produce 7 pounds of honey for every 1 pound of beeswax they produce. Use equivalent ratios to find how many pounds of honey are produced when 25 pounds of beeswax are produced.
_____ pounds

Answer:
175 pounds

Explanation:
Honeybees produce 7 pounds of honey for every 1 pound of beeswax they produce.
7/1
25 pounds of beeswax, 25 × 7 = 175 pounds

Question 10.
A 3-ounce serving of tuna provides 21 grams of protein. Use equivalent ratios to find how many grams of protein are in 9 ounces of tuna.
_____ grams of protein

Answer:
63 grams of protein

Explanation:
A 3-ounce serving of tuna provides 21 grams of protein.
For 9 ounces of tuna, (21 × 9)/3 = 63

Question 11.
Explain how using equivalent ratios is like adding fractions with unlike denominators.
Type below:
___________

Answer:
Equivalent ratios have different numbers but represent the same relationship. In this tutorial, you’ll see how to find equivalent ratios by first writing the given ratio as a fraction. And it cannot be the same by adding tow fraction with different ratio

Lesson Check – Page No. 240

Question 1.
Jaron paid $2.70 for 6 juice boxes. How much should Jaron expect to pay for 18 juice boxes?
$ _____

Answer:
$8.1

Explanation:
Jaron paid $2.70 for 6 juice boxes. For 6 boxes he paid $2.70.
For 18 juice boxes, (18 × $2.70)/6 = $8.1

Question 2.
A certain shade of orange paint is made by mixing 3 quarts of red paint with 2 quarts of yellow paint. To make more paint of the same shade, how many quarts of yellow paint should be mixed with 6 quarts of red paint?
_____ quarts

Answer:
4 quarts

Explanation:
A certain shade of orange paint is made by mixing 3 quarts of red paint with 2 quarts of yellow paint.
3 quarts of red paint is mixed with 2 quarts of yellow paint
So, 6 quarts of red paint is mixed with 6/3 × 2 = 4 quarts of yellow paint

Spiral Review

Question 3.
What is the quotient \(2 \frac{4}{5} \div 1 \frac{1}{3}\)?
______ \(\frac{□}{□}\)

Answer:
2\(\frac{1}{10}\)

Explanation:
2 4/5 = 14/5 = 2.8
1 1/3 = 4/3 = 1.333
2.8/1.333 = 2 1/10

Question 4.
What is the quotient \(-2 \frac{2}{3}\)?
______ \(\frac{□}{□}\)

Answer:
1\(\frac{11}{16}\)

Explanation:
−4 1/2 ÷ -2 2/3
1 11/16

Question 5.
On a map, a clothing store is located at (–2, –3). A seafood restaurant is located 6 units to the right of the clothing store. What are the coordinates of the restaurant?
Type below:
___________

.Answer:
(4, -3)

Explanation:
On a map, a clothing store is located at (–2, –3). A seafood restaurant is located 6 units to the right of the clothing store.
|-2| = 2
2 + 0 = 2
0+4 = 4
2 + 4 = 6 units

Question 6.
Marisol plans to make 9 mini-sandwiches for every 2 people attending her party. Write a ratio that is equivalent to Marisol’s ratio.
Type below:
___________

Answer:
27/6 and 45/10

Explanation:
Marisol plans to make 9 mini-sandwiches for every 2 people attending her party. 9/2 × 3/3 = 27/6
9/2 × 5/5 = 45/10

Mid-Chapter Checkpoint – Vocabulary – Page No. 241

Choose the best term from the box to complete the sentence.
Go Math Grade 6 Answer Key Chapter 4 Model Ratios 33

Question 1.
A _____ is a rate that makes a comparison to 1 unit.
Type below:
___________

Answer:
rate

Question 2.
Two ratios that name the same comparison are _____ .
Type below:
___________

Answer:
Equivalent Ratios

Concepts and Skills
Go Math Grade 6 Answer Key Chapter 4 Model Ratios 34

Question 3.
Write the ratio of red circles to blue squares.
Type below:
___________

Answer:
3 : 5

Explanation:
There are 3 red counter and 5 square boxes.
So, the ratio is 3 : 5

Write the ratio in two different ways.

Question 4.
8 to 12
Type below:
___________

Answer:
\(\frac{8}{12}\)
8 : 12

Explanation:
8 to 12 as a fraction  \(\frac{8}{12}\)
8 to 12 with a colon 8 : 12

Question 5.
7 : 2
Type below:
___________

Answer:
\(\frac{7}{2}\)
7 to 2

Explanation:
7 : 2 as a fraction  \(\frac{7}{2}\)
7 : 2 using words 7 to 2

Question 6.
\(\frac{5}{9}\)
Type below:
___________

Answer:
5 to 9
5 : 9

Explanation:
\(\frac{5}{9}\) using words 5 to 9
\(\frac{5}{9}\) with a colon 5 : 9

Question 7.
11 to 3
Type below:
___________

Answer:
\(\frac{11}{3}\)
11 : 3

Explanation:
11 to 3 as a fraction \(\frac{11}{3}\)
11 to 3 with a colon 11 : 3

Write two equivalent ratios.

Question 8.
\(\frac{2}{7}\)
Type below:
___________

Answer:
\(\frac{2}{7}\) = \(\frac{4}{14}\), \(\frac{6}{21}\)

Explanation:
The original ratio is 2/7. Shade the row for 2 and the row for 7 on the multiplication table.
The column for 2 shows there are 2 ∙ 2, when there are 2 ∙ 7. So, 2/7 equal to 4/14
The column for 3 shows there are 3 ∙ 2, when there are 3 ∙ 7. So, 2/7 equal to 6/21

Question 9.
\(\frac{6}{5}\)
Type below:
___________

Answer:
\(\frac{6}{5}\) = \(\frac{12}{10}\), \(\frac{18}{15}\)

Explanation:
The original ratio is 6/5. Shade the row for 6 and the row for 5 on the multiplication table.
The column for 2 shows there are 2 ∙ 6, when there are 2 ∙ 5. So, 6/5 equal to 12/10
The column for 3 shows there are 3 ∙ 6, when there are 3 ∙ 5. So, 6/5 equal to 18/15

Question 10.
\(\frac{9}{12}\)
Type below:
___________

Answer:
\(\frac{9}{12}\) = \(\frac{18}{24}\), \(\frac{27}{36}\)

Explanation:
The original ratio is 9/12. Shade the row for 9 and the row for 12 on the multiplication table.
The column for 2 shows there are 2 ∙ 9, when there are 2 ∙ 12. So, 9/12 equal to 18/24
The column for 3 shows there are 3 ∙ 9, when there are 3 ∙ 12. So, 9/12 equal to 27/36

Question 11.
\(\frac{18}{6}\)
Type below:
___________

Answer:
\(\frac{18}{6}\) = \(\frac{36}{12}\), \(\frac{54}{18}\)

Explanation:
The original ratio is 18/6. Shade the row for 18 and the row for 6 on the multiplication table.
The column for 2 shows there are 2 ∙ 18, when there are 2 ∙ 6. So, 18/6 equal to 36/12
The column for 3 shows there are 3 ∙ 18, when there are 3 ∙ 6. So, 18/6 equal to 54/18

Find the unknown value.

Question 12.
\(\frac{15}{?}=\frac{5}{10}\)
Type below:
___________

Answer:
30

Explanation:
5/10 × 3/3 = 15/30
So, the unknown number is 30

Question 13.
\(\frac{?}{9}=\frac{12}{3}\)
Type below:
___________

Answer:
36

Explanation:
12/3 × 3/3 = 36/9
So, the unknown number is 36

Question 14.
\(\frac{48}{16}=\frac{?}{8}\)
Type below:
___________

Answer:
24

Explanation:
48/16 ÷ 2/2 = 24/8
So, the unknown number is 24

Question 15.
\(\frac{9}{36}=\frac{3}{?}\)
Type below:
___________

Answer:
12

Explanation:
9/36 ÷ 3/3 = 3/12
So, the unknown number is 12

Page No. 242

Question 16.
There are 36 students in the chess club, 40 students in the drama club, and 24 students in the film club. What is the ratio of students in the drama club to students in the film club?
Type below:
___________

Answer:
40 to 24

Explanation:
There are 36 students in the chess club, 40 students in the drama club, and 24 students in the film club.
The ratio of students in the drama club to students in the film club 40 to 24

Question 17.
A trail mix has 4 cups of raisins, 3 cups of dates, 6 cups of peanuts, and 2 cups of cashews. Which ingredients are in the same ratio as cashews to raisins?
Type below:
___________

Answer:
dates to peanuts

Explanation:
A trail mix has 4 cups of raisins, 3 cups of dates, 6 cups of peanuts, and 2 cups of cashews.
cashews to raisins = 2/4 = 1/2
dates to peanuts = 3/6 = 1/2

Question 18.
There are 32 adults and 20 children at a school play. What is the ratio of children to people at the school play?
Type below:
___________

Answer:
5 to 13

Explanation:
There are 32 adults and 20 children at a school play.
people = 32 + 20 = 52
the ratio of children to people at the school play = 20/52 = 5/13

Question 19.
Sonya got 8 out of 10 questions right on a quiz. She got the same score on a quiz that had 20 questions. How many questions did Sonya get right on the second quiz? How many questions did she get wrong on the second quiz?
Type below:
___________

Answer:
4 wrong

Explanation:
8/10 = x/20
So, 10 × 2 = 20, so 8 × 2=16
so she got 16 out of 20 right and 20 – 16 = 4
She got 4 wrong.

Share and Show – Page No. 245

Write the rate as a fraction. Then find the unit rate.

Question 1.
Sara drove 72 miles on 4 gallons of gas.
_____ miles/gallon

Answer:
18 miles/gallon

Explanation:
Sara drove 72 miles on 4 gallons of gas.
72/4
Divide 72/4 with 4/4
72/4 ÷ 4/4 = 18

Question 2.
Dean paid $27.00 for 4 movie tickets.
$ _____ per ticket

Answer:
$6.75 per ticket

Explanation:
Dean paid $27.00 for 4 movie tickets.
$27.00/4
Divide $27.00/4 with 4/4
$27.00/4 ÷ 4/4 = $6.75

Question 3.
Amy and Mai have to read Bud, Not Buddy for a class. Amy reads 20 pages in 2 days. Mai reads 35 pages in 3 days. Who reads at a faster rate?
___________

Answer:
Mai reads at a faster rate

Explanation:
Amy and Mai have to read Bud, Not Buddy for a class.
Amy reads 20 pages in 2 days. 20/2 = 10 pages for each day
Mai reads 35 pages in 3 days. 35/3 = 11.66 pages for each day
Mai reads at a faster rate

Question 4.
An online music store offers 5 downloads for $6.25. Another online music store offers 12 downloads for $17.40. Which store offers the better deal?
___________

Answer:
An online music store offers 5 downloads for $6.25 offers the better deal

Explanation:
An online music store offers 5 downloads for $6.25.
$6.25/5 = $1.25
Another online music store offers 12 downloads for $17.40.
$17.40/12 = $1.45
An online music store offers 5 downloads for $6.25 offers the better deal

On Your Own

Write the rate as a fraction. Then find the unit rate.

Question 5.
A company packed 108 items in 12 boxes.
Type below:
___________

Answer:
9

Explanation:
A company packed 108 items in 12 boxes.
108/12
Divide 108/12 with 12/12
108/12 ÷ 12/12 = 9

Question 6.
There are 112 students for 14 teachers.
Type below:
___________

Answer:
8

Explanation:
There are 112 students for 14 teachers.
112/14
Divide 112/14 with 14/14
112/14 ÷ 14/14 = 8

Question 7.
Geoff charges $27 for 3 hours of swimming lessons. Anne charges $31 for 4 hours. How much more does Geoff charge per hour than Anne?
$ _____

Answer:
$1.25

Explanation:
Geoff charges $27 for 3 hours of swimming lessons.
$27/3 = $9 for an hour
Anne charges $31 for 4 hours.
$31/4 = $7.75
$9 – $7.75 = $1.25
Geoff charge $1.25 per hour more than Anne

Question 8.
Compare One florist made 16 bouquets in 5 hours. A second florist made 40 bouquets in 12 hours. Which florist makes bouquets at a faster rate?
Type below:
___________

Answer:
A second florist made 40 bouquets in 12 hours at a faster rate

Explanation:
Compare One florist made 16 bouquets in 5 hours.
16/5 = 3.2
A second florist made 40 bouquets in 12 hours.
40/12 = 3.333
A second florist made 40 bouquets in 12 hours at a faster rate

Tell which rate is faster by comparing unit rates.

Question 9.
\(\frac{160 \mathrm{mi}}{2 \mathrm{hr}} \text { and } \frac{210 \mathrm{mi}}{3 \mathrm{hr}}\)
Type below:
___________

Answer:
160mi/2hr

Explanation:
160mi/2hr ÷ 2/2 = 80mi/hr
210mi/3hr = 70mi/hr
80mi/hr > 70mi/hr

Question 10.
\(\frac{270 \mathrm{ft}}{9 \mathrm{min}} \text { and } \frac{180 \mathrm{ft}}{9 \mathrm{min}}\)
Type below:
___________

Answer:
270ft/9min

Explanation:
270ft/9min = 30ft/min
180ft/9min = 20ft/min
30ft/min > 20ft/min

Question 11.
\(\frac{250 \mathrm{m}}{10 \mathrm{s}} \text { and } \frac{120 \mathrm{m}}{4 \mathrm{s}}\)
Type below:
___________

Answer:
250m/10s

Explanation:
250m/10s = 25m/s
120m/4s = 20m/s
25m/s > 20m/s

Unlock the Problem – Page No. 246

Question 12.
Ryan wants to buy treats for his puppy. If Ryan wants to buy the treats that cost the least per pack, which treat should he buy? Explain.
Go Math Grade 6 Answer Key Chapter 4 Model Ratios 35
a. What do you need to find?
Type below:
___________

Answer:
We need to find that cost the least per pack

Question 12.
b. Find the price per pack for each treat.
Type below:
___________

Answer:
Pup bites = $5.76/4 ÷ 4/4 = $1.44
Doggie Treats = $7.38/6 ÷ 6/6 = $1.23
Pupster snacks = $7.86/6 ÷ 6/6 = $1.31
Nutri-Biscuits = $9.44/8 ÷ 8/8 = $1.18

Question 12.
c. Complete the sentences
The treat with the highest price per pack is _____.
The treat with the lowest price per pack is _____.
Ryan should buy _____ because _____.
Type below:
___________

Answer:
The treat with the highest price per pack is Pup bites.
The treat with the lowest price per pack is Nutri-Biscuits.
Ryan should buy Nutri-Biscuits because it has the least cost.

Question 13.
Reason Abstractly What information do you need to consider in order to decide whether one product is a better deal than another? When might the lower unit rate not be the best choice? Explain.
Type below:
___________

Answer:
We will consider the low cost in order to decide whether one product is a better deal than another.
The lower unit rate is not the best choice. Because it will show the highest cost.

Question 14.
Select the cars that get a higher mileage per gallon of gas than a car that gets 25 miles per gallon. Mark all that apply.
Options:
a. Car A 22 miles per 1 gallon
b. Car B 56 miles per 2 gallons
c. Car C 81 miles per 3 gallons
d. Car D 51 miles per 3 gallons

Answer:
b. Car B 56 miles per 2 gallons
c. Car C 81 miles per 3 gallons

Explanation:
22/1 = 22
56/2 = 28
81/3 = 27
51/3 = 17

Find Unit Rates – Page No. 247

Write the rate as a fraction. Then find the unit rate.

Question 1.
A wheel rotates through 1,800º in 5 revolutions.
Type below:
___________

Answer:

Explanation:
A wheel rotates through 1,800º in 5 revolutions.
1,800º/5 revolutions
1,800º/5 revolutions ÷ 5/5 = 360º/1revolution

Question 2.
There are 312 cards in 6 decks of playing cards.
Type below:
___________

Answer:
52 cards/1 deck of playing cards

Explanation:
There are 312 cards in 6 decks of playing cards.
312/6 ÷ 6/6 = 52 cards/1 deck of playing cards

Question 3.
Bana ran 18.6 miles of a marathon in 3 hours.
Type below:
___________

Answer:
6.2 miles/hour

Explanation:
Bana ran 18.6 miles of a marathon in 3 hours.
18.6 miles/ 3 hours ÷ 3/3 = 6.2 miles/hour

Question 4.
Cameron paid $30.16 for 8 pounds of almonds.
Type below:
___________

Answer:
$3.77/1 pound

Explanation:
Cameron paid $30.16 for 8 pounds of almonds.
$30.16/8 pounds ÷ 8/8 = $3.77/1 pound

Compare unit rates.

Question 5.
An online game company offers a package that includes 2 games for $11.98. They also offer a package that includes 5 games for $24.95. Which package is a better deal?
_____ package

Answer:
5 game package

Explanation:
An online game company offers a package that includes 2 games for $11.98.
$11.98/2 = $5.99
They also offer a package that includes 5 games for $24.95.
$24.95/5 = $4.99

Question 6.
At a track meet, Samma finished the 200-meter race in 25.98 seconds. Tom finished the 100-meter race in 12.54 seconds. Which runner ran at a faster average rate?
___________

Answer:
Tom

Explanation:
At a track meet, Samma finished the 200-meter race in 25.98 seconds.
200/25.98 seconds = 7.698 – meter/1 sec
Tom finished the 100-meter race in 12.54 seconds.
100 – meter/12.54 seconds = 7.974 – meter/1 sec

Problem Solving

Question 7.
Sylvio’s flight is scheduled to travel 1,792 miles in 3.5 hours. At what average rate will the plane have to travel to complete the trip on time?
Type below:
___________

Answer:
512 miles per hour

Explanation:
Sylvio’s flight is scheduled to travel 1,792 miles in 3.5 hours.
1,792 miles/3.5 hours ÷ 3.5/3.5 = 512 miles per hour

Question 8.
Rachel bought 2 pounds of apples and 3 pounds of peaches for a total of $10.45. The apples and peaches cost the same amount per pound. What was the unit rate?
Type below:
___________

Answer:
$2.09 per pound

Explanation:
Rachel bought 2 pounds of apples and 3 pounds of peaches for a total of $10.45.
The apples and peaches cost the same amount per pound.
2 + 3 = 5
$10.45/5 = $2.09 per pound

Question 9.
Write a word problem that involves comparing unit rates.
Type below:
___________

Answer:
At a track meet, Samma finished the 200-meter race in 25.98 seconds. Tom finished the 100-meter race in 12.54 seconds. Which runner ran at a faster average rate?
At a track meet, Samma finished the 200-meter race in 25.98 seconds.
200/25.98 seconds = 7.698 – meter/1 sec
Tom finished the 100-meter race in 12.54 seconds.
100 – meter/12.54 seconds = 7.974 – meter/1 sec
Tom

Lesson Check – Page No. 248

Question 1.
Cran–Soy trail mix costs $2.99 for 5 ounces, Raisin–Nuts mix costs $3.41 for 7 ounces, Lots of Cashews mix costs $7.04 for 8 ounces, and Nuts for You mix costs $2.40 for 6 ounces. List the trail mix brands in order from the least expensive to the most expensive.
Type below:
___________

Answer:
Nuts for You, Raisin–Nuts, Cran–Soy trail mix, Lots of Cashews mix

Explanation:
Cran–Soy trail mix costs $2.99 for 5 ounces,
$2.99/5 = $0.598
Raisin–Nuts mix costs $3.41 for 7 ounces,
$3.41/7 = $0.487
Lots of Cashews mix costs $7.04 for 8 ounces,
$7.04/8 = $0.88
and Nuts for You mix costs $2.40 for 6 ounces.
$2.40/6 = $0.4

Question 2.
Aaron’s heart beats 166 times in 120 seconds. Callie’s heart beats 88 times in 60 seconds. Emma’s heart beats 48 times in 30 seconds. Galen’s heart beats 22 times in 15 seconds. Which two students’ heart rates are equivalent?
Type below:
___________

Answer:
Callie and Galen

Explanation:
Aaron’s heart beats 166 times in 120 seconds.
166/120 = 1.3833
Callie’s heart beats 88 times in 60 seconds.
88/60 = 1.4666
Emma’s heart beats 48 times in 30 seconds.
48/30 = 1.6
Galen’s heart beats 22 times in 15 seconds.
22/15 = 1.4666

Spiral Review

Question 3.
Courtlynn combines \(\frac{7}{8}\) cup sour cream with \(\frac{1}{2}\) cup cream cheese. She then divides the mixture between 2 bowls. How much mixture does Courtlynn put in each bowl?
\(\frac{□}{□}\) cup

Answer:
\(\frac{11}{16}\) cup

Explanation:
Courtlynn combines \(\frac{7}{8}\) cup sour cream with \(\frac{1}{2}\) cup cream cheese.
7/8 + 1/2 = 11/8
11/8 ÷ 2 = 11/8 × 1/2 = 11/16 cup

Question 4.
Write a comparison using < or > to show the relationship between |-\(\frac{2}{3}\)| and – \(\frac{5}{6}\).
Type below:
___________

Answer:
>

Explanation:
|-\(\frac{2}{3}\)| = 2/3 = 0.666
– \(\frac{5}{6}\) = -0.8333
|-\(\frac{2}{3}\)| > – \(\frac{5}{6}\)

Question 5.
There are 18 tires on one truck. How many tires are on 3 trucks of the same type?
_____ tires

Answer:
54 tires

Explanation:
There are 18 tires on one truck.
For 3 trucks, (3 × 18)/1 = 54 tires

Question 6.
Write two ratios that are equivalent to \(\frac{5}{6}\).
Type below:
___________

Answer:
\(\frac{5}{6}\) = \(\frac{10}{12}\), \(\frac{15}{18}\)

Explanation:
5/6 × 2/2 = 10/12
5/6 × 3/3 = 15/18

Share and Show – Page No. 251

Use a unit rate to find the unknown value.

Question 1.
\(\frac{10}{?}=\frac{6}{3}\)
_____

Answer:
5

Explanation:
6/3 ÷ 3/3 = 2/1
2/1 × 5/5 = 10/1
The unknown value is 5

Question 2.
\(\frac{6}{8}=\frac{?}{20}\)
_____

Answer:
15

Explanation:
6/8 ÷ 8/8 = 0.75/1
0.75/1 × 20/20 = 15/20
The unknown value is 15

On Your Own

Use a unit rate to find the unknown value.

Question 3.
\(\frac{40}{8}=\frac{45}{?}\)
_____

Answer:
9

Explanation:
40/8 ÷ 8/8 = 5/1
5/1 × 9/9 = 45/9
The unknown value is 9

Question 4.
\(\frac{42}{14}=\frac{?}{5}\)
_____

Answer:
15

Explanation:
42/14 ÷ 14/14 = 3/1
3/1 × 5/5 = 15/5
The unknown value is 15

Question 5.
\(\frac{?}{2}=\frac{56}{8}\)
_____

Answer:
14

Explanation:
56/8 ÷ 8/8 = 7/1
7/1 × 2/2 = 14/2
The unknown value is 14

Question 6.
\(\frac{?}{4}=\frac{26}{13}\)
_____

Answer:
8

Explanation:
26/13 ÷ 13/13 = 2/1
2/1 × 4/4 = 8/4
The unknown value is 8

Practice: Copy and Solve Draw a bar model to find the unknown value.

Question 7.
\(\frac{4}{32}=\frac{9}{?}\)
_____

Answer:
Grade 6 Chapter 4 image 29

Explanation:
4/32 ÷ 32/32 = 0.125/1
0.125/1 × 72/72 = 9/72
The unknown value is 72

Question 8.
\(\frac{9}{3}=\frac{?}{4}\)
_____

Answer:
Grade 6 Chapter 4 image 30
12

Explanation:
9/3 ÷ 3/3 = 3/1
3/1 × 4/4 = 12/4
The unknown value is 12

Question 9.
\(\frac{?}{14}=\frac{9}{7}\)
_____

Answer:
Grade 6 Chapter 4 image 31

Explanation:
9/7 ÷ 7/7 = 1.2857/1
1.2857/1 × 14/14 = 18/14
The unknown value is 18

Question 10.
\(\frac{3}{?}=\frac{2}{1.25}\)
_____

Answer:
1.875

Explanation:
2/1.25 ÷ 1.25/1.25 = 1.6/1
1.6/1 × 1.875/1.875 = 3/1.875
The unknown value is 1.875

Question 11.
Communicate Explain how to find an unknown value in a ratio by using a unit rate.
Type below:
___________

Answer:
Firstly, Identify the known ratio, where both values are known. Then, Identify the ratio with one known value and one unknown value. Next, Use the two ratios to create a proportion. Finally, Cross-multiply to solve the problem.

Question 12.
Savannah is tiling her kitchen floor. She bought 8 cases of tile for $192. She realizes she bought too much tile and returns 2 unopened cases to the store. What was her final cost for tile?
$ _____

Answer:
$144

Explanation:
Savannah is tiling her kitchen floor. She bought 8 cases of tile for $192.
$192/8 ÷ 8/8 = $24 per each case of tile
She realizes she bought too much tile and returns 2 unopened cases to the store.
So, she bought 8 – 2 = 6 cases of tiles.
6 × $24 = $144

Problem Solving + Applications – Page No. 251

Pose a Problem

Question 13.
Josie runs a T-shirt printing company. The table shows the length and width of four sizes of T-shirts. The measurements of each size T-shirt form equivalent ratios.
Go Math Grade 6 Answer Key Chapter 4 Model Ratios 36
What is the length of an extra-large T-shirt?
Write two equivalent ratios and find the unknown value:
Go Math Grade 6 Answer Key Chapter 4 Model Ratios 37
The length of an extra-large T-shirt is 36 inches.
Write a problem that can be solved by using the information in the table and could be solved by using equivalent ratios
Type below:
___________

Answer:
Small = 27/18 ÷ 18/18 = 1.5
Medium = 30/20 = 3/2 = 1.5
Large = 1.5/1 × 22/22 = 33/22
the length of an extra-large T-shirt = 1.5/1 × 24/24 = 36/24
What is the length of an large T-shirt?
Write two equivalent ratios and find the unknown value?
Large = 1.5/1 × 22/22 = 33/22
33/22 × 2/2 = 66/44
33/22 × 3/3 = 99/66

Question 14.
Peri earned $27 for walking her neighbor’s dog 3 times. If Peri earned $36, how many times did she walk her neighbor’s dog? Use a unit rate to find the unknown value.
_____ times

Answer:
4 times

Explanation:
Peri earned $27 for walking her neighbor’s dog 3 times.
If Peri earned $36, ($36 × 3)/$27 = 4

Use Unit Rates – Page No. 253

Use a unit rate to find the unknown value.

Question 1.
\(\frac{34}{7}=\frac{?}{7}\)
_____

Answer:
34

Explanation:
34/7 ÷ 7/7 = 4.8571/1
4.8571/1 × 7/7 = 34
The unknown value is 34

Question 2.
\(\frac{16}{32}=\frac{?}{14}\)
_____

Answer:
7

Explanation:
16/32 ÷ 32/32 = 0.5/1
0.5/1 × 14/14 = 7/1
The unknown value is 7

Question 3.
\(\frac{18}{?}=\frac{21}{7}\)
_____

Answer:
6

Explanation:
21/7 ÷ 7/7 = 3/1
3/1 × 6/6 = 18/6
The unknown value is 6

Question 4.
\(\frac{?}{16}=\frac{3}{12}\)
_____

Answer:
4

Explanation:
3/12 ÷ 12/12 = 0.25/1
0.25/1 × 16/16 = 4
The unknown value is 4

Draw a bar model to find the unknown value.

Question 5.
\(\frac{15}{45}=\frac{6}{?}\)
_____

Answer:
Grade 6 Chapter 4 image 32
18

Explanation:
15/45 ÷ 45/45 = 1/3
1/3 × 6/6 = 6/18
The unknown value is 18

Question 6.
\(\frac{3}{6}=\frac{?}{7}\)
_____

Answer:
Grade 6 Chapter 4 image 33
3.5

Explanation:
3/6 ÷ 6/6 = 1/2
1/2 × 3.5/3.5 = 3.5/7
The unknown value is 3.5

Problem Solving

Question 7.
To stay properly hydrated, a person should drink 32 fluid ounces of water for every 60 minutes of exercise. How much water should Damon drink if he rides his bike for 135 minutes?
_____ fluid ounces

Answer:
72 fluid ounces

Explanation:
To stay properly hydrated, a person should drink 32 fluid ounces of water for every 60 minutes of exercise.
If he rides his bike for 135 minutes, (135 × 32)/60 = 72

Question 8.
Lillianne made 6 out of every 10 baskets she attempted during basketball practice. If she attempted to make 25 baskets, how many did she make?
_____ baskets

Answer:
15 baskets

Explanation:
Lillianne made 6 out of every 10 baskets she attempted during basketball practice. If she attempted to make 25 baskets,
(25 × 6)/10 = 15 baskets

Question 9.
Give some examples of real-life situations in which you could use unit rates to solve an equivalent ratio problem.
Type below:
___________

Answer:
1) If a 10-ounce box of cereal costs $3 and a 20-ounce box of cereal costs $5, the 20 ounce box is the better value because each ounce of cereal is cheaper.
2) Yoda Soda is the intergalactic party drink that will have all your friends saying, “Mmmmmm, good this is!”
You are throwing a party, and you need 555 liters of Yoda Soda for every 121212 guests.
If you have 363636 guests, how many liters of Yoda Soda do you need?

Lesson Check – Page No. 254

Question 1.
Randi’s school requires that there are 2 adult chaperones for every 18 students when the students go on a field trip to the museum. If there are 99 students going to the museum, how many adult chaperones are needed?
_____ chaperones

Answer:
11 chaperones

Explanation:
Randi’s school requires that there are 2 adult chaperones for every 18 students when the students go on a field trip to the museum.
If there are 99 students going to the museum, (99 × 2)/18 = 11 chaperones

Question 2.
Landry’s neighbor pledged $5.00 for every 2 miles he swims in a charity swim-a-thon. If Landry swims 3 miles, how much money will his neighbor donate?
$ _____

Answer:
$7.5

Explanation:
Landry’s neighbor pledged $5.00 for every 2 miles he swims in a charity swim-a-thon. If Landry swims 3 miles, 15/2 = $7.5

Spiral Review

Question 3.
Describe a situation that could be represented by –8.
Type below:
___________

Answer:
In Alaska the normal temperature in December was 3 degrees. Scientist predicted that by February the temperature would drop 11 degrees. What is the predicted temperature for February? The answer is -8.

Question 4.
What are the coordinates of point G?
Go Math Grade 6 Answer Key Chapter 4 Model Ratios 38
Type below:
___________

Answer:
(-2, 0.5)

Explanation:
The x-coordinate is -2
The y-coordinate is 0.5

Question 5.
Gina bought 6 containers of yogurt for $4. How many containers of yogurt could Gina buy for $12?
_____ containers

Answer:
18 containers

Explanation:
Gina bought 6 containers of yogurt for $4.
For $12, ($12 × 6)/$4 = 18

Question 6.
A bottle containing 64 fluid ounces of juice costs $3.84. What is the unit rate?
$ _____

Answer:
$0.06

Explanation:
A bottle containing 64 fluid ounces of juice costs $3.84.
$3.84/64 = $0.06

Share and Show – Page No. 257

A redwood tree grew at a rate of 4 feet per year. Use this information for 1–3.

Question 1.
Complete the table of equivalent ratios for the first 5 years.
Go Math Grade 6 Answer Key Chapter 4 Model Ratios 39
Type below:
___________

Answer:
Grade 6 Chapter 4 image 34

Explanation:
A redwood tree grew at a rate of 4 feet per year.
For 2 years, 2 × 4 = 8ft
For 3 years, 3 × 4 = 12ft
For 4 years, 4 × 4 = 16ft
For 5 years, 5 × 4 = 20ft

Question 2.
Write ordered pairs, letting the x-coordinate represent time in years and the y-coordinate represent height in feet.
Type below:
___________

Answer:
(1, 4), (2, 8), (3, 12), (4, 16), (5, 20)

Question 3.
Use the ordered pairs to graph the tree’s growth over time.
Type below:
___________

Answer:
Grade 6 Chapter 4 image 35

On Your Own

The graph shows the rate at which Luis’s car uses gas, in miles per gallon. Use the graph for 4–8.
Go Math Grade 6 Answer Key Chapter 4 Model Ratios 40

Question 4.
Complete the table of equivalent ratios.
Type below:
___________

Answer:
30/1, 60/2, 90/3, 120/4, 150/5

Question 5.
Find the car’s unit rate of gas usage.
Type below:
___________

Answer:
30mi/gal

Question 6.
How far can the car go on 5 gallons of gas?
_____ miles

Answer:
150 miles

Explanation:
the car go on 5 gallons of gas, 150/5

Question 7.
Estimate the amount of gas needed to travel 50 miles.
Type below:
___________

Answer:
5/3

Explanation:
30/1,
50/30 = 5/3

Question 8.
Ellen’s car averages 35 miles per gallon of gas. If you used equivalent ratios to graph her car’s gas usage, how would the graph differ from the graph of Luis’s car’s gas usage?
Type below:
___________

Answer:
Grade 6 Chapter 4 image 36
The distance is high for Ellen’s car’s gas usage compared to Luis’s car’s gas usage per one gal

Explanation:
35/1 × 2/2 = 70/2
35/1 × 3/3 = 105/3
35/1 × 4/4 = 140/4
35/1 × 5/5 = 175/5

Problem Solving + Applications – Page No. 258

Question 9.
Look for Structure The graph shows the depth of a submarine over time. Use equivalent ratios to find the number of minutes it will take the submarine to descend 1,600 feet.
Go Math Grade 6 Answer Key Chapter 4 Model Ratios 41
_____ minutes

Answer:
8 minutes

Explanation:
200/1 × 8/8 = 1600/8

Question 10.
The graph shows the distance that a plane flying at a steady rate travels over time. Use equivalent ratios to find how far the plane travels in 13 minutes.
Go Math Grade 6 Answer Key Chapter 4 Model Ratios 42
_____ miles

Answer:
91 miles

Explanation:
7/1 × 13/13 = 91/13

Question 11.
Sense or Nonsense? Emilio types at a rate of 84 words per minute. He claims that he can type a 500-word essay in 5 minutes. Is Emilio’s claim sense or nonsense? Use a graph to help explain your answer.
Type below:
___________

Answer:
He said that he can write 84 in 60sec ,500 words will be written in 500×60/84=357 it’s a nonsense

Question 12.
The Tuckers drive at a rate of 20 miles per hour through the mountains. Use the ordered pairs to graph the distance traveled over time.
Go Math Grade 6 Answer Key Chapter 4 Model Ratios 43
Type below:
___________

Answer:
Grade 6 Chapter 4 image 37

Equivalent Ratios and Graphs – Page No. 259

Christie makes bracelets. She uses 8 charms for each bracelet. Use this information for 1–3.

Question 1.
Complete the table of equivalent ratios for the first 5 bracelets.
Type below:
___________

Answer:
Grade 6 Chapter 4 image 38

Explanation:

Question 2.
Write ordered pairs, letting the x-coordinate represent the number of bracelets and the y-coordinate represent the number of charms.
Type below:
___________

Answer:
(1, 8), (2, 16), (3, 24), (4, 32), (5, 40)

Question 3.
Use the ordered pairs to graph the charms and bracelets.
Type below:
___________

Answer:
Grade 6 Chapter 4 image 39

The graph shows the number of granola bars that are in various numbers of boxes of Crunch N Go. Use the graph for 4–5.
Go Math Grade 6 Answer Key Chapter 4 Model Ratios 44

Question 4.
Complete the table of equivalent ratios.
Go Math Grade 6 Answer Key Chapter 4 Model Ratios 45
Type below:
___________

Answer:
Grade 6 Chapter 4 image 40

Question 5.
Find the unit rate of granola bars per box.
Type below:
___________

Answer:
10 bars/1 box

Problem Solving

Question 6.
Look at the graph for Christie’s Bracelets. How many charms are needed for 7 bracelets?
_____ charms

Answer:
56 charms

Question 7.
Look at the graph for Crunch N Go Granola Bars. Stefan needs to buy 90 granola bars. How many boxes must he buy?
_____ boxes

Answer:
9 boxes

Question 8.
Choose a real-life example of a unit rate. Draw a graph of the unit rate. Then explain how another person could use the graph to find the unit rate.
Type below:
___________

Answer:
Sam prepares 4 bracelets per month. How many bracelets does she prepare in a span of 6 months?
For 1 month, 1 × 4 = 4 bracelets
For 2 months, 2 × 4 = 8 bracelets
For 3 months, 3 × 4 = 12 bracelets
For 4 months, 4 × 4 = 16 bracelets
For 5 months, 5 × 4 = 20 bracelets

Lesson Check – Page No. 260

Question 1.
A graph shows the distance a car traveled over time. The x-axis represents time in hours, and the y-axis represents distance in miles. The graph contains the point (3, 165). What does this point represent?
Type below:
___________

Answer:

Explanation:
In 3 hours the car traveled 165 miles.
(3,165) is (x,y) so 3 = x and 165 = y, and
3=x=time in hours
165=y= miles…. soooo
In 3 hours the car traveled 165 miles

Question 2.
Maura charges $11 per hour to babysit. She makes a graph comparing the amount she charges (the y-coordinate) to the time she babysits (the x-coordinate). Which ordered pair shown is NOT on the graph?
(4, 44) (11, 1) (1, 11) (11, 12)
Type below:
___________

Answer:
(11, 1)

Explanation:
It is not 11,1 because she charges 11 hours per hour (y coordinate) and x would be time to babysit. so it can’t be 11,1

Spiral Review

Question 3.
List 0, –4, and 3 from least to greatest.
Type below:
___________

Answer:
-4, 0, 3

Question 4.
What two numbers can be used in place of the ? to make the statement true?
|?| = \(\frac{8}{9}\)
Type below:
___________

Answer:
–\(\frac{8}{9}\), \(\frac{8}{9}\)

Explanation:
|-\(\frac{8}{9}\)| = \(\frac{8}{9}\)
|\(\frac{8}{9}\)| = \(\frac{8}{9}\)

Question 5.
Morgan plots the point (4, –7) on a coordinate plane. If she reflects the point across the y-axis, what are the coordinates of the reflected point?
Type below:
___________

Answer:
(-4, -7)

Explanation:
Morgan plots the point (4, –7) on a coordinate plane. If she reflects the point across the y-axis, it will be (-4, -7)

Question 6.
Jonathan drove 220 miles in 4 hours. Assuming he drives at the same rate, how far will he travel in 7 hours?
_____ miles

Answer:
385 miles

Explanation:
Jonathan drove 220 miles in 4 hours.
If he travel in 7 hours, (7 × 220)/4 = 385 miles

Chapter 4 Review/Test – Page No. 261

Question 1.
Kendra has 4 necklaces, 7 bracelets, and 5 rings. Draw a model to show the ratio that compares rings to bracelets
Type below:___________

Answer:
Grade 6 Chapter 4 image 41

Question 2.
There are 3 girls and 2 boys taking swimming lessons. Write the ratio that compares the girls taking swimming lessons to the total number of students taking swimming lessons.
Type below:
___________

Answer:
3 : 5

Explanation:
There are 3 girls and 2 boys taking swimming lessons.
the total number of students taking swimming lessons = 5
3 : 5

Question 3.
Luis adds 3 strawberries for every 2 blueberries in his fruit smoothie. Draw a model to show the ratio that compares strawberries to blueberries.
Type below:
___________

Answer:
Grade 6 Chapter 4 image 42

Question 4.
Write the ratio 3 to 10 in two different ways.
Type below:
___________

Answer:
3/10, 3 : 10

Question 5.
Alex takes 3 steps every 5 feet he walks. As Alex continues walking, he takes more steps and walks a longer distance. Complete the table by writing two equivalent ratios.
Go Math Grade 6 Answer Key Chapter 4 Model Ratios 46
Type below:
___________

Answer:
Grade 6 Chapter 4 image 43

Explanation:
Alex takes 3 steps every 5 feet he walks. As Alex continues walking, he takes more steps and walks a longer distance.
3/5 × 2/2 = 6/10
3/5 × 3/3 = 9/15

Page No. 262

Question 6.
Sam has 3 green apples and 4 red apples. Select the ratios that compare the number of red apples to the total number of apples. Mark all that apply.
Options:
a. 4 to 7
b. 3 to 7
c. 4 : 7
d. 4 : 3
e. \(\frac{3}{7}\)
f. \(\frac{4}{7}\)

Answer:
a. 4 to 7
c. 4 : 7
f. \(\frac{4}{7}\)

Explanation:
Sam has 3 green apples and 4 red apples.
the total number of apples = 3 + 4 = 7
4 : 7

Question 7.
Jeff ran 2 miles in 12 minutes. Ju Chan ran 3 miles in 18 minutes. Did Jeff and Ju Chan run the same number of miles per minute? Complete the tables of equivalent ratios to support your answer.
Go Math Grade 6 Answer Key Chapter 4 Model Ratios 47
Type below:
___________

Answer:
Grade 6 Chapter 4 image 44

Explanation:
2/12 × 2/2 = 4/24
2/12 × 3/3 = 6/39
2/12 × 4/4 = 8/48
3/18 × 2/2 = 6/36
3/18 × 3/3 = 9/24
3/18 × 4/4 = 12/72

Question 8.
Jen bought 2 notebooks for $10. Write the rate as a fraction. Then find the unit rate.
Type below:
___________

Answer:
$10/2
unit rate = $5

Explanation:
Jen bought 2 notebooks for $10.
$10/2 ÷ 2/2 = $5

Page No. 263

Question 9.
Determine whether each ratio is equivalent to \(\frac{1}{2}, \frac{2}{3}, \text { or } \frac{4}{7}\). Write the ratio in the correct box.
Go Math Grade 6 Answer Key Chapter 4 Model Ratios 48
Type below:
___________

Answer:
Grade 6 Chapter 4 image 45

Explanation:
1/2 × 2/2 = 4/8
7/14 ÷ 2/2 = 1/2
20/35 ÷ 5/5 = 4/7
40/80 ÷ 40/40 = 1/2
8/14 ÷ 2/2 = 4/7
4/6 ÷ 2/2 = 2/3
8/12 ÷ 4/4 = 2/3

Question 10.
Amos bought 5 cantaloupes for $8. How many cantaloupes can he buy for $24? Show your work.
_____ cantaloupes

Answer:
15 cantaloupes

Explanation:
Amos bought 5 cantaloupes for $8.
For $24, ($24 × 5)/$8 = 15

Question 11.
Camille said \(\frac{4}{5}\) is equivalent to \(\frac{24}{30}\). Check her work by making a table of equivalent ratios.
Type below:
___________

Answer:
Grade 6 Chapter 4 image 46

Question 12.
A box of oat cereal costs $3.90 for 15 ounces. A box of rice cereal costs $3.30 for 11 ounces. Which box of cereal costs less per ounce? Use numbers and words to explain your answer.
Type below:
___________

Answer:
A box of oat cereal costs $3.90 for 15 ounces.
$3.90/15 = $0.26
A box of rice cereal costs $3.30 for 11 ounces.
$3.30/11 = $0.3
$0.26 < $0.3

Page No. 264

Question 13.
Scotty earns $35 for babysitting for 5 hours. If Scotty charges the same rate, how many hours will it take him to earn $42?
_____ hours

Answer:
6 hours

Explanation:
Scotty earns $35 for babysitting for 5 hours
For $42, (42 × 5)/35 = 6

Question 14.
Use a unit rate to find the unknown value.
Go Math Grade 6 Answer Key Chapter 4 Model Ratios 49
Type below:
___________

Answer:
Grade 6 Chapter 4 image 47

Explanation:
(9 × 42)/14 = 3

Question 15.
Jenna saves $3 for every $13 she earns. Vanessa saves $6 for every $16 she earns. Is Jenna’s ratio of money saved to money earned equivalent to Vanessa’s ratio of money saved to money earned?
Type below:
___________

Answer:
No, 3/13 = 6/26. Vanessa ratio is 6/16

Question 16.
The Hendersons are on their way to a national park. They are traveling at a rate of 40 miles per hour. Use the ordered pairs to graph the distance traveled over time
Go Math Grade 6 Answer Key Chapter 4 Model Ratios 50
Type below:
___________

Answer:
Grade 6 Chapter 4 image 48

Page No. 265

Question 17.
Abby goes to the pool to swim laps. The graph shows how far Abby swam over time. Use equivalent ratios to find how far Abby swam in 7 minutes
Go Math Grade 6 Answer Key Chapter 4 Model Ratios 51
_____ meters

Answer:
350 meters

Explanation:
50/1 × 7/7 = 350/7

Question 18.
Caleb bought 6 packs of pencils for $12.
Part A
How much will he pay for 9 packs of pencils? Use numbers and words to explain your answer
$ _____

Answer:
$18

Explanation:
Caleb bought 6 packs of pencils for $12.
6/12 = 1/2 × 9/9 = 9/18
So, $18 is the answer

Question 18.
Part B
Describe how to use a bar model to solve the problem.
Type below:
___________

Answer:
Take the known ratio and identify the unknown value using known ratio.

Page No. 266

Question 19.
A rabbit runs 35 miles per hour. Select the animals who run at a faster unit rate per hour than the rabbit. Mark all that apply.
Options:
a. Reindeer: 100 miles in 2 hours
b. Ostrich: 80 miles in 2 hours
c. Zebra: 90 miles in 3 hours
d. Squirrel: 36 miles in 3 hours

Answer:
a. Reindeer: 100 miles in 2 hours
b. Ostrich: 80 miles in 2 hours

Explanation:
A rabbit runs 35 miles per hour.
35/1
100/2 = 50/1
80/2 = 40/1
90/3 = 30/1
36/3 = 12/1

Question 20.
Water is filling a bathtub at a rate of 3 gallons per minute.
Part A
Complete the table of equivalent ratios for the first five minutes of the bathtub filling up.
Go Math Grade 6 Answer Key Chapter 4 Model Ratios 52
Type below:
___________

Answer:
Grade 6 Chapter 4 image 50

Question 20.
Part B
Emily said there will be 36 gallons of water in the bathtub after 12 minutes. Explain how Emily could have found her answer
Type below:
___________

Answer:
Emily said there will be 36 gallons of water in the bathtub after 12 minutes.
36/12 ÷ 12/12 = 3/1
She can find the answer using the unit rate.

Conclusion:

Hope the solutions provided in Go Math Grade 6 Answer Key Chapter 4 Model Ratios helped you to enhance your math skills. Also, this Go Math Answer Key helps you to score the highest marks in the exam. Share this pdf link with your friends so that they can overcome the difficulties in maths. All the Best!!!

Go Math Grade 8 Answer Key Chapter 7 Solving Linear Equations

go-math-grade-8-chapter-7-solving-linear-equations-answer-key

Are you searching for Go Math Grade 8 Answer Key Chapter 7 Solving Linear Equations? If your answer is yes, then you are on the right page. Students of 8th standard can get the Download link of HMH Go Math Grade 8 Solution Key Chapter 7 Solving Linear Equations for free. Get the best maths tactics and ways of solving the problems for all the questions with the help of the Go Math Grade 8 Chapter 7 Solving Linear Equations Answer Key. You can practice all the questions to have a perfect grip on the Grade 8 maths subject. Go Math Grade 8 Answer Key is the best resource to improve math skills.

Go Math Grade 8 Chapter 7 Solving Linear Equations Answer Key

Improve your performance in exams with the help of Go Math Grade 8 Answer Key Chapter 7 Solving Linear Equations. You can find the review questions at the end of the chapter to test your knowledge. The topics covered in this Solving Linear Equations chapter are equations with the variable on both sides, equations with rational numbers, equations with the distributive property, equations with many solutions or no solution, etc. Just go through the online pdf and start practicing now. By looking into the questions and answers available on Go Math Grade 8 Answer Key Chapter 7 Solving Linear Equations.

Lesson 1: Equations with the Variable on Both Sides

Lesson 2: Equations with Rational Numbers

Lesson 3: Equations with the Distributive Property

Lesson 4: Equations with Many Solutions or No Solution

Lesson 5: Equations with the Variable on Both Sides

Reviews

Guided Practice – Equations with the Variable on Both Sides – Page No. 200

Use algebra tiles to model and solve each equation.

Question 1.
x + 4 = -x – 4
x = ________

Answer:
x = -4

Explanation:
Model x + 4 on the left side of the mat and -x -4 on the right side.
grade 8 chapter 7 image 1
Add one c-tile to both sides. This represents adding x to both sides of the equation. Remove zero pairs.
grade 8 chapter 7 image 2
Place four -1-tiles on both sides. This represents subtracting -4 from both sides of the equation. Remove zero pairs.
grade 8 chapter 7 image 3
Separate each side into 2 equal groups. One x-tile is equivalent to four -1-tiles.
grade 8 chapter 7 image 4
x = -4

Question 2.
2 – 3x = -x – 8
x = ________

Answer:

Explanation:
Given 2 – 3x = -x – 8
Model 2-3x on the left side of the mat and -x-8 on the right side.
grade 8 chapter 7 image 5
Place one x tile to both sides. This represents subtracting from both sides of the equation.
grade 8 chapter 7 image 6
Remove 2 1 tiles from sides. This represents subtracting from both sides of the equation.
grade 8 chapter 7 image 7
Separate each side into 2 equal groups. One -x tile is equivalent to 5 – 1 tile.
grade 8 chapter 7 image 8
The solution is -x = -5 or x = 5

Question 3.
At Silver Gym, membership is $25 per month, and personal training sessions are $30 each. At Fit Factor, membership is $65 per month, and personal training sessions are $20 each. In one month, how many personal training sessions would Sarah have to buy to make the total cost at the two gyms equal?
________ sessions

Answer:
4 sessions

Explanation:
At Silver Gym, membership is $25 per month, and personal training sessions are $30 each.
Membership + Personal training session = 25 + 30x
At Fit Factor, membership is $65 per month, and personal training sessions are $20 each.
Membership + Personal training session = 65 + 20x
Membership at Silver Gym = Membership at Fit Factor
25 + 30x = 65 + 20x
30x – 20x = 65 – 25
10x = 40
x = 4
Sarah would have to buy 4 sessions for the total cost at the two gyms to be equal.

Question 4.
Write a real-world situation that could be modeled by the equation 120 + 25x = 45x.
Type below:
_______________

Answer:
120 + 25x = 45x
Sarah offers a plan to tutor a student at $25 per her plus a one-time registration fee of $ 120.
Surah offers an alternative plan to tutor a student at $45 per hour and no registration fee.
120 + 25x = 45x

Question 5.
Write a real-world situation that could be modeled by the equation 100 – 6x = 160 – 10x.
Type below:
_______________

Answer:
100 – 6x = 160 – 10x
The initial water in Tank A is 100 gallons and leaks at 6 gallons per week.
The initial water in Tank B is 160 gallon and leaks at 10 gallons per week
100 – 6x = 160 – 10x

Essential Question Check-In

Question 6.
How can you solve an equation with the variable on both sides?
Type below:
_______________

Answer:
Isolate the variable on one side. Add/subtract the variable with a lower coefficient from both sides. Add/subtract the constant (with the variable) from both sides. Divide both sides by coefficient of isolated variable.

Independent Practice – Equations with the Variable on Both Sides – Page No. 201

Question 7.
Derrick’s Dog Sitting and Darlene’s Dog Sitting are competing for new business. The companies ran the ads shown.
Go Math Grade 8 Answer Key Chapter 7 Solving Linear Equations Lesson 1: Equations with the Variable on Both Sides img 1
a. Write and solve an equation to find the number of hours for which the total cost will be the same for the two services.
________ hours

Answer:
3 hours

Explanation:
Hourly rate + One time fee = 5x + 12
Hourly rate + One time fee = 3x + 18
5x + 12 = 3x + 18
5x – 3x = 18 – 12
2x = 6
x = 3
the cost of the two dog sitting would be same for 3 hrs.

Question 7.
b. Analyze Relationships
Which dog sitting service is more economical to use if you need 5 hours of service? Explain.
____________

Answer:
Darlene’s Dog Sitting would be cheaper

Explanation:
Let y be the cost of dog sitting after x hours for both companies
y = 5x +12
y = 3x +18
Substitute x = 5
y = 5(5) + 12 = 37
y = 3 (5) + 18 = 33
compare the cost for both companies for x = 5hr.
$37 > $33
Darlene’s Dog Sitting would be cheaper

Question 8.
Country Carpets charges $22 per square yard for carpeting, and an additional installation fee of $100. City Carpets charges $25 per square yard for the same carpeting, and an additional installation fee of $70.
a. Write and solve an equation to find the number of square yards of carpeting for which the total cost charged by the two companies will be the same.
_______ square yards

Answer:
10 square yards

Explanation:
Unit square rate + One time installation fee = 22x + 100
Unit square rate + One time installation fee = 25x + 70
22x + 100 = 25x + 70
25x – 22x = 100 – 70
3x = 30
x = 10
the total cost charged by the two companies will be the same for 10 square yards of carpeting.

Question 8.
b. Justify Reasoning
Mr. Shu wants to hire one of the two carpet companies to install carpeting in his basement. Is he more likely to hire Country Carpets or City Carpets? Explain your reasoning.
___________

Answer:
City Carpets are cheaper when x < 10
y = 25(9) + 70 = 295
y = 22(9) + 100 = 298
Country Carpets are cheaper when x > 10
y = 25(11) + 70 = 345
y = 25(11) + 100 = 342
If Mr.Shu needs the carpenting done for less than 10square yards, he will hire City Carpets and if he needs carpenting for more than 10 square yard, he will hire Country Carpets.

Write an equation to represent each relationship. Then solve the equation.

Question 9.
Two less than 3 times a number is the same as the number plus 10.
________

Answer:
3x – 2 = x + 10
x = 6

Explanation:
Two less than 3 times a number is the same as the number plus 10.
Two less than 3 times x is the same as the x plus 10.
Two less than 3x is the same as the x + 10
3x – 2 is the same as x + 10
3x – 2 = x + 10
3x – x = 10 + 2
2x = 12
x = 6

Question 10.
A number increased by 4 is the same as 19 minus 2 times the number.
______

Answer:
x + 4 = 19 – 2x
x = 5

Explanation:
A number increased by 4 is the same as 19 minus 2 times the number.
x increased by 4 is the same as 19 minus 2x.
x + 4 is the same as 19 – 2x
x + 4 = 19 – 2x
x + 2x = 19 – 4
3x = 15
x = 15/3
x = 5

Question 11.
Twenty less than 8 times a number is the same as 15 more than the number.
Type below:
____________

Answer:
8x – 20 = x + 15
x = 5

Explanation:
Twenty less than 8 times a number is the same as 15 more than the number.
Twenty less than 8 times x is the same as 15 more than the x.
Twenty less than 8x is the same as 15 more than the x
8x – 20 is the same as x + 15
8x – 20 = x + 15
8x – x = 15 + 20
7x = 35
x = 35/7 = 5
x = 5

Equations with the Variable on Both Sides – Page No. 202

Question 12.
The charges for an international call made using the calling card for two phone companies are shown in the table.
Go Math Grade 8 Answer Key Chapter 7 Solving Linear Equations Lesson 1: Equations with the Variable on Both Sides img 2
a. What is the length of a phone call that would cost the same no matter which company is used?
_______ minutes

Answer:
10 minutes

Explanation:
Cost of minutes + One time fee = 3x + 35
Cost of minutes + One time fee = 2x + 45
3x + 35 = 2x + 45
3x – 2x = 45 – 35
x = 10
The cost would be same for 10 minutes.

Question 12.
b. Analyze Relationships
When is it better to use the card from Company B?
Type below:
____________

Answer:
y = 3x + 35
y = 3(11) + 35 = $68
y = 2x + 45
y = 2(11) + 45 = $67
Since when x > 10, Company B is cheaper so it should be better to use when the length of the call is greater than 10 minutes.

H.O.T.

Focus on Higher Order Thinking

Question 13.
Draw Conclusions
Liam is setting up folding chairs for a meeting. If he arranges the chairs in 9 rows of the same length, he has 3 chairs left over. If he arranges the chairs in 7 rows of that same length, he has 19 left over. How many chairs does Liam have?
______ chairs

Answer:
75 chairs

Explanation:
Number of chairs in 9 row + left over chairs = 9x + 3
Number of chairs in 7 row + left over chairs = 7x + 19
9x + 3 = 7x + 19
9x – 7x = 19 – 3
2x = 16
x = 16/2
x = 8
Total number of chairs = 9(8) + 3 = 75

Question 14.
Explain the Error
Rent-A-Tent rents party tents for a flat fee of $365 plus $125 a day. Capital Rentals rents party tents for a flat fee of $250 plus $175 a day. Delia wrote the following equation to find the number of days for which the total cost charged by the two companies would be the same:
365x + 125 = 250x + 175
Find and explain the error in Delia’s work. Then write the correct equation.
Type below:
____________

Answer:
Delia’s equation
365x + 125 = 250x + 175
The error is that she attached the variable with the flat fee (which is constant) and put the daily rent as a constant (which is variable).
Correct equation
125x + 365 = 175x + 250

Question 15.
Persevere in Problem Solving
Lilliana is training for a marathon. She runs the same distance every day for a week. On Monday, Wednesday, and Friday, she runs 3 laps on a running trail and then runs 6 more miles.On Tuesday and Sunday, she runs 5 laps on the trail and then runs 2 more miles. On Saturday, she just runs laps. How many laps does Lilliana run on Saturday?
________ laps

Answer:
2 laps

Explanation:
Miles covered by lap + Addition number of miles = 3x + 6
Miles covered by lap + Addition number of miles = 5x + 2
3x + 6 = 5x + 2
5x – 3x = 6 – 2
2x = 4
x = 4/2
x = 2

Guided Practice – Equations with Rational Numbers – Page No. 206

Question 1.
Sandy is upgrading her Internet service. Fast Internet charges $60 for installation and $50.45 per month. Quick Internet has free installation but charges $57.95 per month.
a. Write an equation that can be used to find the number of months at which the Internet service would cost the same.
Type below:
____________

Answer:
50.45x + 60 = 57.95x

Explanation:
Write an equation for Fast Internet, where x is the number of months.
Charge per Month × Number of Month + Installation Fee
50.45x + 60
Write an equation for Quick Internet, where x is the number of months.
Charge per Month × Number of Month + Installation Fee
57.95x
50.45x + 60 = 57.95x

Question 1.
b. Solve the equation.
_______ hours

Answer:
8

Explanation:
50.45x + 60 = 57.95x
57.95x – 50.45x = 60
7.5x = 60
x = 60/7.5
x = 8
The total cost will be the same for 8 months.

Solve.

Question 2.
\(\frac{3}{4}\) n – 18 = \(\frac{1}{4}\) n – 4
______

Answer:
n = 28

Explanation:
3/4 . n – 18 = 1/4 . n – 4
Determine the least common multiple of the denominators
LCM is 4
Multiply both sides of the equation by the LCM
4(3/4 . n – 18) = 4(1/4 . n – 4)
3n – 72 = n – 16
3n – n = -16 + 72
2n = 56
n = 56/2
n = 28

Question 3.
6 + \(\frac{4}{5}\) b = \(\frac{9}{10}\) b
_______

Answer:
b = 60

Explanation:
6 + \(\frac{4}{5}\) b = \(\frac{9}{10}\) b
LCM is 10
10(6 + \(\frac{4}{5}\) b) = 10(\(\frac{9}{10}\) b)
60 + 8b = 9b
9b – 8b = 60
b = 60

Question 4.
\(\frac{2}{11}\) m + 16 = 4 + \(\frac{6}{11}\) m
_______

Answer:
m = 33

Explanation:
\(\frac{2}{11}\) m + 16 = 4 + \(\frac{6}{11}\) m
The LCM is 11
11(\(\frac{2}{11}\) m + 16) = 11(4 + \(\frac{6}{11}\) m)
2m + 176 = 44 + 6m
6m – 2m = 176 – 44
4m = 132
m = 132/4
m = 33

Question 5.
2.25t + 5 = 13.5t + 14
_______

Answer:
t = -0.8

Explanation:
2.25t + 5 = 13.5t + 14
13.5t – 2.25t = 5 – 14
11.25t = -9
t = -9/11.25
t = -0.8

Question 6.
3.6w = 1.6w + 24
_______

Answer:
w = 12

Explanation:
3.6w = 1.6w + 24
3.6w – 1.6w = 24
2w = 24
w = 24/2
w = 12

Question 7.
-0.75p – 2 = 0.25p
_______

Answer:
p = -2

Explanation:
-0.75p – 2 = 0.25p
-2 = 0.25p + 0.75p
-2 = p
p = -2

Question 8.
Write a real-world problem that can be modeled by the equation 1.25x = 0.75x + 50.
Type below:
______________

Answer:
1.25x = 0.75x + 50.
Cell offer Plan A for no base fee and $1.25 per minute.
Cell offer Plan B for a $50 base fee and $0.75 per minute.
The equation shows when the total cost of the plan would be equal.

Essential Question Check-In

Question 9.
How does the method for solving equations with fractional or decimal coefficients and constants compare with the method for solving equations with integer coefficients and constants?
Type below:
______________

Answer:
When solving equations with fractional or decimal coefficients, the equations need to be multiplied by the multiple of denominator such that the equations have integer coefficients and constants.

Independent Practice – Equations with Rational Numbers – Page No. 207

Question 10.
Members of the Wide Waters Club pay $105 per summer season, plus $9.50 each time they rent a boat. Nonmembers must pay $14.75 each time they rent a boat. How many times would a member and a non-member have to rent a boat in order to pay the same amount?
_______ times

Answer:
20 times

Explanation:
Members of the Wide Waters Club pay $105 per summer season, plus $9.50 each time they rent a boat.
9.5x + $105
Nonmembers must pay $14.75 each time they rent a boat.
9.5x + $105 = 14.75x
9.5x – 14.75x = $105
5.25x = 105
x = 105/5.25
x = 20
The cost for members and non-members will be the same for 8 visits.

Question 11.
Margo can purchase tile at a store for $0.79 per tile and rent a tile saw for $24. At another store she can borrow the tile saw for free if she buys tiles there for $1.19 per tile. How many tiles must she buy for the cost to be the same at both stores?
_______ tiles

Answer:
60 tiles

Explanation:
Margo can purchase tile at a store for $0.79 per tile and rent a tile saw for $24.
0.79x + 24
At another store she can borrow the tile saw for free if she buys tiles there for $1.19 per tile.
1.19x
0.79x + 24 = 1.19x
1.19x – 0.79x = 24
0.4x = 24
x = 24/0.4
x = 60
Margo should buy 60 tiles for the cost to be the same at both stores.

Question 12.
The charges for two shuttle services are shown in the table. Find the number of miles for which the cost of both shuttles is the same.
Go Math Grade 8 Answer Key Chapter 7 Solving Linear Equations Lesson 2: Equations with Rational Numbers img 3
_______ miles

Answer:
40 miles

Explanation:
0.1x + 10
0.35x
0.1x + 10 = 0.35x
0.35x – 0.1x = 10
0.25x = 10
x = 10/0.25
x = 40
The cost of shuttles would be the same for 40 miles.

Question 13.
Multistep
Rapid Rental Car charges a $40 rental fee, $15 for gas, and $0.25 per mile driven. For the same car, Capital Cars charges $45 for rental and gas and $0.35 per mile.
a. For how many miles is the rental cost at both companies the same?
_______ miles

Answer:
100 miles

Explanation:
0.25x + 40 + 15 = 0.35x + 45
0.35x – 0.25x = 55 – 45
0.1x = 10
x = 10/0.1
x = 100
The cost of car rentals would be the same for 100 miles.

Question 13.
b. What is that cost?
$ _______

Answer:
$80

Explanation:
Let y be the total cost. Substitute 100 miles in any one of the two equations
y = 0.35x + 45
y = 0.35(100) + 45 = $80
Total cost would be $80.

Question 14.
Write an equation with the solution x = 20. The equation should have the variable on both sides, a fractional coefficient on the left side, and a fraction anywhere on the right side.
Type below:
______________

Answer:
4/3x + 10 = 50/3 + x

Explanation:
Write an equation with the solution x = 20. The equation should have the variable on both sides, a fractional coefficient on the left side, and a fraction anywhere on the right side.
1/3 . x = 1/3 . 20
1/3 . x + x = 1/3 . 20 + x
4/3x = 20/3 + x
4/3x + 10 = 20/3 + x + 10
4/3x + 10 = 50/3 + x

Question 15.
Write an equation with the solution x = 25. The equation should have the variable on both sides, a decimal coefficient on the left side, and a decimal anywhere on the right side. One of the decimals should be written in tenths, the other in hundredths.
Type below:
______________

Answer:
x=25
divide both sides by 25
x/25 = 1
convert 1/25 to decimal form 0.04
0.04x = 1
add x on both sides
1.04x = 1 + x
add 0.1 on both sides
1.04x + 0.1 = x + 1.1

Question 16.
Geometry
The perimeters of the rectangles shown are equal. What is the perimeter of each rectangle?
Go Math Grade 8 Answer Key Chapter 7 Solving Linear Equations Lesson 2: Equations with Rational Numbers img 4
Perimeter = _______

Answer:
Perimeter = 3.2

Explanation:
Perimeter of the first rectangle
P = 2(n + n + 0.6) = 2(2n + 0.6) = 4n + 1.2
Perimeter of the second rectangle
P = 2(n + 0.1 + 2n) = 2(3n + 0.1) = 6n + 0.2
the perimeter is equal
4n + 1.2 = 6n + 0.2
6n – 4n = 1.2 – 0.2
2n = 1
n = 1/2
n = 0.5
P = 4n + 1.2 = 4(0.5) + 1.2 = 3.2

Question 17.
Analyze Relationships
The formula F = 1.8C + 32 gives the temperature in degrees Fahrenheit (F) for a given temperature in degrees Celsius (C). There is one temperature for which the number of degrees Fahrenheit is equal to the number of degrees Celsius. Write an equation you can solve to find that temperature and then use it to find the temperature
Type below:
______________

Answer:
x = 1.8x + 32

Explanation:
F = 1.8C +32
let x be the temperature such that it is same in both celsius and in fahrenheit
Then the required equation is
x = 1.8x + 32
subtract 1.8x from both sides
-0.8x = 32
divide by -0.8 on both sides
x = -40
So -40 degree celsius

Equations with Rational Numbers – Page No. 208

Question 18.
Explain the Error
Agustin solved an equation as shown. What error did Agustin make? What is the correct answer?
Go Math Grade 8 Answer Key Chapter 7 Solving Linear Equations Lesson 2: Equations with Rational Numbers img 5
x = _______

Answer:
x = -12

Explanation:
Agustin did not multiply by 12 on both sides in step 2. He only partially multiplied the variable and left the constants as such, which doesn’t make any sense.
The correct solution is
12(x/3 – 4) = 12(3x/4 + 1)
4x – 48 = 9x + 12
subtract 12 on both sides
4x – 60 = 9x
subtract 4x on both sides
-60 = 5x
x = -12

H.O.T.

Focus on Higher Order Thinking

Question 19.
Draw Conclusions
Solve the equation \(\frac{1}{2} x-5+\frac{2}{3} x=\frac{7}{6} x+4\). Explain your results.
Type below:
_____________

Answer:
\(\frac{1}{2} x-5+\frac{2}{3} x=\frac{7}{6} x+4\)
The least common multiple of the denominators: LCM(2, 3, 6) = 6
6(\(\frac{1}{2} x-5+\frac{2}{3} x=\frac{7}{6} x+4\))
6.1/2x – 6.5 + 6.2/3x = 6.7/6x +6.4
3x – 30 + 4x = 7x + 24
7x – 30 = 7x + 24
-30 = 24
This is not true. The equation has no solution.

Question 20.
Look for a Pattern
Describe the pattern in the equation. Then solve the equation.
0.3x + 0.03x + 0.003x + 0.0003x + .. = 3
x = ______

Answer:
x = 9

Explanation:
0.3x + 0.03x + 0.003x + 0.0003x + .. = 3
0.3x = 3
0.9x = 9
x = 9

Question 21.
Critique Reasoning
Jared wanted to find three consecutive even integers whose sum was 4 times the first of those integers. He let k represent the first integer, then wrote and solved this equation : k + (k + 1) + (k + 2) = 4k. Did he get the correct answer? Explain.
__________

Answer:
No, it is wrong on two accounts.
First, he has not specified if k is even or not. An easy way of doing so would assume x to be any integer and k=2a
This ensures that k is an even integer.
Nest the question asks for 3 consecutive even integers, Jared just took 3 consecutive integers, and thus at least 1 of them is odd.
So correct representation would be
k + (k+2) + (k + 4) = 4k
which upon solving yields k=6

Guided Practice – Equations with the Distributive Property – Page No. 212

Solve each equation.

Question 1.
4(x + 8) – 4 = 34 – 2x
________

Answer:
x = 1

Explanation:
4(x + 8) – 4 = 34 – 2x
4x + 32 – 4 = 34 – 2x
4x + 2x = 34 – 28
6x = 6
x = 6/6
x = 1

Question 2.
\(\frac{2}{3}\)(9 + x) = -5(4 – x)
________

Answer:
x = 6

Explanation:
\(\frac{2}{3}\)(9 + x) = -5(4 – x)
2/3(9 + x) = -5(4 – x)
3 (2/3(9 + x)) = 3(-5(4 – x))
2(9 + x ) = -15 (4 – x)
18 + 2x = -60 + 15x
15x – 2x = 18 + 60
13x = 78
x = 78/13
x = 6

Question 3.
-3(x + 4) + 15 = 6 – 4x
________

Answer:
x = 3

Explanation:
-3(x + 4) + 15 = 6 – 4x
-3x – 12 + 15 = 6 – 4x
-3x + 3 = 6 – 4x
-3x + 4x = 6 – 3
x = 3

Question 4.
10 + 4x = 5(x – 6) + 33
________

Answer:
x = 7

Explanation:
10 + 4x = 5(x – 6) + 33
10 + 4x = 5x – 30 + 33
10 + 4x = 5x + 3
5x – 4x = 10 – 3
x = 7

Question 5.
x – 9 = 8(2x + 3) – 18
________

Answer:
x = -1

Explanation:
x – 9 = 8(2x + 3) – 18
x – 9 = 16x + 24 – 18
x – 9 = 16x + 6
16x – x = -9 – 6
15x = – 15
x = -15/15
x = -1

Question 6.
-6(x – 1) – 7 = -7x + 2
________

Answer:
x = 3

Explanation:
-6(x – 1) – 7 = -7x + 2
-6x + 6 – 7 = -7x + 2
-6x – 1 = -7x + 2
-7x + 6x = -1 -2
-x = -3
x = 3

Question 7.
\(\frac{1}{10}\)(x + 11) = -2(8 – x)
________

Answer:
x = 9

Explanation:
\(\frac{1}{10}\)(x + 11) = -2(8 – x)
10(\(\frac{1}{10}\)(x + 11)) = 10 (-2(8 – x))
x + 11 = -20(8 – x)
x + 11 = -160 + 20x
20x – x = 11 + 160
19x = 171
x = 171/19 = 9

Question 8.
-(4 – x) = \(\frac{3}{4}\)(x – 6)
________

Answer:
x = -2

Explanation:
-(4 – x) = \(\frac{3}{4}\)(x – 6)
4(-(4 – x)) = 4 (3/4(x – 6))
-16 + 4x = 3x – 18
4x – 3x = -18 + 16
x = -2

Question 9.
-8(8 – x) = \(\frac{4}{5}\)(x + 10)
________

Answer:
x = 10

Explanation:
-8(8 – x) = \(\frac{4}{5}\)(x + 10)
5(-8(8 – x)) = 5(\(\frac{4}{5}\)(x + 10))
-40(8 – x) = 4(x + 10)
-320 + 40x = 4x + 40
40x – 4x = 40 + 320
36x = 360
x = 360/36
x = 10

Question 10.
\(\frac{1}{2}\)(16 – x) = -12(x + 7)
________

Answer:
x = 8

Explanation:
\(\frac{1}{2}\)(16 – x) = -12(x + 7)
2 (\(\frac{1}{2}\)(16 – x)) = 2 (-12(x + 7))
16 – x = -24 (x + 7)
16 – x = -24x – 168
24x – x = -168 – 16
23x = 184
x = 184/23
x = 8

Question 11.
Sandra saves 12% of her salary for retirement. This year her salary was $3,000 more than in the previous year, and she saved $4,200.What was her salary in the previous year?
Write an equation _____
Sandra’s salary in the previous year was _____
Salary = $ _____

Answer:
Write an equation 0.12x + 360 = 4200
Sandra’s salary in the previous year was $32000
Salary = $3000

Explanation:
0.12(x + 3000) = 4200
0.12x + 360 = 4200
0.12x = 4200 – 360
0.12x = 3840
x = 3840/0.12
x = 32000
Sandra’s salary in the previous year was $32000

Essential Question Check-In

Question 12.
When solving an equation using the Distributive Property, if the numbers being distributed are fractions, what is your first step? Why?
Type below:
___________

Answer:
Multiply both sides by the denominator of the fraction

Independent Practice – Equations with the Distributive Property – Page No. 213

Question 13.
Multistep
Martina is currently 14 years older than her cousin Joey. In 5 years she will be 3 times as old as Joey. Use this information to answer the following questions.
a. If you let x represent Joey’s current age, what expression can you use to represent Martina’s current age?
Type below:
___________

Answer:
y = x + 14

Explanation:
y = x + 14
where x is Joey’s current age and t is Martna’s current age.

Question 13.
b. Based on your answer to part a, what expression represents Joey’s age in 5 years? What expression represents Martina’s age in 5 years?
Type below:
___________

Answer:
Ages in 5 years
Joey’s age = x + 5
Martina’s age = x + 14 + 5 = x + 19

Question 13.
c. What equation can you write based on the information given?
Type below:
___________

Answer:
3(x + 5) = x + 19

Explanation:
In 5 years, Martina will be three times as old as Joey
3(x + 5) = x + 19

Question 13.
d. What is Joey’s current age? What is Martina’s current age?
Joey’s current age ___________
Martina’s current age ___________

Answer:
Joey’s current age 2
Martina’s current age 16

Explanation:
3(x + 5) = x + 19
3x + 15 = x + 19
3x – x = 19 – 15
2x = 4
x = 2

Question 14.
As part of a school contest, Sarah and Luis are playing a math game. Sarah must pick a number between 1 and 50 and give Luis clues so he can write an equation to find her number. Sarah says, “If I subtract 5 from my number, multiply that quantity by 4, and then add 7 to the result, I get 35.” What equation can Luis write based on Sarah’s clues and what is Sarah’s number?
Type below:
___________

Answer:
x = 12

Explanation:
As part of a school contest, Sarah and Luis are playing a math game. Sarah must pick a number between 1 and 50 and give Luis clues so he can write an equation to find her number. Sarah says, “If I subtract 5 from my number, multiply that quantity by 4, and then add 7 to the result, I get 35.”
4 (x – 5) + 7 = 35
4x – 20 + 7 = 35
4x – 13 = 35
4x = 35 + 13
4x = 48
x = 48/4
x = 12

Question 15.
Critical Thinking
When solving an equation using the Distributive Property that involves distributing fractions, usually the first step is to multiply by the LCD to eliminate the fractions in order to simplify computation. Is it necessary to do this to solve \(\frac{1}{2}\)(4x + 6) = 13(9x – 24)? Why or why not?
___________

Answer:
It is not necessary. In this case, distributing the fractions directly results in whole-number coefficients and constants, however, if the results are not in whole-number coefficients and constants it is harder to solve fractions.

Question 16.
Solve the equation given in Exercise 15 with and without using the LCD of the fractions. Are your answers the same?
___________

Answer:
x = 11

Explanation:
\(\frac{1}{2}\)(4x + 6) = 13(9x – 24)
6(\(\frac{1}{2}\)(4x + 6)) = 6(13(9x – 24))
3(4x + 6) = 2(9x – 24)
12x + 18 = 18x – 48
18x – 12x = 18 + 48
6x = 66
x = 66/6
x = 11

Equations with the Distributive Property – Page No. 214

Question 17.
Represent Real-World Problems
A chemist mixed x milliliters of 25% acid solution with some 15% acid solution to produce 100 milliliters of a 19% acid solution. Use this information to fill in the missing information in the table and answer the questions that follow.
Go Math Grade 8 Answer Key Chapter 7 Solving Linear Equations Lesson 3: Equations with the Distributive Property img 6
a. What is the relationship between the milliliters of acid in the 25% solution, the milliliters of acid in the 15% solution, and the milliliters of acid in the mixture?
Type below:
_____________

Answer:
The milliliters of acid in the 25% solution plus the milliliters of acid in the 15% solution equals the milliliters of acid in the mixture

Explanation:
grade 8 chapter 7 image 9

Question 17.
b. What equation can you use to solve for x based on your answer to part a?
Type below:
_____________

Answer:
0.25x + 0.15(100 – x) = 19

Question 17.
c. How many milliliters of the 25% solution and the 15% solution did the chemist use in the mixture?
Go Math Grade 8 Answer Key Chapter 7 Solving Linear Equations Lesson 3: Equations with the Distributive Property img 7
Type below:
_____________

Answer:
0.25x + 0.15(100 – x) = 19
0.25x + 15 – 0.15x = 19
0.1x + 15 = 19
0.1x = 4
x = 4/0.1
x = 40
The chemist used 40ml of the 25% solution and 100 – 40 = 60ml of the 15% solution.

H.O.T.

Focus on Higher Order Thinking

Question 18.
Explain the Error
Anne solved 5(2x) – 3 = 20x + 15 for x by first distributing 5 on the left side of the equation. She got the answer x = -3. However, when she substituted -3 into the original equation for x, she saw that her answer was wrong. What did Anne do wrong, and what is the correct answer?
x = ________

Answer:
x = -1.8

Explanation:
Dado que 5 solo se multiplica por 2x, no tiene sentido usar la distribución aquí. Básicamente, distribuir 5 fue el problema
Solución correcta:
5 (2x) – 3 = 20x + 15
10x -3 = 20x + 15
restar 15 en ambos lados
10x – 18 = 20x
restar 10x de ambos lados
-18 = 10x
x = -1.8

Question 19.
Communicate Mathematical Ideas
Explain a procedure that can be used to solve 5[3(x + 4) – 2(1 – x)] – x – 15 = 14x + 45. Then solve the equation.
x = ________

Answer:
x = 1

Explanation:
5[3(x + 4) – 2(1 – x)] – x – 15 = 14x + 45
5[3x + 12 – 2 + 2x] – x – 15 = 14x + 45
5[5x + 10] – x – 15 = 14x + 45
25x + 50 – x – 15 = 14x + 45
24x + 35 = 14x + 45
24x – 14x = 45 – 35
10x = 10
x = 1

Guided Practice – Equations with Many Solutions or No Solution – Page No. 218

Use the properties of equality to simplify each equation. Tell whether the final equation is a true statement.

Question 1.
Go Math Grade 8 Answer Key Chapter 7 Solving Linear Equations Lesson 4: Equations with Many Solutions or No Solution img 8
The statement is: _______

Answer:
The statement is: true

Explanation:
3x – 2 = 25 – 6x
3x + 6x -2 = 25 -6x + 6x
9x – 2 = 25
9x -2 + 2 = 25 + 2
9x = 27
x = 27/9
x = 3
The statement is true.

Question 2.
Go Math Grade 8 Answer Key Chapter 7 Solving Linear Equations Lesson 4: Equations with Many Solutions or No Solution img 9
____________

Answer:
The statement is false.

Explanation:
2x – 4 = 2(x – 1) + 3
2x – 4 = 2x – 2 + 3
2x – 4 = 2x + 1
2x – 4 – 2x = 2x + 1 – 2x
-4 not equal to 1
The statement is false.

Question 3.
How many solutions are there to the equation in Exercise 2?
____________

Answer:
There is no solution to exercise 2.

Question 4.
After simplifying an equation, Juana gets 6 = 6. Explain what this means.
____________

Answer:
When 6 = 6, there are infinite solutions.

Write a linear equation in one variable that has infinitely many solutions.

Question 5.
Start with a _____ statement.
Add the _____ to both sides.
Add the _____ to both sides.
Combine _____ terms.
Go Math Grade 8 Answer Key Chapter 7 Solving Linear Equations Lesson 4: Equations with Many Solutions or No Solution img 10
Type below:
____________

Answer:
Start with a “true” statement
Add the “same variable” to both sides
Add the “same constant” to both sides
Combine “like” terms

Explanation:
Start with a “true” statement
10 = 10
Add the “same variable” to both sides
10 + x = 10 + x
Add the “same constant” to both sides
10 + x + 5 = 10 + x + 5
Combine “like” terms
15 + x = 15 + x

Essential Question Check-In

Question 6.
Give an example of an equation with an infinite number of solutions. Then make one change to the equation so that it has no solution.
Type below:
____________

Answer:
An equation with infinitely many solutions
x – 2x + 3 = 3 – x
-x + 3 = 3 – x
+x/3 = +x/3
An equation for no solution
x – 2x + 3 = 3 – x + 4
-x + 3 = 7 – x
-x/3 = -x/7

Independent Practice – Equations with Many Solutions or No Solution – Page No. 219

Tell whether each equation has one, zero, or infinitely many solutions.

Question 7.
-(2x + 2) – 1 = -x – (x + 3)
____________

Answer:
The statement is true

Explanation:
-(2x + 2) – 1 = -x – (x + 3)
-2x – 2 – 1 = -x – x + 3
-2x – 3 = -2x + 3
-3 = -3
The statement is true

Question 8.
-2(z + 3) – z = -z – 4(z + 2)
____________

Answer:
The statement is false.

Explanation:
-2(z + 3) – z = -z – 4(z + 2)
-3z – 6 = -3z -8
-3z -6 + 3z = -3z – 8 + 3z
-6 not equal to -8
The statement is false.

Create an equation with the indicated number of solutions.

Question 9.
No solution:
3(x – \(\frac{4}{3}\)) = 3x + _____
Type below:
______________

Answer:
3(x – \(\frac{4}{3}\)) = 3x + ?
3x – 4 = 3x + ?
3x – 4 = 3x + 2
When there is no solution, the statement should be false. Any number except -4 would make the equation have no solutions.

Question 10.
Infinitely many solutions:
2(x – 1) + 6x = 4( _____ – 1) + 2
Type below:
______________

Answer:
2(x – 1) + 6x = 4( _____ – 1) + 2
2(x – 1) + 6x = 4( ? – 1) + 2
2x – 2 + 6x = 4(? – 1) + 2
8x – 2 = 4(? – 1) + 2
8x – 2 = 4(2x – 1) + 2
8x – 2 = 8x – 4 + 2
8x – 2 = 8x – 2
When there are infinitely many solutions, the statement should be true

Question 11.
One solution of x = -1:
5x – (x – 2) = 2x – ( _____ )
Type below:
______________

Answer:
Put x = -1 in the equation
-5 – (-1 – 2) = -2 – blank
simplifying
-2 = -2 – blank
add 2 on both sides
0 = blank

Question 12.
Infinitely many solutions:
-(x – 8) + 4x = 2( _____ ) + x
Type below:
______________

Answer:
-(x – 8) + 4x = 2( ?) + x
-x + 8 + 4x = 2(?) + x
3x + 8 = 2(?) + x
3x + 8 = 2 (x + 4) + x
3x + 8 = 2x + 8x + x
3x + 8 = 3x + 8
When there are infinitely many solutions, the statement should be true.

Question 13.
Persevere in Problem Solving
The Dig It Project is designing two gardens that have the same perimeter. One garden is a trapezoid whose nonparallel sides are equal. The other is a quadrilateral. Two possible designs are shown at the right.
Go Math Grade 8 Answer Key Chapter 7 Solving Linear Equations Lesson 4: Equations with Many Solutions or No Solution img 11
a. Based on these designs, is there more than one value for x? Explain how you know this.
______________

Answer:
There are more than one value of x

Explanation:
Perimeter of the trapezoid
P = 2x – 2 + x + 1 + x + x + 1 = 5x
Perimeter of the quadrilateral
P = 2x – 9 + x + x + 8 + x + 1 = 5x
5x = 5x
There are more than one value of x

Question 13.
b. Why does your answer to part a make sense in this context?
Type below:
______________

Answer:
The condition was that the two perimeters are to be equal. However, a specific number was not given, so there are an infinite number of possible perimeters

Explanation:
Interpretation of part a in this context
The condition was that the two perimeters are to be equal. However, a specific number was not given, so there are an infinite number of possible perimeters

Question 13.
c. Suppose the Dig It Project wants the perimeter of each garden to be 60 meters. What is the value of x in this case? How did you find this?
______ meters

Answer:
12 meters

Explanation:
2x – 2 + x + 1 + x + x + 1 = 60
5x = 60
x = 60/5
x = 12

Equations with Many Solutions or No Solution – Page No. 220

Question 14.
Critique Reasoning
Lisa says that the indicated angles cannot have the same measure. Marita disagrees and says she can prove that they can have the same measure. Who do you agree with? Justify your answer.
Go Math Grade 8 Answer Key Chapter 7 Solving Linear Equations Lesson 4: Equations with Many Solutions or No Solution img 12
I agree with: ______________

Answer:
I agree with: Marita

Explanation:
9x – 25 + x = x + 50 + 2x – 12
10x – 25 = 3x + 38
10x – 3x = 38 + 25
7x = 63
x = 63/7
x = 9
When x = 9 the angles will be same and for any other value of x, the angles will not be the same.

Question 15.
Represent Real-World Problems
Adele opens an account with $100 and deposits $35 a month. Kent opens an account with $50 and also deposits $35 a month. Will they have the same amount in their accounts at any point? If so, in how many months and how much will be in each account? Explain.
______________

Answer:
Adele’s amount after x months
A = 100 + 35x
Kent’s amount after x months
A = 50 + 35x
100 + 35x = 50 + 35x
100 is not equal to 50
The statement is false, the amounts in two accounts would never be equal.

H.O.T.

Focus on Higher Order Thinking

Question 16.
Communicate Mathematical Ideas
Frank solved an equation and got the result x = x. Sarah solved the same equation and got 12 = 12. Frank says that one of them is incorrect because you cannot get different results for the same equation. What would you say to Frank? If both results are indeed correct, explain how this happened.
Frank is: ____________

Answer:
Both of them can be correct as both equations give the same result i.e. there are infinitely many solutions. Frank eliminated the constant from both sides while Sarah eliminated the variable from both sides.

Question 17.
Critique Reasoning
Matt said 2x – 7 = 2(x – 7) has infinitely many solutions. Is he correct? Justify Matt’s answer or show how he is incorrect.
Matt is: ____________

Answer:

Explanation:
2x – 7 = 2(x – 7)
2x – 7 = 2x – 14
-7 not equal to -14
The statement is false, there is no solution. Matt is incorrect.

7.1 Equations with the Variable on Both Sides – Model Quiz – Page No. 221

Solve.

Question 1.
4a – 4 = 8 + a
_______

Answer:
a = 4

Explanation:
4a – 4 = 8 + a
4a – a = 8 + 4
3a = 12
a = 12/3
a = 4

Question 2.
4x + 5 = x + 8
_______

Answer:
x = 1

Explanation:
4x + 5 = x + 8
4x – x = 8 – 5
3x = 3
x = 3/3
x = 1

Question 3.
Hue is arranging chairs. She can form 6 rows of a given length with 3 chairs left over, or 8 rows of that same length if she gets 11 more chairs. Write and solve an equation to find how many chairs are in that row length.
_______ chairs

Answer:
7 chairs

Explanation:
Hue is arranging chairs. She can form 6 rows of a given length with 3 chairs left over, or 8 rows of that same length if she gets 11 more chairs.
6x + 3 = 8x – 11
8x – 6x = 3 + 11
2x = 14
x = 14/2
x = 7
There are 7 chairs in each row.

7.2 Equations with Rational Numbers

Solve.

Question 4.
\(\frac{2}{3} n-\frac{2}{3}=\frac{n}{6}+\frac{4}{3}\)
_______

Answer:
n = 4

Explanation:
\(\frac{2}{3} n-\frac{2}{3}=\frac{n}{6}+\frac{4}{3}\)
The LCM is 6.
6(2/3n – 2/3) = 6(n/6 + 4/3)
6(2/3n) -6(2/3) = 6(n/6) + 6(4/3)
4n – 4 = n + 8
4n – n = 8 + 4
3n = 12
n = 12/3
n = 4

Question 5.
1.5d + 3.25 = 1 + 2.25d
_______

Answer:
d = 3

Explanation:
1.5d + 3.25 = 1 + 2.25d
2.25d – 1.5d = 3.25 – 1
0.75d = 2.25
d = 2.25/0.75
d = 3

Question 6.
Happy Paws charges $19.00 plus $1.50 per hour to keep a dog during the day. Woof Watchers charges $14.00 plus $2.75 per hour. Write and solve an equation to find for how many hours the total cost of the services is equal.
_______ hours

Answer:
3.2 hours

Explanation:
Happy Paws charges $19.00 plus $1.50 per hour to keep a dog during the day.
1.5x + 19
Woof Watchers charges $14.00 plus $2.75 per hour.
2.75x + 15
1.5x + 19 = 2.75x + 15
2.75x – 1.5x = 19 – 15
1.25x = 4
x = 4/1.25
x = 3.2
The total cost of the services is equal after 3.2 hrs.

7.3 Equations with the Distributive Property

Solve.

Question 7.
14 + 5x = 3(-x + 3) – 11
_______

Answer:
x = -2

Explanation:
14 + 5x = 3(-x + 3) – 11
14 + 5x = -3x + 9 – 11
14 + 5x = -3x – 2
5x + 3x = -2 –  14
8x = – 16
x = -16/8
x = -2

Question 8.
\(\frac{1}{4}\)(x – 7) = 1 + 3x
_______

Answer:
x = -1

Explanation:
\(\frac{1}{4}\)(x – 7) = 1 + 3x
4(\(\frac{1}{4}\)(x – 7)) = 4(1 + 3x)
(x – 7) = 4 + 12x
12x – x = -7 – 4
11x = -11
x = -11/11
x = -1

Question 9.
-5(2x – 9) = 2(x – 8) – 11
_______

Answer:
x = 6

Explanation:
-5(2x – 9) = 2(x – 8) – 11
-10x + 45 = 2x – 16 – 11
-10x + 45 = 2x – 27
2x + 10x = 45 + 27
12x = 72
x = 72/12
x = 6

Question 10.
3(x + 5) = 2(3x + 12)
_______

Answer:
x = -3

Explanation:
3(x + 5) = 2(3x + 12)
3x + 15 = 6x + 24
6x – 3x = 15 – 24
3x = -9
x = -9/3
x = -3

7.4 Equations with Many Solutions or No Solution

Tell whether each equation has one, zero, or infinitely many solutions.

Question 11.
5(x – 3) + 6 = 5x – 9
____________

Answer:
There are infinitely many solutions

Explanation:
5(x – 3) + 6 = 5x – 9
5x – 15 + 6 = 5x – 9
5x – 9 = 5x – 9
The statement is true. There are infinitely many solutions.

Question 12.
5(x – 3) + 6 = 5x – 10
____________

Answer:
There are no solutions

Explanation:
5(x – 3) + 6 = 5x – 10
5x – 15 + 6 = 5x – 10
5x – 9 = 5x – 10
-9 not equal to -10
The statement is false. There are no solutions.

Question 13.
5(x – 3) + 6 = 4x + 3
____________

Answer:
There is one solution

Explanation:
5(x – 3) + 6 = 4x + 3
5x – 15 + 6 = 4x + 3
5x – 9 = 4x + 3
5x – 4x = 3 + 9
x = 12
There is one solution

Selected Response – Mixed Review – Page No. 222

Question 1.
Two cars are traveling in the same direction. The first car is going 40 mi/h, and the second car is going 55 mi/h. The first car left 3 hours before the second car. Which equation could you solve to find how many hours it will take for the second car to catch up to the first car?
Options:
a. 55t + 3 = 40t
b. 55t + 165 = 40t
c. 40t + 3 = 55t
d. 40t + 120 = 55t

Answer:
d. 40t + 120 = 55t

Explanation:
Two cars are traveling in the same direction. The first car is going 40 mi/h, and the second car is going 55 mi/h. The first car left 3 hours before the second car.
3 × 40 + 40t = 120 + 40t
55t
40t + 120 = 55t

Question 2.
Which linear equation is represented by the table?
Go Math Grade 8 Answer Key Chapter 7 Solving Linear Equations Mixed Review img 13
Options:
a. y = -x + 5
b. y = 2x – 1
c. y = x + 3
d. y = -3x + 11

Answer:
a. y = -x + 5

Explanation:
Find the slope using
m = (y2 – y1)/(x2 – x1)
where (x1, y1) = (3, 2), (x2, y2) = (1, 4)
Slope = (4 – 2)/(1 – 3) = -2/2 = -1

Question 3.
Shawn’s Rentals charges $27.50 per hour to rent a surfboard and a wetsuit. Darla’s Surf Shop charges $23.25 per hour to rent a surfboard plus $17 extra for a wetsuit. For what total number of hours are the charges for Shawn’s Rentals the same as the charges for Darla’s Surf Shop?
Options:
a. 3
b. 4
c. 5
d. 6

Answer:
b. 4

Explanation:
Shawn’s Rentals charges $27.50 per hour to rent a surfboard and a wetsuit.
27.5x
Darla’s Surf Shop charges $23.25 per hour to rent a surfboard plus $17 extra for a wetsuit.
23.25x + 17
23.25x + 17 = 27.5x
27.5x – 23.25x = 17
4.25x = 17
x = 17/4.25
x = 4
The charge would be equal after 4 hrs

Question 4.
Which of the following is irrational?
Options:
a. -8
b. 4.63
c. \(\sqrt { x } \)
d. \(\frac{1}{3}\)

Answer:
c. \(\sqrt { x } \)

Explanation:
\(\sqrt { x } \) is irrational

Question 5.
Greg and Jane left a 15% tip after dinner. The amount of the tip was $9. Greg’s dinner cost $24. Which equation can you use to find x, the cost of Jane’s dinner?
Options:
a. 0.15x + 24 = 9
b. 0.15(x + 24) = 9
c. 15(x + 24) = 9
d. 0.15x = 24 + 9

Answer:
b. 0.15(x + 24) = 9

Explanation:
Let x be the cost of Jane’s dinner. The amount of tip is the 15% of the total cost of dinner.
0.15(x + 24) = 9

Question 6.
For the equation 3(2x − 5) = 6x + k, which value of k will create an equation with infinitely many solutions?
Options:
a. 15
b. -5
c. 5
d. -15

Answer:
d. -15

Explanation:
3(2x – 5) = 6x + k
6x – 15 = 6x + k
6x – 15 = 6x – 15
The statement is true. k = -15

Question 7.
Which of the following is equivalent to 2−4?
Options:
a. \(\frac{1}{16}\)
b. \(\frac{1}{8}\)
c. -2
d. -16

Answer:
a. \(\frac{1}{16}\)

Explanation:
2−4
1/24
1/16

Mini-Task

Question 8.
Use the figures below for parts a and b.
Go Math Grade 8 Answer Key Chapter 7 Solving Linear Equations Mixed Review img 14
a. Both figures have the same perimeter. Solve for x.
_______

Answer:
x=12

Explanation:
4x+10=3x+22
4x – 3x = 22 – 10
x = 12
Answer: x=12

Question 8.
b. What is the perimeter of each figure?
_______

Answer:
Both are 58

Explanation:
x + x + 5 + x + x + 5
12 + 12 + 5 + 12 + 12 + 5
58
x + 7 + x + 4 + x + 11
12 + 7 + 12 + 4 + 12 + 11
58

Conclusion:

Go Math Grade 8 Answer Key is the best source to enhance your math skills. Learn all the solutions from Go Math Grade 8 Answer Key Chapter 7 Solving Linear Equations and complete your homework. Hope the information provided in Go Math Grade 8 Chapter 7 Solving Linear Equations Answer Key is satisfactory for all the 8th standard students. Stay with our CCSS Math Answers website to get the solutions of all grade 8 chapters.

Go Math Grade 4 Answer Key Chapter 9 Relate Fractions and Decimals

go-math-grade-4-chapter-9-relate-fractions-and-decimals-answer-key

Go Math Grade 4 Answer Key Chapter 9 Relate Fractions and Decimals assists you to examine your preparation level. Get numerous practice questions and exercise questions of Chapter 9 from the 4th Grade Go Math Answer Key Ch 9 Relate Fractions and Decimals and secure high marks. We have provided step-by-step solutions for all the problems covered in HMH Go math Grade 4 Answer Key Chapter Test, Practice Test, Assessment Tests. So that you can understand the Chapter 9 topics very easily.

HMH Go Math Grade 4 Chapter 9 Relate Fractions and Decimals Answer Key

While practicing the concepts of Chapter 9 Relate Fractions and Decimals, click on the links available over here and download the HMH Go Math Grade 4 Chapter 9 Relate Fractions and Decimals Solution Key for free. Hence, students can seek help to examine their strengths and weaknesses using the 4th Grade HMH Go Math  Ch 9 Relate Fractions and Decimals Answer Key.

Lesson 1: Relate Tenths and Decimals

Lesson 2: Relate Hundredths and Decimals

Lesson 3: Equivalent Fractions and Decimals

Lesson 4: Relate Fractions, Decimals, and Money

Lesson 5: Problem Solving • Money

Mid-Chapter Checkpoint

Lesson 6: Add Fraction Parts of 10 and 100

Lesson 7: Compare Decimals

Review/Test

Common Core – New – Page No. 499

Relate Tenths and Decimals

Write the fraction or mixed number and the decimal shown by the model.

Question 1
Go Math Grade 4 Answer Key Chapter 9 Relate Fractions and Decimals Common Core - New img 1

Answer:
Go Math Grade 4 Answer Key Chapter 9 Relate Fractions and Decimals Common Core - New img 1

Question 2.
Go Math Grade 4 Answer Key Chapter 9 Relate Fractions and Decimals Common Core - New img 2
Type below:
________

Answer:
1\(\frac{2}{10}\)

Explanation:
The model is divided into 10 equal parts. Each part represents one-tenth.
1 2/10 is 1 whole and 2 tenths.

Question 3.
Go Math Grade 4 Answer Key Chapter 9 Relate Fractions and Decimals Common Core - New img 3
Type below:
________

Answer:
2\(\frac{3}{10}\) = 2.3

Explanation:
grade 4 chapter 9 Common Core Image 1 499

Question 4.
Go Math Grade 4 Answer Key Chapter 9 Relate Fractions and Decimals Common Core - New img 4
Type below:
________

Answer:
4\(\frac{8}{10}\) = 4.8

Explanation:
grade 4 chapter 9 Common Core Image 2 499

Write the fraction or mixed number as a decimal.

Question 5.
\(\frac{4}{10}\)
_____

Answer:
0.4

Explanation:
Write down 4 with the decimal point 1 space from the right (because 10 has 1 zero)
0.4

Question 6.
3 \(\frac{1}{10}\)
_____

Answer:
3.1

Explanation:
Multiply 3 x 10 = 30.
Add 30 + 1 = 31.
So, 31/10.
Write down 31 with the decimal point 1 space from the right (because 10 has 1 zero)
3.1

Question 7.
\(\frac{7}{10}\)
_____

Answer:
0.7

Explanation:
Write down 7 with the decimal point 1 space from the right (because 10 has 1 zero)
0.7

Question 8.
6 \(\frac{5}{10}\)
_____

Answer:
6.5

Explanation:
Multiply 6 x 10 = 60.
Add 60 + 5 = 65.
So, 65/10.
Write down 35 with the decimal point 1 space from the right (because 10 has 1 zero)
6.5

Question 9.
\(\frac{9}{10}\)
_____

Answer:
0.9

Explanation:
Write down 9 with the decimal point 1 space from the right (because 10 has 1 zero)
0.9

Problem Solving

Question 10.
There are 10 sports balls in the equipment closet. Three are kickballs. Write the portion of the balls that are kickballs as a fraction, as a decimal, and in word form.
Type below:
_________

Answer:
\(\frac{3}{10}\) = 0.3 = three tenths

Explanation:
There are 10 sports balls in the equipment closet. Three are kickballs. So, 3/10 kickballs are available.

Question 11.
Peyton has 2 pizzas. Each pizza is cut into 10 equal slices. She and her friends eat 14 slices. What part of the pizzas did they eat? Write your answer as a decimal.
_________

Answer:
1.4 pizzas

Explanation:
Peyton has 2 pizzas. Each pizza is cut into 10 equal slices.
So, total number of slices = 2 x 10 = 20.
She and her friends eat 14 slices.
So, they ate 1 whole pizza and 4 parts out of 10 slices in the second pizza.
1 4/10 = 14/10 = 1.4 pizzas

Common Core – New – Page No. 500

Lesson Check

Question 1.
Valerie has 10 CDs in her music case. Seven of the CDs are pop music CDs. What is this amount written as a decimal?
Options:
a. 70.0
b. 7.0
c. 0.7
d. 0.07

Answer:
c. 0.7

Explanation:
Valerie has 10 CDs in her music case. Seven of the CDs are pop music CDs.
Seven CDs out of 10 CDs = 7/10 =0.7

Question 2.
Which decimal amount is modeled below?
Go Math Grade 4 Answer Key Chapter 9 Relate Fractions and Decimals Common Core - New img 5
Options:
a. 140.0
b. 14.0
c. 1.4
d. 0.14

Answer:
c. 1.4

Explanation:
1\(\frac{4}{10}\)
Multiply 10 x 1 = 10.
Add 10 + 4 = 14.
So, 14/10 = 1.4.

Spiral Review

Question 3.
Which number is a factor of 13?
Options:
a. 1
b. 3
c. 4
d. 7

Answer:
a. 1

Explanation:
13 has 1 and 13 as its factors.

Question 4.
An art gallery has 18 paintings and 4 photographs displayed in equal rows on a wall, with the same number of each type of art in each row. Which of the following could be the number of rows?
Options:
a. 2 rows
b. 3 rows
c. 4 rows
d. 6 rows

Answer:
a. 2 rows

Explanation:
An art gallery has 18 paintings and 4 photographs displayed in equal rows on a wall, with the same number of each type of art in each row. So, 18 paintings and 4 photographs need to be divided into equal parts.
18/2 = 9; 4/2 = 2.
2 rows can be possible with 9 pictures and 2 pictures in each row.

Question 5.
How do you write the mixed number shown as a fraction greater than 1?
Go Math Grade 4 Answer Key Chapter 9 Relate Fractions and Decimals Common Core - New img 6
Options:
a. \(\frac{32}{5}\)
b. \(\frac{14}{4}\)
c. \(\frac{6}{4}\)
d. \(\frac{4}{4}\)

Answer:
b. \(\frac{14}{4}\)

Explanation:
3\(\frac{2}{4}\) = 14/4. 14 divided by 4 is equal to 3 with a remainder of 2. The 3 is greater than 1. So, 14/4 > 1.

Question 6.
Which of the following models has an amount shaded that is equivalent to the fraction \(\frac{1}{5}\)?
a. Go Math Grade 4 Answer Key Chapter 9 Relate Fractions and Decimals Common Core - New img 7
b. Go Math Grade 4 Answer Key Chapter 9 Relate Fractions and Decimals Common Core - New img 8
c. Go Math Grade 4 Answer Key Chapter 9 Relate Fractions and Decimals Common Core - New img 9
d. Go Math Grade 4 Answer Key Chapter 9 Relate Fractions and Decimals Common Core - New img 10

Answer:
c. Go Math Grade 4 Answer Key Chapter 9 Relate Fractions and Decimals Common Core - New img 9

Explanation:
a. \(\frac{2}{3}\)
b. \(\frac{5}{10}\) = \(\frac{1}{2}\)
c. \(\frac{2}{10}\) = \(\frac{1}{5}\)
d. \(\frac{1}{10}\)

Page No. 503

Question 1.
Shade the model to show \(\frac{31}{100}\).
Write the amount as a decimal.
Go Math Grade 4 Answer Key Chapter 9 Relate Fractions and Decimals img 11
_____

Answer:
grade 4 chapter 9 Relate Fractions and Decimals Image 1 503

Write the fraction or mixed number and the decimal shown by the model.

Question 2.
Go Math Grade 4 Answer Key Chapter 9 Relate Fractions and Decimals img 12
Type below:
_________

Answer:
\(\frac{68}{100}\) = 0.68

Explanation:
68 boxes are shaded out of 100 boxes.

Question 3.
Go Math Grade 4 Answer Key Chapter 9 Relate Fractions and Decimals img 13
Type below:
_________

Answer:
\(\frac{8}{100}\) = 0.08

Explanation:
8 boxes are shaded out of 100 boxes.

Question 4.
Go Math Grade 4 Answer Key Chapter 9 Relate Fractions and Decimals img 14
Type below:
_________

Answer:
6\(\frac{19}{100}\) = 6.19

Explanation:
0.5 is 5 tenths and 0.50 is 5 tenths 0 hundredths. Since both 0.5 and 0.50 have 5 tenths and no hundredths, they are equivalent

Write the fraction or mixed number and the decimal shown by the model.

Question 5.
Go Math Grade 4 Answer Key Chapter 9 Relate Fractions and Decimals img 15
Type below:
_________

Answer:
1\(\frac{83}{100}\) = 1.83

Explanation:
1 whole number(all the square boxes are shaded) and 83 squares boxes shaded out from 100 boxes.

Question 6.
Go Math Grade 4 Answer Key Chapter 9 Relate Fractions and Decimals img 16
Type below:
_________

Answer:
\(\frac{75}{100}\)

Explanation:
75 boxes are shaded out of 100 boxes.

Question 7.
Go Math Grade 4 Answer Key Chapter 9 Relate Fractions and Decimals img 17
Type below:
_________

Answer:
\(\frac{47}{100}\) = 0.47

Explanation:
The point lies between \(\frac{40}{100}\) and \(\frac{50}{100}\). The number of lines in between \(\frac{40}{100}\) and \(\frac{50}{100}\) are 10. The point is placed at 7th line. So, 40 + 7 = 47. Answer = \(\frac{47}{100}\)

Practice: Copy and Solve Write the fraction or mixed number as a decimal.

Question 8.
\(\frac{9}{100}\) = _____

Answer:
0.09

Explanation:
Write down 9 with the decimal point 2 spaces from the right (because 100 has 2 zeros)

Question 9.
4 \(\frac{55}{100}\) = _____

Answer:
4.55

Explanation:
4 \(\frac{55}{100}\) = \(\frac{455}{100}\)
Write down 455 with the decimal point 2 spaces from the right (because 100 has 2 zeros). So, 4.55 is the answer

Question 10.
\(\frac{10}{100}\) = _____

Answer:
0.10 = 0.1

Explanation:
Write down 10 with the decimal point 2 spaces from the right (because 100 has 2 zeros). So, 0.10 =0.1 is the answer

Question 11.
9 \(\frac{33}{100}\) = _____

Answer:
9.33

Explanation:
9 \(\frac{33}{100}\) = \(\frac{933}{100}\)
Write down 933 with the decimal point 2 spaces from the right (because 100 has 2 zeros). So, 9.33 is the answer.

Question 12.
\(\frac{92}{100}\) = _____

Answer:
0.92

Explanation:
Write down 92 with the decimal point 2 spaces from the right (because 100 has 2 zeros). So, 0.92 is the answer

Question 13.
14 \(\frac{16}{100}\) = _____

Answer:
14.16

Explanation:
14 \(\frac{16}{100}\) = \(\frac{1416}{100}\)
Write down 1416 with the decimal point 2 spaces from the right (because 100 has 2 zeros). So, 14.16 is the answer.

Page No. 504

Question 14.
Shade the grids to show three different ways to represent \(\frac{16}{100}\) using models.
Go Math Grade 4 Answer Key Chapter 9 Relate Fractions and Decimals img 18
Type below:
_________

Answer:
grade 4 chapter 9 Relate Fractions and Decimals Image 1 504

Question 15.
Describe Relationships Describe how one whole, one tenth, and one hundredth are related.
Type below:
_________

Answer:
One whole = 1.00
One tenth: 0.1
One hundredth: 0.01
One whole is 10 times the one-tenth, and one-tenth is 10 times the one hundredth.

Question 16.
Shade the model to show 1 \(\frac{24}{100}\). Then write the mixed number in decimal form.
Go Math Grade 4 Answer Key Chapter 9 Relate Fractions and Decimals img 19
_____

Answer:
grade 4 chapter 9 Relate Fractions and Decimals Image 2 504
1\(\frac{24}{100}\) = \(\frac{124}{100}\) = 1.24

Question 17.
The Memorial Library is 0.3 mile from school. Whose statement makes sense? Whose statement is nonsense? Explain your reasoning.
Go Math Grade 4 Answer Key Chapter 9 Relate Fractions and Decimals img 20
Type below:
_________

Answer:
The boy’s statement makes sense. Because The Memorial Library is 0.3 miles from the school. Digit 3 in the tenths place after the first place of decimal.
The girl’s statement makes non-sense. Because there she said 3 miles that is not equal to 0.3 miles.

Common Core – New – Page No. 505

Relate Hundredths and Decimals

Write the fraction or mixed number and the decimal shown by the model.

Question 1.
Go Math Grade 4 Answer Key Chapter 9 Relate Fractions and Decimals Common Core - New img 21

Answer:
Go Math Grade 4 Answer Key Chapter 9 Relate Fractions and Decimals Common Core - New img 21

Question 2.
Go Math Grade 4 Answer Key Chapter 9 Relate Fractions and Decimals Common Core - New img 22
Type below:
_________

Answer:
\(\frac{29}{100}\) = 0.29

Explanation:
0.20 names the same amount as 20/100. So, the given point is at 29/100 = 0.29

Question 3.
Go Math Grade 4 Answer Key Chapter 9 Relate Fractions and Decimals Common Core - New img 23
Type below:
_________

Answer:
1\(\frac{54}{100}\) = 1.54

Explanation:
From the given image, one model is one whole and another model 54 boxes shaded out of 100. So, the answer is 1\(\frac{54}{100}\) = 1.54

Question 4.
Go Math Grade 4 Answer Key Chapter 9 Relate Fractions and Decimals Common Core - New img 24
Type below:
_________

Answer:
4\(\frac{62}{100}\) = 4.62

Explanation:
4.60 names the same amount as 4\(\frac{60}{100}\). So, the given point is at 4\(\frac{62}{100}\) = 4.62

Write the fraction or mixed number as a decimal.

Question 5.
\(\frac{37}{100}\)
_____

Answer:
0.37

Explanation:
Write down 37 with the decimal point 2 spaces from the right (because 100 has 2 zeros). 0.37

Question 6.
8 \(\frac{11}{100}\)
_____

Answer:
8.11

Explanation:
8\(\frac{11}{100}\) = \(\frac{811}{100}\)
Write down 811 with the decimal point 2 spaces from the right (because 100 has 2 zeros). So, 8.11 is the answer.

Question 7.
\(\frac{98}{100}\)
_____

Answer:
0.98

Explanation:
Write down 98 with the decimal point 2 spaces from the right (because 100 has 2 zeros). 0.98

Question 8.
25 \(\frac{50}{100}\)
_____

Answer:
25.50

Explanation:
25\(\frac{50}{100}\) = \(\frac{2550}{100}\)
Write down 2550 with the decimal point 2 spaces from the right (because 100 has 2 zeros). So, 25.50 is the answer.

Question 9.
\(\frac{6}{100}\)
_____

Answer:
0.06

Explanation:
Write down 6 with the decimal point 2 spaces from the right (because 100 has 2 zeros). 0.06

Problem Solving

Question 10.
There are 100 pennies in a dollar. What fraction of a dollar is 61 pennies? Write it as a fraction, as a decimal, and in word form.
Type below:
_________

Answer:
\(\frac{61}{100}\) pennies = 0.61 = sixty-one hundredths

Explanation:
There are 100 pennies in a dollar. So, for 61 pennies, there are \(\frac{61}{100}\) pennies = 0.61 = sixty-one hundredths.

Question 11.
Kylee has collected 100 souvenir thimbles from different places she has visited with her family. Twenty of the thimbles are carved from wood. Write the fraction of thimbles that are wooden as a decimal.
_________

Answer:
It is easier to work with decimals then fractions because it is like adding whole numbers in a normal way.

Common Core – New – Page No. 506

Lesson Check

Question 1.
Which decimal represents the shaded section of the model below?
Go Math Grade 4 Answer Key Chapter 9 Relate Fractions and Decimals Common Core - New img 25
Options:
a. 830.0
b. 83.0
c. 8.30
d. 0.83

Answer:
d. 0.83

Explanation:
The model is divided into 100 equal parts. Each part represents one hundredth. 83 boxes are shaded out of 100. So, the answer is \(\frac{83}{100}\) = 0.83

Question 2.
There were 100 questions on the unit test. Alondra answered 97 of the questions correctly. What decimal represents the fraction of questions Alondra answered correctly?
Options:
a. 0.97
b. 9.70
c. 90.70
d. 970.0

Answer:
a. 0.97

Explanation:
There were 100 questions on the unit test. Alondra answered 97 of the questions correctly. So, \(\frac{97}{100}\) questions answered correctly. = 0.97

Spiral Review

Question 3.
Which is equivalent to \(\frac{7}{8}\)?
Options:
a. \(\frac{5}{8}+\frac{3}{8}\)
b. \(\frac{4}{8}+\frac{1}{8}+\frac{1}{8}\)
c. \(\frac{3}{8}+\frac{2}{8}+\frac{2}{8}\)
d. \(\frac{2}{8}+\frac{2}{8}+\frac{1}{8}+\frac{1}{8}\)

Answer:
c. \(\frac{3}{8}+\frac{2}{8}+\frac{2}{8}\)

Explanation:
c. \(\frac{3}{8}+\frac{2}{8}+\frac{2}{8}\) = \(\frac{7}{8}\)

Question 4.
What is \(\frac{9}{10}-\frac{6}{10}\)?
Go Math Grade 4 Answer Key Chapter 9 Relate Fractions and Decimals Common Core - New img 26
Options:
a. \(\frac{1}{10}\)
b. \(\frac{3}{10}\)
c. \(\frac{4}{10}\)
d. \(\frac{6}{10}\)

Answer:
b. \(\frac{3}{10}\)

Explanation:
\(\frac{9}{10}-\frac{6}{10}\). From 9 parts, 6 parts are removed. So, remaining parts are 3.

Question 5.
Misha used \(\frac{1}{4}\) of a carton of 12 eggs to make an omelet. How many eggs did she use?
Options:
a. 2
b. 3
c. 4
d. 6

Answer:
b. 3

Explanation:
Misha used \(\frac{1}{4}\) of a carton of 12 eggs to make an omelet. \(\frac{1}{4}\) x 12 = 3 eggs.

Question 6.
Kurt used the rule add 4, subtract 1 to generate a pattern. The first term in his pattern is 5. Which number could be in Kurt’s pattern?
Options:
a. 4
b. 6
c. 10
d. 14

Answer:
d. 14

Explanation:
Kurt used the rule add 4, subtract 1 to generate a pattern. The first term in his pattern is 5. The pattern numbers are 5, 8, 11, 14, 17, 20, etc. So, the answer is 14.

Page No. 509

Question 1.
Write \(\frac{4}{10}\) as hundredths.
Write \(\frac{4}{10}\) as an equivalent fraction.
\(\frac{4}{10}\) =\(\frac{4 × ■}{10× ■}\)
Write \(\frac{4}{10}\) as a decimal.
Go Math Grade 4 Answer Key Chapter 9 Relate Fractions and Decimals img 27
Type below:
_________

Answer:
\(\frac{40}{100}\)
grade 4 chapter 9 Relate Fractions and Decimals Image 1 509
0.40

Explanation:
Write \(\frac{4}{10}\) as an equivalent fraction.
\(\frac{4}{10}\) =\(\frac{4 × 10}{10× 10}\) = \(\frac{40}{100}\)
6 tenths is the same as 6 tenths 0 hundredths. So the decimal form = 0.40

Write the number as hundredths in fraction form and decimal form.

Question 2.
\(\frac{7}{10}\)
Type below:
_________

Answer:
\(\frac{70}{100}\)
grade 4 chapter 9 Relate Fractions and Decimals Image 2 509
0.70

Explanation:
Write \(\frac{7}{10}\) as an equivalent fraction.
\(\frac{7}{10}\) =\(\frac{7 × 10}{10× 10}\) = \(\frac{70}{100}\)
7 tenths is the same as 7 tenths 0 hundredths. So the decimal form = 0.70

Question 3.
0.5
Type below:
_________

Answer:
\(\frac{50}{100}\)
grade 4 chapter 9 Relate Fractions and Decimals Image 3 509
0.50

Explanation:
Write 0.5 = \(\frac{5}{10}\) as an equivalent fraction.
\(\frac{5}{10}\) =\(\frac{5 × 10}{10× 10}\) = \(\frac{50}{100}\)
5 tenths is the same as 5 tenths 0 hundredths and also 0.5

Question 4.
\(\frac{3}{10}\)
Type below:
_________

Answer:
\(\frac{30}{100}\)
grade 4 chapter 9 Relate Fractions and Decimals Image 4 509
0.30

Explanation:
Write \(\frac{3}{10}\) as an equivalent fraction.
\(\frac{3}{10}\) =\(\frac{3 × 10}{10× 10}\) = \(\frac{30}{100}\)
3 tenths is the same as 3 tenths 0 hundredths. So the decimal form = 0.30

Write the number as tenths in fraction form and decimal form.

Question 5.
0.40
Type below:
_________

Answer:
\(\frac{4}{10}\) = 0.4

Explanation:
grade 4 chapter 9 Relate Fractions and Decimals Image 1 509
There are no hundredths.
0.40 is equivalent to 4 tenths.
Write 0.40 as 4 tenths = 0.4 = \(\frac{4}{10}\)

Question 6.
\(\frac{80}{100}\)
Type below:
_________

Answer:
\(\frac{8}{10}\)
grade 4 chapter 9 Relate Fractions and Decimals Image 5 509
0.8

Explanation:
10 is a common factor of the numerator and the denominator.
\(\frac{80}{100}\) = \(\frac{80 ÷ 10}{100 ÷ 10}\) = \(\frac{8}{10}\)
0.8

Question 7.
\(\frac{20}{100}\)
Type below:
_________

Answer:
\(\frac{2}{10}\)
grade 4 chapter 9 Relate Fractions and Decimals Image 6 509
0.2

Explanation:
10 is a common factor of the numerator and the denominator.
\(\frac{20}{100}\) = \(\frac{20 ÷ 10}{100 ÷ 10}\) = \(\frac{2}{10}\)
0.2

Practice: Copy and Solve Write the number as hundredths in fraction form and decimal form.

Question 8.
\(\frac{8}{10}\)
Type below:
_________

Answer:
\(\frac{80}{100}\)
grade 4 chapter 9 Relate Fractions and Decimals Image 5 509
0.8

Explanation:
Write \(\frac{8}{10}\) as an equivalent fraction.
\(\frac{8}{10}\) =\(\frac{8 × 10}{10× 10}\) = \(\frac{80}{100}\)
8 tenths is the same as 8 tenths 0 hundredths. So the decimal form = 0.8

Question 9.
\(\frac{2}{10}\)
Type below:
_________

Answer:
\(\frac{20}{100}\)
grade 4 chapter 9 Relate Fractions and Decimals Image 6 509
0.2

Explanation:
Write \(\frac{2}{10}\) as an equivalent fraction.
\(\frac{2}{10}\) =\(\frac{2 × 10}{10× 10}\) = \(\frac{20}{100}\)
2 tenths is the same as 2 tenths 0 hundredths. So the decimal form = 0.2

Question 10.
0.1
Type below:
_________

Answer:
\(\frac{50}{100}\)
grade 4 chapter 9 Relate Fractions and Decimals Image 7 509
0.50

Explanation:
Write 0.1 = \(\frac{1}{10}\) as an equivalent fraction.
\(\frac{1}{10}\) =\(\frac{1 × 10}{10× 10}\) = \(\frac{10}{100}\)
1 tenth is the same as 1 tenth 0 hundredths and also 0.1

Practice: Copy and Solve Write the number as tenths in fraction form and decimal form.

Question 11.
\(\frac{60}{100}\)
Type below:
_________

Answer:
\(\frac{6}{10}\)
grade 4 chapter 9 Relate Fractions and Decimals Image 8 509
0.6

Explanation:
10 is a common factor of the numerator and the denominator.
\(\frac{60}{100}\) = \(\frac{60 ÷ 10}{100 ÷ 10}\) = \(\frac{6}{10}\)
0.6

Question 12.
\(\frac{90}{100}\)
Type below:
_________

Answer:
\(\frac{9}{10}\)
grade 4 chapter 9 Relate Fractions and Decimals Image 9 509
0.9

Explanation:
10 is a common factor of the numerator and the denominator.
\(\frac{90}{100}\) = \(\frac{90 ÷ 10}{100 ÷ 10}\) = \(\frac{9}{10}\)
= 0.9

Question 13.
0.70
Type below:
_________

Answer:
\(\frac{7}{10}\)
grade 4 chapter 9 Relate Fractions and Decimals Image 2 509
0.7

Explanation:
grade 4 chapter 9 Relate Fractions and Decimals Image 2 509
There are no hundredths.
0.70 is equivalent to 7 tenths.
Write 0.70 as 7 tenths = 0.7 = \(\frac{7}{10}\)

Write the number as an equivalent mixed number with hundredths.

Question 14.
1 \(\frac{4}{10}\) = _____

Answer:
1 \(\frac{40}{100}\)

Explanation:
1 \(\frac{4 x 10}{10 x 10}\) = 1 \(\frac{40}{100}\)

Question 15.
3 \(\frac{5}{10}\) = _____

Answer:
3 \(\frac{50}{100}\)

Explanation:
3 \(\frac{5}{10}\) = 3 \(\frac{5 x 10}{10 x 10}\) = 3 \(\frac{50}{100}\)

Question 16.
2 \(\frac{9}{10}\) = _____

Answer:
2 \(\frac{90}{100}\)

Explanation:
2 \(\frac{9}{10}\) = 2 \(\frac{9 x 10}{10 x 10}\) = 2 \(\frac{90}{100}\)

Page No. 510

Question 17.
Carter says that 0.08 is equivalent to \(\frac{8}{10}\). Describe and correct Carter’s error.
Type below:
_________

Answer:
grade 4 chapter 9 Relate Fractions and Decimals Image 1 510
8 hundredths = \(\frac{8}{100}\)
The decimal point is before the 2 numbers. So, the denominator should be 100.

Question 18.
For numbers 18a–18e, choose True or False for the statement.
a. 0.6 is equivalent to \(\frac{6}{100}\).
i. True
ii. False

Answer:
ii. False

Explanation:
grade 4 chapter 9 Relate Fractions and Decimals Image 8 509
0.60 = 6 tenths.
6 tenths = \(\frac{6}{10}\)

Question 18.
b. \(\frac{3}{10}\) is equivalent to 0.30.
i. True
ii. False

Answer:
i. True

Explanation:
grade 4 chapter 9 Relate Fractions and Decimals Image 4 509
0.30 = 3 tenths.
3 tenths = \(\frac{3}{10}\)

Question 18.
c. \(\frac{40}{100}\) is equivalent to \(\frac{4}{10}\).
i. True
ii. False

Answer:
i. True

Explanation:
10 is a common factor of the numerator and the denominator.
\(\frac{40}{100}\) = \(\frac{40 ÷ 10}{100 ÷ 10}\) = \(\frac{4}{10}\)

Question 18.
d. 0.40 is equivalent to \(\frac{4}{100}\).
i. True
ii. False

Answer:
ii. False

Explanation:
grade 4 chapter 9 Relate Fractions and Decimals Image 1 509
4 tenths and 0 hundreds = \(\frac{4}{10}\)

Question 18.
e. 0.5 is equivalent to 0.50.
i. True
ii. False

Answer:
i. True

Explanation:
If you add any zeros after the 5 it will be equal to 0.5. So, 0.5 is equivalent to 0.50

Inland Water
How many lakes and rivers does your state have? The U.S. Geological Survey defines inland water as water that is surrounded by land. The Atlantic Ocean, the Pacific Ocean, and the Great Lakes are not considered inland water.
Go Math Grade 4 Answer Key Chapter 9 Relate Fractions and Decimals img 28

Question 19.
Just over \(\frac{2}{100}\) of the entire United States is inland water. Write \(\frac{2}{100}\) as a decimal.
_____

Answer:
0.02

Explanation:
Write down 2 with the decimal point 2 spaces from the right (because 100 has 2 zeros). So, \(\frac{2}{100}\) = 0.02 is the answer

Question 20.
Can you write 0.02 as tenths? Explain.
_____ tenth

Answer:
0.2 tenth

Explanation:
0.02 = \(\frac{2}{100}\) = \(\frac{2 ÷ 10}{100 ÷ 10}\) = \(\frac{0.2}{10}\)

Question 21.
About 0.17 of the area of Rhode Island is inland water. Write 0.17 as a fraction.
\(\frac{□}{□}\)

Answer:
\(\frac{17}{100}\)

Explanation:
grade 4 chapter 9 Relate Fractions and Decimals Image 2 510
1 tenth and 7 hundred.
So, write 0.17 as \(\frac{17}{100}\)

Question 22.
Louisiana’s lakes and rivers cover about \(\frac{1}{10}\) of the state. Write \(\frac{1}{10}\) as hundredths in words, fraction form, and decimal form.
Type below:
_________

Answer:
Ten hundredths = \(\frac{10}{100}\) = 0.10

Explanation:
1 tenth is the same as the 1 tenth and 0 hundred
grade 4 chapter 9 Relate Fractions and Decimals Image 7 509
0.1 = 0.10 = \(\frac{10}{100}\)

Common Core – New – Page No. 511

Equivalent Fractions and Decimals

Write the number as hundredths in fraction form and decimal form.

Question 1.
\(\frac{5}{10}\) \(\frac{5}{10}\) = \(\frac{5 \times 10}{10 \times 10}\) = \(\frac{50}{100}\)
Go Math Grade 4 Answer Key Chapter 9 Relate Fractions and Decimals Common Core - New img 29
Think: 5 tenths is the same as 5 tenths and 0 hundredths. Write 0.50.

Question 2.
\(\frac{9}{10}\)
Type below:
_________

Answer:
\(\frac{90}{100}\); 0.90

Explanation:
\(\frac{9}{10}\) = \(\frac{9 \times 10}{10 \times 10}\) = \(\frac{90}{100}\)
9 tenths is the same as 9 tenths and 0 hundredths. Write 0.90

Question 3.
0.2
Type below:
_________

Answer:
\(\frac{20}{100}\)
0.20

Explanation:
2 tenths is the same as 2 tenths and 0 hundredths. Write 0.20.
grade 4 chapter 9 Relate Fractions and Decimals Image 6 509
\(\frac{2}{10}\) = \(\frac{2 x 10}{10 x 10}\) = \(\frac{20}{100}\)

Question 4.
0.8
Type below:
_________

Answer:
\(\frac{80}{100}\) = 0.80

Explanation:
8 tenths is the same as 8 tenths and 0 hundredths. Write 0.80.
grade 4 chapter 9 Relate Fractions and Decimals Image 5 509
\(\frac{8}{10}\) = \(\frac{8 x 10}{10 x 10}\) = \(\frac{80}{100}\)

Write the number as tenths in fraction form and decimal form.

Question 5.
\(\frac{40}{100}\)
Type below:
_________

Answer:
\(\frac{4}{10}\) = 0.4

Explanation:
10 is a common factor of the numerator and the denominator.
\(\frac{40}{100}\) = \(\frac{40 ÷ 10}{100 ÷ 10}\) = \(\frac{4}{10}\)
= 0.4

Question 6.
\(\frac{10}{100}\)
Type below:
_________

Answer:
\(\frac{1}{10}\) = 0.1

Explanation:
10 is a common factor of the numerator and the denominator.
\(\frac{10}{100}\) = \(\frac{10 ÷ 10}{100 ÷ 10}\) = \(\frac{1}{10}\)
= 0.1

Question 7.
0.60
Type below:
_________

Answer:
\(\frac{6}{10}\) = 0.6

Explanation:
0.60 is 60 hundredths.
\(\frac{60}{100}\).
10 is a common factor of the numerator and the denominator.
\(\frac{60}{100}\) = \(\frac{60 ÷ 10}{100 ÷ 10}\) = \(\frac{6}{10}\)
= 0.6

Problem Solving

Question 8.
Billy walks \(\frac{6}{10}\) mile to school each day. Write \(\frac{6}{10}\) as hundredths in fraction form and in decimal form.
Type below:
________

Answer:
\(\frac{60}{100}\)
0.60

Explanation:
Billy walks \(\frac{6}{10}\) mile to school each day.
\(\frac{6}{10}\) = \(\frac{6 x 10}{10 x 10}\) = \(\frac{60}{100}\)
grade 4 chapter 9 Relate Fractions and Decimals Image 8 509
0.60

Question 9.
Four states have names that begin with the letter A. This represents 0.08 of all the states. Write 0.08 as a fraction.
\(\frac{□}{□}\)

Answer:
\(\frac{8}{100}\)

Explanation:
0.08 is 8 hundredths. So, the fraction is \(\frac{8}{100}\)

Common Core – New – Page No. 512

Lesson Check

Question 1.
The fourth-grade students at Harvest School make up 0.3 of all students at the school. Which fraction is equivalent to 0.3?
Options:
a. \(\frac{3}{10}\)
b. \(\frac{30}{10}\)
c. \(\frac{3}{100}\)
d. \(\frac{33}{100}\)

Answer:
a. \(\frac{3}{10}\)

Explanation:
0.3 is same as the 3 tenths. So, the answer is \(\frac{3}{10}\)

Question 2.
Kyle and his brother have a marble set. Of the marbles, 12 are blue. This represents \(\frac{50}{100}\) of all the marbles. Which decimal is equivalent to \(\frac{50}{100}\)?
Options:
a. 50
b. 5.0
c. 0.50
d. 5,000

Answer:
c. 0.50

Explanation:

Write down 50 with the decimal point 2 spaces from the right (because 100 has 2 zeros). So, 0.50 is the answer

Spiral Review

Question 3.
Jesse won his race by 3 \(\frac{45}{100}\) seconds. What is this number written as a decimal?
Options:
a. 0.345
b. 3.45
c. 34.5
d. 345

Answer:
b. 3.45

Explanation:
3 \(\frac{45}{100}\) = \(\frac{345}{100}\). Write down 345 with the decimal point 2 spaces from the right (because 100 has 2 zeros). So, 3.45 is the answer

Question 4.
Marge cut 16 pieces of tape for mounting pictures on poster board. Each piece of tape was \(\frac{3}{8}\) inch long. How much tape did Marge use?
Options:
a. 2 inches
b. 4 inches
c. 5 inches
d. 6 inches

Answer:
d. 6 inches

Explanation:
\(\frac{3}{8}\) x 16 = 6 inches

Question 5.
Of Katie’s pattern blocks, \(\frac{9}{12}\) are triangles. What is \(\frac{9}{12}\) in simplest form?
Options:
a. \(\frac{1}{4}\)
b. \(\frac{2}{3}\)
c. \(\frac{3}{4}\)
d. \(\frac{9}{12}\)

Answer:
c. \(\frac{3}{4}\)

Explanation:
\(\frac{9}{12}\) is divided by 3. So, \(\frac{3}{4}\) is the answer.

Question 6.
A number pattern has 75 as its first term. The rule for the pattern is subtract 6. What is the sixth term?
Options:
a. 39
b. 45
c. 51
d. 69

Answer:
b. 45

Explanation:
75 is the first term.
75 – 6 =69
69 – 6 = 63
63 – 6 = 57
57 – 6 = 51
51 – 6 = 45.
The sixth term is 45.

Page No. 515

Question 1.
Write the amount of money as a decimal in terms of dollars.
Go Math Grade 4 Answer Key Chapter 9 Relate Fractions and Decimals img 30
5 pennies = \(\frac{5}{100}\) of a dollar = _____ of a dollar.
_____ of a dollar

Answer:
5 pennies = \(\frac{5}{100}\) of a dollar = 0.05 of a dollar.
0.05 of a dollar

Explanation:
Write down 5 with the decimal point 2 spaces from the right (because 100 has 2 zeros). 0.05

Write the total money amount. Then write the amount as a fraction or a mixed number and as a decimal in terms of dollars.

Question 2.
Go Math Grade 4 Answer Key Chapter 9 Relate Fractions and Decimals img 31
Type below:
_________

Answer:
\(\frac{109}{100}\) = 1.09

Explanation:
1 dollar = 1/10 dimes
1 dollar = 1/100 pennies
1 dollar = 25/100 quarters
(3 x 1/10) + (4 x 1/100) + (3 x 25/100)
3/10 + 4/100 + 75/100
30/100 + 4/100 + 75/100 = 109/100 = 1.09

Question 3.
Go Math Grade 4 Answer Key Chapter 9 Relate Fractions and Decimals img 32
Type below:
_________

Answer:
\(\frac{60}{100}\) = 0.60

Explanation:
Given that 1 quarter, 2 dimes, and 3 cents.
10 dimes = 1 dollars
100 pennies = 1 dollar
4 quarters = 1 dollar
2 cents = 1 dollar
(25/100) + (2 x 1/10) + (3 x 5/100) = 25/100 + 20/100 + 15/100 = 60/100 = 0.60

Write as a money amount and as a decimal in terms of dollars.

Question 4.
\(\frac{92}{100}\)
amount: _____ decimal: _____of a dollar

Answer:
amount: $0.92 decimal: 0.92 of a dollar

Explanation:
\(\frac{92}{100}\) = 0.92

Question 5.
\(\frac{7}{100}\)
money amount: $ _____ decimal: _____ of a dollar

Answer:
money amount: $0.07 decimal: 0.07 of a dollar

Explanation:
\(\frac{7}{100}\) = 0.07

Question 6.
\(\frac{16}{100}\)
money amount: $ _____ decimal: _____ of a dollar

Answer:
money amount: $0.16 decimal: 0.16 of a dollar

Explanation:
\(\frac{16}{100}\) = 0.16

Question 7.
\(\frac{53}{100}\)
money amount: $ _____ decimal: _____ of a dollar

Answer:
money amount: $0.53 decimal: 0.53 of a dollar

Explanation:
\(\frac{53}{100}\) = 0.53

Write the total money amount. Then write the amount as a fraction or a mixed number and as a decimal in terms of dollars.

Question 8.
Go Math Grade 4 Answer Key Chapter 9 Relate Fractions and Decimals img 33
Type below:
_________

Answer:
\(\frac{46}{100}\) = 0.46

Explanation:
Given that 3 dimes, 3 nickels, 1 pennies
(3 x 10/100) + (3 x 5/100) + 1/100 = 30/100 + 15/100 + 1/100 = 46/100 = 0.46

Question 9.
Go Math Grade 4 Answer Key Chapter 9 Relate Fractions and Decimals img 34
Type below:
_________

Answer:
\(\frac{136}{100}\) = 1.36

Explanation:
Given that 1 dollar, 1 quarter, 1 pennies, 2 nickels
1 + 25/100 + 1/100 + (2 x 5/100)
1 + 25/100 + 1/100 + 10/100
1 + 36/100
136/100 = 1.36

Write as a money amount and as a decimal in terms of dollars.

Question 10.
\(\frac{27}{100}\)
money amount: $ _____ decimal: _____ of a dollar

Answer:
amount: $0.27 decimal: 0.27 of a dollar

Explanation:
\(\frac{27}{100}\) = 0.27

Question 11.
\(\frac{4}{100}\)
money amount: $ _____ decimal: _____ of a dollar

Answer:
amount: $0.04 decimal: 0.04 of a dollar

Explanation:
\(\frac{4}{100}\) = 0.04

Question 12.
\(\frac{75}{100}\)
money amount: $ _____ decimal: _____ of a dollar

Answer:
amount: $0.75 decimal: 0.75 of a dollar

Explanation:
\(\frac{75}{100}\) = 0.75

Question 13.
\(\frac{100}{100}\)
money amount: $ _____ decimal:_____ of a dollar

Answer:
money amount: $1 decimal: 1 of a dollar

Explanation:
\(\frac{100}{100}\) = 1

Write the total money amount. Then write the amount as a fraction and as a decimal in terms of dollars.

Question 14.
1 quarter 6 dimes 8 pennies
Type below:
_________

Answer:
money amount: $0.39; fraction: \(\frac{39}{100}\) decimal: 0.39 of a dollar

Explanation:
1 dollar = 100 cents
1 quarter = 25 cents
1 dime = 10 cents
1 penny = 1 cent
1 quarter 6 dimes 8 pennies = (25/100) + (6 x 10/100) + (8 x 1/100)
25/100 + 60/100 + 8/100 = 39/100 = 0.39

Question 15.
3 dimes 5 nickels 20 pennies
Type below:
_________

Answer:
money amount: $0.75; fraction: \(\frac{75}{100}\) decimal: 0.75 of a dollar

Explanation:
1 dollar = 100 cents
1 quarter = 25 cents
1 dime = 10 cents
1 penny = 1 cent
3 dimes 5 nickels 20 pennies = (3 x 10/100) + (5 x 5/100) + (20 x 1/100)
30/100 + 25/100 + 20/100 = 75/100 = 0.75

Page No. 516

Make Connections Algebra Complete to tell the value of each digit.

Question 16.
a.
$1.05 = _____ dollar + _____ pennies;

Answer:
$1.05 = 1 dollar + 5 pennies

Explanation:
grade 4 chapter 9 Relate Fractions and Decimals Image 1 516
$1.05 = 1 dollar and 05 pennies
There are 100 pennies in 1 dollar.
So, $1.05 = 105 pennies.

Question 16.
b.
1.05 = _____ one + _____ hundredths

Answer:
1.05 = 1 one and 05 hundredths

Explanation:
grade 4 chapter 9 Relate Fractions and Decimals Image 2 516
1.05 = 1 one and 05 hundredths
There are 100 hundredths in 1 one.
So, 1.05 = 105 hundredths.

Question 17.
a.
$5.18 = _____ dollars + _____ dime + _____ pennies;

Answer:
$5.18 = 5 dollars + 1 dime + 8 pennies;

Explanation:
grade 4 chapter 9 Relate Fractions and Decimals Image 3 516
$5.18 = 5 dollar and 1 dime and 8 pennies
There are 500 pennies in 5 dollars.
1 dime = 10 pennies
So, $5.18 = 518 pennies.

Question 17.
b.
5.18 = _____ ones + _____ tenth + _____ pennies

Answer:
5.18 = 5 ones + 1 tenths + 8 pennies

Explanation:
grade 4 chapter 9 Relate Fractions and Decimals Image 4 516
5.18 = 5 ones and 1 tenths and 8 pennies
There are 100 hundredths in 1 one. So, 500 hundredths in 5 ones.
So, 5.18 = 518 hundredths.

Use the table for 18–19.
Go Math Grade 4 Answer Key Chapter 9 Relate Fractions and Decimals img 35

Question 18.
The table shows the coins three students have. Write Nick’s total amount as a fraction in terms of dollars.
\(\frac{□}{□}\) of a dollar

Answer:
\(\frac{92}{100}\) of a dollar

Explanation:
Nick’s total amount = 2 quarters + 4 dimes + 0 Nickels + 2 pennies
= (2 x 25/100) + (4 x 10/100) + (2 x 1/100) = 50/100 + 40/100 + 2/100 = 92/100

Question 19.
Kim spent \(\frac{40}{100}\) of a dollar on a snack. Write as a money amount the amount she has left.
$ _____

Answer:
$0.28

Explanation:
Kim’s total amount = 1 quarter + 3 dimes + 2 nickels + 3 pennies
= 25/100 + (3 x 10/100) + (2 x 5/100) + (3 x 1/100) = 25/100 + 30/100 + 10/100 + 3/100 = 68/100.
Kim spent \(\frac{40}{100}\) of a dollar on a snack. So, 68/100 – 40/100 = 28/100 = 0.28

Question 20.
Travis has \(\frac{1}{2}\) of a dollar. He has at least two different types of coins in his pocket. Draw two possible sets of coins that Travis could have.
Type below:
_________

Answer:
grade 4 chapter 9 Relate Fractions and Decimals Image 6 516

Explanation:
1 Quarter + 2 dimes + 5 Pennies = 25/100 + 10/100 + 10/100 + 5/100 = 50/100 = 1/2 of a dollar
1 Quarter + 2 dimes + 1 Nickel = 25/100 + 10/100 + 10/100 + 5/100 = 50/100 = 1/2 of a dollar

Question 21.
Complete the table.
Go Math Grade 4 Answer Key Chapter 9 Relate Fractions and Decimals img 36
Type below:
_________

Answer:
grade 4 chapter 9 Relate Fractions and Decimals Image 7 516

Common Core – New – Page No. 517

Relate Fractions, Decimals, and Money

Write the total money amount. Then write the amount as a fraction or a mixed number and as a decimal in terms of dollars.

Question 1.
Go Math Grade 4 Answer Key Chapter 9 Relate Fractions and Decimals Common Core - New img 37

Answer:
$0.18 = \(\frac{18}{100}\) = 0.18

Explanation:
Given that 3 Pennies + 3 Nickels = 3/100 + 15/100 = 18/100

Question 2.
Go Math Grade 4 Answer Key Chapter 9 Relate Fractions and Decimals Common Core - New img 38
Type below:
_________

Answer:
$0.56 = \(\frac{56}{100}\) = 0.56

Explanation:
Given that 1 Quarter + 3 dime + 1 Pennies = 25/100 + 30/100 + 1/100 = 56/100

Write as a money amount and as a decimal in terms of dollars.

Question 3.
\(\frac{25}{100}\)
Dollars: _____ Decimal: _____

Answer:
Dollars: 1 quarter = $0.25; Decimal: 0.25

Explanation:
25 our of 100 dollars = 1 quarter.
So, 25/100 = 0.25

Question 4.
\(\frac{79}{100}\)
Dollars: _____ Decimal: _____

Answer:
amount: $0.79 decimal: 0.79 of a dollar

Explanation:
\(\frac{79}{100}\) = 0.79

Question 5.
\(\frac{31}{100}\)
Dollars: _____ Decimal: _____

Answer:
amount: $0.31 decimal: 0.31 of a dollar

Explanation:
\(\frac{31}{100}\) = 0.31

Question 6.
\(\frac{8}{100}\)
Dollars: _____ Decimal: _____

Answer:
amount: $0.08 decimal: 0.08 of a dollar

Explanation:
\(\frac{8}{100}\) = 0.08

Question 7.
\(\frac{42}{100}\)
Dollars: _____ Decimal: _____

Answer:
amount: $0.42 decimal: 0.42 of a dollar

Explanation:
\(\frac{42}{100}\) = 0.42

Write the money amount as a fraction in terms of dollars.

Question 8.
$0.87
\(\frac{□}{□}\)

Answer:
\(\frac{87}{100}\) of a dollar

Explanation:
grade 4 chapter 9 Relate Fractions and Decimals Image 1 517
$0.87 = 87 pennies
There are 100 pennies in 1 dollar.
So, $0.87 = \(\frac{87}{100}\) of a dollar.

Question 9.
$0.03
\(\frac{□}{□}\)

Answer:
\(\frac{3}{100}\)

Explanation:
grade 4 chapter 9 Relate Fractions and Decimals Image 2 517
$0.03 = 3 pennies
There are 100 pennies in 1 dollar.
So, $0.03 = \(\frac{3}{100}\).

Question 10.
$0.66
\(\frac{□}{□}\)

Answer:
\(\frac{66}{100}\)

Explanation:
grade 4 chapter 9 Relate Fractions and Decimals Image 3 517
$0.66 = 66 pennies
There are 100 pennies in 1 dollar.
So, $0.66 = \(\frac{66}{100}\).

Question 11.
$0.95
\(\frac{□}{□}\)

Answer:
\(\frac{95}{100}\)

Explanation:
grade 4 chapter 9 Relate Fractions and Decimals Image 4 517
$0.95 = 95 pennies
There are 100 pennies in 1 dollar.
So, $0.95 = \(\frac{95}{100}\).

Question 12.
$1.00
\(\frac{□}{□}\)

Answer:
\(\frac{100}{100}\)

Explanation:
grade 4 chapter 9 Relate Fractions and Decimals Image 5 517
$1.00 = 1 dollar
There are 100 pennies in 1 dollar.
So, $1.00 = \(\frac{100}{100}\).

Write the total money amount. Then write the amount as a fraction and as a decimal in terms of dollars.

Question 13.
2 quarters 2 dimes
Type below:
_________

Answer:
money amount: $0.70; fraction: \(\frac{70}{100}\); decimal: 0.70

Explanation:
Given that 2 quarters 2 dimes = (2 x 25/100) + (2 x 10/100) = 50/100 + 20/100 = 70/100

Question 14.
3 dimes 4 pennies
Type below:
_________

Answer:
money amount: $0.34; fraction: \(\frac{34}{100}\); decimal: 0.34

Explanation:
Given that 3 dimes 4 pennies = (3 x 10/100) + (4 x 1/100) = 30/100 + 4/100 = 34/100

Question 15.
8 nickels 12 pennies
Type below:
_________

Answer:
money amount: $0.57; fraction: \(\frac{57}{100}\); decimal: 0.57

Explanation:
Given that 8 nickels 12 pennies = (8 x 5/100) + (12 x 1/100) = 45/100 + 12/100 = 57/100

Problem Solving

Question 16.
Kate has 1 dime, 4 nickels, and 8 pennies. Write Kate’s total amount as a fraction in terms of a dollar.
\(\frac{□}{□}\)

Answer:
fraction: \(\frac{38}{100}\)

Explanation:
Kate has 1 dime, 4 nickels, and 8 pennies.
10/100 + (4 x 5/100) + (8/100) = 10/100 + 20/100 + 8/100 = 38/100

Question 17.
Nolan says he has \(\frac{75}{100}\) of a dollar. If he only has 3 coins, what are the coins?
_________

Answer:
3 quarters

Explanation:
3 quarters = \(\frac{25}{100}\) + \(\frac{25}{100}\) + \(\frac{25}{100}\) = \(\frac{75}{100}\)

Common Core – New – Page No. 518

Lesson Check

Question 1.
Which of the following names the total money amount shown as a fraction in terms of a dollar?
Go Math Grade 4 Answer Key Chapter 9 Relate Fractions and Decimals Common Core - New img 39
Options:
a. \(\frac{43}{1}\)
b. \(\frac{43}{10}\)
c. \(\frac{43}{57}\)
d. \(\frac{43}{100}\)

Answer:
d. \(\frac{43}{100}\)

Explanation:
Given that 1 quarter + 1 nickel + 1 dime + 3 pennies = 25/100 + 5/100 + 10/100 + 3/100 = 43/100

Question 2.
Crystal has \(\frac{81}{100}\) of a dollar. Which of the following could be the coins Crystal has?
Options:
a. 3 quarters, 1 dime, 1 penny
b. 2 quarters, 6 nickels, 1 penny
c. 2 quarters, 21 pennies
d. 1 quarter, 4 dimes, 1 nickel, 1 penny

Answer:
b. 2 quarters, 6 nickels, 1 penny

Explanation:
2 quarters, 6 nickels, 1 penny = (2 x 25/100) + (6 x 5/100) + 1/100 = 50/100 + 30/100 + 1/100 = 81/100

Spiral Review

Question 3.
Joel gives \(\frac{1}{3}\) of his baseball cards to his sister. Which fraction is equivalent to \(\frac{1}{3}\)?
Options:
a. \(\frac{3}{5}\)
b. \(\frac{2}{6}\)
c. \(\frac{8}{9}\)
d. \(\frac{4}{10}\)

Answer:
b. \(\frac{2}{6}\)

Explanation:
\(\frac{2}{6}\) is divided by 2. The remaining answer after the dividion is \(\frac{1}{3}\).

Question 4.
Penelope bakes pretzels. She salts \(\frac{3}{8}\) of the pretzels. Which fraction is equivalent to \(\frac{3}{8}\)?
Options:
a. \(\frac{9}{24}\)
b. \(\frac{15}{20}\)
c. \(\frac{3}{16}\)
d. \(\frac{1}{5}\)

Answer:
a. \(\frac{9}{24}\)

Explanation:
a. \(\frac{9}{24}\) is divided by 3. The remaining fraction after the division is \(\frac{3}{8}\).

Question 5.
Which decimal is shown by the model?
Go Math Grade 4 Answer Key Chapter 9 Relate Fractions and Decimals Common Core - New img 40
Options:
a. 10.0
b. 1.0
c. 0.1
d. 0.01

Answer:
d. 0.01

Explanation:
1 box is shaded out of 100. So, the fraction is 1/100 = 0.01.

Question 6.
Mr. Guzman has 100 cows on his dairy farm. Of the cows, 57 are Holstein. What decimal represents the portion of cows that are Holstein?
Options:
a. 0.43
b. 0.57
c. 5.7
d. 57.0

Answer:
b. 0.57

Explanation:
Mr. Guzman has 100 cows on his dairy farm. Of the cows, 57 are Holstein. So, 57/100 Holstein cows are available.
57/100 = 0.57

Page No. 521

Question 1.
Juan has $3.43. He is buying a paint brush that costs $1.21 to paint a model race car. How much will Juan have after he pays for the paint brush?
First, use bills and coins to model $3.43.
Go Math Grade 4 Answer Key Chapter 9 Relate Fractions and Decimals img 41
Next, you need to subtract. Remove bills and coins that have a value of $1.21. Mark Xs to show what you remove.
Last, count the value of the bills and coins that are left. How much will Juan have left?
$ _____

Answer:
Juan has $3.43. He is buying a paint brush that costs $1.21 to paint a model race car. Subtract $3.43 – $1.21
grade 4 chapter 9 Relate Fractions and Decimals Image 1 521
2 dollars, 2 dimes, and 2 pennies left.
2 + (2 x 10/100) + (2/100) = 2 + 20/100 + 2/100 = 2 + 22/100 = 2.22.
Juan has left $2.22

Question 2.
What if Juan has $3.43, and he wants to buy a paint brush that costs $2.28? How much money will Juan have left then? Explain.
$ _____

Answer:
$1.15

Explanation:
Juan has $3.43. He wants to buy a paint brush that costs $2.28.
$3.43 – $2.28 = $1.15

Question 3.
Sophia has $2.25. She wants to give an equal amount to each of her 3 young cousins. How much will each cousin receive?
$ _____ each cousin receive

Answer:
$0.75 each cousin receive

Explanation:
Sophia has $2.25. She wants to give an equal amount to each of her 3 young cousins.
Divide $2.25 with 3 = $2.25/3 = $0.75

Page No. 522

Question 4.
Marcus saves $13 each week. In how many weeks will he have saved at least $100?
_____ weeks

Answer:
8 weeks

Explanation:
Marcus saves $13 each week. He saves $100 in $100/$13 weeks = 7.96 weeks that is nearly equal to 8 weeks.

Question 5.
Analyze Relationships Hoshi has $50. Emily has $23 more than Hoshi. Karl has $16 less than Emily. How much money do they have all together?
$ _____

Answer:
$180

Explanation:
Hoshi has $50.
Emily has $23 more than Hoshi = $50 + $23 = $73.
Karl has $16 less than Emily = $73 – $16 = $57.
All together = $50 +$73 + $57 = $180.

Question 6.
Four girls have $5.00 to share equally. How much money will each girl get? Explain.
$ _____ each girl

Answer:
$1.25 for each girl

Explanation:
Four girls have $5.00 to share equally. So, each girl get $5.00/4 = $1.25

Question 7.
What if four girls want to share $5.52 equally? How much money will each girl get? Explain.
$ _____

Answer:
$1.38

Explanation:
Four girls have $5.52 to share equally. So, each girl get $5.52/4 = $1.38. If the amount shares equally, each girl get 1 dollar, 1 dime, 8 pennies.

Question 8.
Aimee and three of her friends have three quarters and one nickel. If Aimee and her friends share the money equally, how much will each person get? Explain how you found your answer.
$ _____

Answer:
$0.2

Explanation:
Aimee and three of her friends have three quarters and one nickel. If Aimee and her friends share the money equally. Four members shared (3 x 25/100) + 5/100 = 75/100 + 5/100 = 80/100 = 0.8.
Four members shared $0.8 equally, $0.8/4 = $0.2.

Common Core – New – Page No. 523

Problem Solving Money

Use the act it out strategy to solve.

Question 1.
Carl wants to buy a bicycle bell that costs $4.50. Carl has saved $2.75 so far. How much more money does he need to buy the bell?
Use 4 $1 bills and 2 quarters to model $4.50. Remove bills and coins that have a value of $2.75. First, remove 2 $1 bills and 2 quarters.
Next, exchange one $1 bill for 4 quarters and remove 1 quarter.
Count the amount that is left. So, Carl needs to save $1.75 more.
Go Math Grade 4 Answer Key Chapter 9 Relate Fractions and Decimals Common Core - New img 42

Answer:
Go Math Grade 4 Answer Key Chapter 9 Relate Fractions and Decimals Common Core - New img 42

Question 2.
Together, Xavier, Yolanda, and Zachary have $4.44. If each person has the same amount, how much money does each person have?
$ __________

Answer:
$1.11

Explanation:
Together, Xavier, Yolanda, and Zachary have $4.44. If each person has the same amount, $4.44/4 = $1.11

Question 3.
Marcus, Nan, and Olive each have $1.65 in their pockets. They decide to combine the money. How much money do they have altogether?
$ __________

Answer:
$4.95

Explanation:
Marcus, Nan, and Olive each have $1.65 in their pockets. They decide to combine the money. So, $1.65 + $1.65 + $1.65 = $4.95

Question 4.
Jessie saves $6 each week. In how many weeks will she have saved at least $50?
__________ weeks

Answer:
9 weeks

Explanation:
Jessie saves $6 each week. To save $50, $50/$6 = 9 weeks (approximately)

Question 5.
Becca has $12 more than Cece. Dave has $3 less than Cece. Cece has $10. How much money do they have altogether?
$ __________

Answer:
$39

Explanation:
Cece has $10.
Becca has $12 more than Cece = $10 + $12 = $22.
Dave has $3 less than Cece = $10 – $3 = $7.
All together = $10 + $22 + $7 = $39.

Common Core – New – Page No. 524

Lesson Check

Question 1.
Four friends earned $5.20 for washing a car. They shared the money equally. How much did each friend get?
Options:
a. $1.05
b. $1.30
c. $1.60
d. $20.80

Answer:
b. $1.30

Explanation:
Four friends earned $5.20 for washing a car. They shared the money equally.
$5.20/4 = $1.30

Question 2.
Which represents the value of one $1 bill and 5 quarters?
Options:
a. $1.05
b. $1.25
c. $1.50
d. $2.25

Answer:
d. $2.25

Explanation:
one $1 bill and 5 quarters. 5 quarters = 5 x 0.25 = 1.25.
$1 + $1.25 = $2.25

Spiral Review

Question 3.
Bethany has 9 pennies. What fraction of a dollar is this?
Options:
a. \(\frac{9}{100}\)
b. \(\frac{9}{10}\)
c. \(\frac{90}{100}\)
d. \(\frac{99}{100}\)

Answer:
a. \(\frac{9}{100}\)

Explanation:
1 dollar = 100 pennies.
So, 9 pennies = 9/100 of a dollar

Question 4.
Michael made \(\frac{9}{12}\) of his free throws at practice. What is \(\frac{9}{12}\) in simplest form?
Options:
a. \(\frac{1}{4}\)
b. \(\frac{3}{9}\)
c. \(\frac{1}{2}\)
d. \(\frac{3}{4}\)

Answer:
d. \(\frac{3}{4}\)

Explanation:
\(\frac{9}{12}\) is divided by 3 that is equal to d. \(\frac{3}{4}\).

Question 5.
I am a prime number between 30 and 40. Which number could I be?
Options:
a. 31
b. 33
c. 36
d. 39

Answer:
a. 31

Explanation:
31 has fractions 1 and 31.

Question 6.
Georgette is using the benchmark \(\frac{1}{2}\) to compare fractions. Which statement is correct?
Options:
a. \(\frac{3}{8}>\frac{1}{2}\)
b. \(\frac{2}{5}<\frac{1}{2}\)
c. \(\frac{7}{12}<\frac{1}{2}\)
d. \(\frac{9}{10}=\frac{1}{2}\)

Answer:
b. \(\frac{2}{5}<\frac{1}{2}\)

Explanation:
From the given details, \(\frac{2}{5}<\frac{1}{2}\) is the correct answer.

Page No. 525

Choose the best term from the box to complete the sentence.
Go Math Grade 4 Answer Key Chapter 9 Relate Fractions and Decimals img 43

Question 1.
A symbol used to separate the ones and the tenths place is called a __________.
__________

Answer:
decimal point

Question 2.
The number 0.4 is written as a ____________.
__________

Answer:
4 tenths or 40 hundredths

Question 3.
A ______________ is one of one hundred equal parts of a whole.
__________

Answer:
hundredth

Write the fraction or mixed number and the decimal shown by the model.

Question 4.
Go Math Grade 4 Answer Key Chapter 9 Relate Fractions and Decimals img 44
Type below:
________

Answer:
\(\frac{4}{10}\) = 0.4

Explanation:
From the given model, 4 boxes are shaded out of 10 boxes. So, the fraction is \(\frac{4}{10}\) = 0.4

Question 5.
Go Math Grade 4 Answer Key Chapter 9 Relate Fractions and Decimals img 45
Type below:
________

Answer:
1\(\frac{3}{100}\) = 1.03

Explanation:
The model is divided into 100 equal parts. Each part represents the one-hundredth.
1\(\frac{3}{100}\) is 1 whole and 3 hundredths.

Write the number as hundredths in fraction form and decimal form.

Question 6.
\(\frac{8}{10}\)
Type below:
________

Answer:
\(\frac{80}{100}\)
grade 4 chapter 9 Relate Fractions and Decimals Image 5 509
0.80

Explanation:
Write \(\frac{8}{10}\) as an equivalent fraction.
\(\frac{8}{10}\) =\(\frac{8 × 10}{10× 10}\) = \(\frac{80}{100}\)
8 tenths is the same as 8 tenths 0 hundredths. So the decimal form = 0.80

Question 7.
0.5
Type below:
________

Answer:
\(\frac{50}{100}\)
grade 4 chapter 9 Relate Fractions and Decimals Image 3 509
0.50

Explanation:
Write 0.5 = \(\frac{5}{10}\) as an equivalent fraction.
\(\frac{5}{10}\) =\(\frac{5 × 10}{10× 10}\) = \(\frac{50}{100}\)
5 tenths is the same as 5 tenths 0 hundredths and also 0.50

Question 8.
Type below:
________

Answer:
b. \(\frac{2}{5}<\frac{1}{2}\)

Explanation:

Write the fraction or mixed number as a money amount, and as a decimal in terms of dollars.

Question 9.
\(\frac{95}{100}\)
amount: $ _____ decimal: _____ of a dollar

Answer:
amount: $0.95; decimal: 0.95

Explanation:
Write down 95 with the decimal point 2 spaces from the right (because 100 has 2 zeros)

Question 10.
1 \(\frac{48}{100}\)
amount: $ _____ decimal: _____ of a dollar

Answer:
amount: $1.48; decimal: 1.48

Explanation:
1\(\frac{48}{100}\) = \(\frac{148}{100}\)
Write down 148 with the decimal point 2 spaces from the right (because 100 has 2 zeros). So, 1.48 is the answer

Question 11.
\(\frac{4}{100}\)
amount: $ _____ decimal: _____ of a dollar

Answer:
amount: $0.04; decimal: 0.04

Explanation:
Write down 4 with the decimal point 2 spaces from the right (because 100 has 2 zeros). So, the answer is 0.04

Page No. 526

Question 12.
Ken’s turtle competed in a 0.50-meter race. His turtle had traveled \(\frac{4}{100}\)
meter when the winning turtle crossed the finish line. What is \(\frac{4}{100}\) written as a decimal?
_____

Answer:
decimal: 0.04

Explanation:
Write down 4 with the decimal point 2 spaces from the right (because 100 has 2 zeros). So, the answer is 0.04

Question 13.
Alex lives eight tenths of a mile from Sarah. What is eight tenths written as a decimal?
_____

Answer:
decimal: 0.8

Explanation:
Write down 8 with the decimal point 1 space from the right (because 100 has 1 zero). The decimal value of eight tenths is 0.8

Question 14.
What fraction and decimal, in hundredths, is equivalent to \(\frac{7}{10}\)?
Type below:
________

Answer:
\(\frac{7 x 10}{10 x 10}\) = 0.70

Explanation:
\(\frac{7}{10}\) = \(\frac{7 x 10}{10 x 10}\) = 0.70

Question 15.
Elaine found the following in her pocket. How much money was in her pocket?
Go Math Grade 4 Answer Key Chapter 9 Relate Fractions and Decimals img 46
$ _____

Answer:
$\(\frac{140}{100}\)

Explanation:
Given that 1 dollar, 1 quarter, 1 dime, 1 Nickel.
1 + 25/100 + 10/100 + 5/100 = 1 + 40/100 = 140/100

Question 16.
Three girls share $0.60. Each girl gets the same amount. How much money does each girl get?
$ _____

Answer:
$0.20

Explanation:
Three girls share $0.60. Each girl gets the same amount. So, $0.60/3 = $0.20

Question 17.
The deli scale weighs meat and cheese in hundredths of a pound. Sam put \(\frac{5}{10}\) pound of pepperoni on the deli scale. What weight does the deli scale show?
_____ hundredths

Answer:
50 hundredths

Explanation:
\(\frac{5}{10}\) = \(\frac{5 x 10}{10 x 10}\) = \(\frac{50}{100}\).
\(\frac{50}{100}\) written as 50 hundredths.

Page No. 529

Question 1.
Find \(\frac{7}{10}+\frac{5}{100}\)
Think: Write the addends as fractions with a common denominator.
\(\frac{■}{100}\) + \(\frac{■}{100}\) = \(\frac{■}{■}\)
\(\frac{□}{□}\)

Answer:
\(\frac{75}{100}\)

Explanation:
\(\frac{7}{10}+\frac{5}{100}\).
Write the addends as fractions with a common denominator
\(\frac{7}{10}\) = \(\frac{7 X 10}{10 X 10}\) = \(\frac{70}{100}\).
\(\frac{70}{100}+\frac{5}{100}\) = \(\frac{75}{100}\)

Find the sum.

Question 2.
\(\frac{1}{10}+\frac{11}{100}\) = \(\frac{□}{□}\)

Answer:
\(\frac{21}{100}\)

Explanation:
\(\frac{1}{10}+\frac{11}{100}\).
Write the addends as fractions with a common denominator
\(\frac{1}{10}\) = \(\frac{1 X 10}{10 X 10}\) = \(\frac{10}{100}\).
\(\frac{10}{100}+\frac{11}{100}\) = \(\frac{21}{100}\)

Question 3.
\(\frac{36}{100}+\frac{5}{10}\) = \(\frac{□}{□}\)

Answer:
\(\frac{86}{100}\)

Explanation:
\(\frac{36}{100}+\frac{5}{10}\).
Write the addends as fractions with a common denominator
\(\frac{5}{10}\) = \(\frac{5 X 10}{10 X 10}\) = \(\frac{50}{100}\).
\(\frac{36}{100}+\frac{50}{100}\) = \(\frac{86}{100}\).

Question 4.
$0.16 + $0.45 = $ _____

Answer:
$0.61

Explanation:
Think 0.16 as 16 hundredths = \(\frac{16}{100}\).
Think 0.45 as 45 hundredths = \(\frac{45}{100}\).
Write the addends as fractions with a common denominator
\(\frac{16}{100}\) + \(\frac{45}{100}\) = \(\frac{61}{100}\) = 0.61

Question 5.
$0.08 + $0.88 = $ _____

Answer:
$0.96

Explanation:
Think 0.08 as 8 hundredths = \(\frac{8}{100}\).
Think 0.88 as 88 hundredths = \(\frac{88}{100}\).
Write the addends as fractions with a common denominator.
\(\frac{8}{100}\) + \(\frac{88}{100}\) = \(\frac{96}{100}\) = 0.96

Question 6.
\(\frac{6}{10}+\frac{25}{100}\) = \(\frac{□}{□}\)

Answer:
\(\frac{85}{100}[/latex

Explanation:
[latex]\frac{6}{10}+\frac{25}{100}\)
Write the addends as fractions with a common denominator.
\(\frac{6}{10}\) = \(\frac{6 X 10}{10 X 10}\) = \(\frac{60}{100}\).
\(\frac{60}{100}+\frac{25}{100}\) = \(\frac{85}{100}\).

Question 7.
\(\frac{7}{10}+\frac{7}{100}\) = \(\frac{□}{□}\)

Answer:
50 hundredths

Explanation:
\(\frac{7}{10}+\frac{7}{100}\)
Write the addends as fractions with a common denominator.
\(\frac{7}{10}\) = \(\frac{7 X 10}{10 X 10}\) = \(\frac{70}{100}\).
\(\frac{70}{100}+\frac{7}{100}\) = \(\frac{77}{100}\).

Question 8.
$0.55 + $0.23 = $ _____

Answer:
$0.78

Explanation:
Think 0.55 as 55 hundredths = \(\frac{55}{100}\).
Think 0.23 as 23 hundredths = \(\frac{23}{100}\).
Write the addends as fractions with a common denominator.
\(\frac{55}{100}\) + \(\frac{23}{100}\) = \(\frac{78}{100}\) = 0.78.

Question 9.
$0.19 + $0.13 = $ _____

Answer:
$0.32

Explanation:
Think 0.19 as 19 hundredths = \(\frac{19}{100}\).
Think 0.13 as 13 hundredths = \(\frac{13}{100}\).
Write the addends as fractions with a common denominator.
\(\frac{19}{100}\) + \(\frac{13}{100}\) = \(\frac{32}{100}\) = 0.32.

Reason Quantitatively Algebra Write the number that makes the equation true.

Question 10.
\(\frac{20}{100}+\frac{■}{10}\) = \(\frac{60}{100}\)
■ = _____

Answer:
■ = 4

Explanation:
\(\frac{20}{100}+\frac{■}{10}\) = \(\frac{60}{100}\).
Let the unknown number = s.
If s = 4,
\(\frac{20}{100}+\frac{4}{10}\).
Write the addends as fractions with a common denominator.
\(\frac{4}{10}\) = \(\frac{4 X 10}{10 X 10}\) = \(\frac{40}{100}\).
\(\frac{20}{100}+\frac{40}{100}\) = \(\frac{60}{100}\).
So, the unknown number is 4.

Question 11.
\(\frac{2}{10}+\frac{■}{100}\) = \(\frac{90}{100}\)
■ = _____

Answer:
■ = 70

Explanation:
\(\frac{2}{10}+\frac{■}{100}\) = \(\frac{90}{100}\).
Let the unknown number = s.
If s = 70,
\(\frac{2}{10}+\frac{7}{100}\).
Write the addends as fractions with a common denominator.
\(\frac{2}{10}\) = \(\frac{2 X 10}{10 X 10}\) = \(\frac{20}{100}\).
\(\frac{20}{100}+\frac{70}{100}\) = \(\frac{90}{100}\).
So, the unknown number is 70.

Question 12.
Jerry had 1 gallon of ice cream. He used \(\frac{3}{10}\) gallon to make chocolate milkshakes and 0.40 gallon to make vanilla milkshakes. How much ice cream does Jerry have left after making the milkshakes?
_____ gallon

Answer:
0.30 gallon

Explanation:
Jerry had 1 gallon of ice cream. He used \(\frac{3}{10}\) gallon to make chocolate milkshakes and 0.40 gallon to make vanilla milkshakes.
So, write 0.40 as \(\frac{40}{100}\) gallon.
She used \(\frac{3}{10}\) + \(\frac{40}{100}\).
\(\frac{3}{10}\) = \(\frac{3 X 10}{10 X 10}\) = \(\frac{30}{100}\).
\(\frac{30}{100}\) + \(\frac{40}{100}\) = \(\frac{70}{100}\)
Jerry have left 1 – \(\frac{70}{100}\) = \(\frac{30}{100}\) = 0.30 gallon

Page No. 530

Use the table for 13−16.
Go Math Grade 4 Answer Key Chapter 9 Relate Fractions and Decimals img 47

Question 13.
Dean selects Teakwood stones and Buckskin stones to pave a path in front of his house. How many meters long will each set of one Teakwood stone and one Buckskin stone be?
_____ meter

Answer:
\(\frac{71}{100}\) meter

Explanation:
Dean selects Teakwood stones and Buckskin stones to pave a path in front of his house.
Teakwood stone and one Buckskin stone = \(\frac{3}{10}\) + \(\frac{41}{100}\).
Write the addends as fractions with a common denominator.
\(\frac{3}{10}\) = \(\frac{3 X 10}{10 X 10}\) = \(\frac{30}{100}\).
\(\frac{30}{100}\) + \(\frac{41}{100}\) = \(\frac{71}{100}\)

Question 14.
The backyard patio at Nona’s house is made from a repeating pattern of one Rose stone and one Rainbow stone. How many meters long is each pair of stones?
_____ meter

Answer:
\(\frac{68}{100}\) meter

Explanation:
The backyard patio at Nona’s house is made from a repeating pattern of one Rose stone and one Rainbow stone.
Each pair of stone = \(\frac{8}{100}\) + \(\frac{6}{10}\).
\(\frac{6}{10}\) = \(\frac{6 X 10}{10 X 10}\) = \(\frac{60}{100}\).
Each pair of stone = \(\frac{8}{100}\) + \(\frac{60}{100}\) = \(\frac{68}{100}\).

Question 15.
For a stone path, Emily likes the look of a Rustic stone, then a Rainbow stone, and then another Rustic stone. How long will the three stones in a row be? Explain.
_____ meter

Answer:
\(\frac{90}{100}\) meter

Explanation:
For a stone path, Emily likes the look of a Rustic stone, then a Rainbow stone, and then another Rustic stone. If three stones in a row, then
\(\frac{15}{100}\) + \(\frac{6}{10}\) + \(\frac{15}{100}\).
\(\frac{30}{100}\) + \(\frac{6}{10}\).
\(\frac{6}{10}\) = \(\frac{6 X 10}{10 X 10}\) = \(\frac{60}{100}\).
\(\frac{30}{100}\) + \(\frac{60}{100}\) = \(\frac{90}{100}\).

Question 16.
Which two stones can you place end-to-end to get a length of 0.38 meter? Explain how you found your answer.
Type below:
________

Answer:
If you add Teakwood stones and Rose stones, then you get a length of 0.38 meter.
\(\frac{3}{10}\) + \(\frac{8}{100}\).
\(\frac{3}{10}\) = \(\frac{3 X 10}{10 X 10}\) = \(\frac{30}{100}\).
\(\frac{30}{100}\) + \(\frac{8}{100}\) = latex]\frac{38}{100}[/latex] = 0.38.
If you add any other two stones, the answer will not equal to 0.38.

Question 17.
Christelle is making a dollhouse. The dollhouse is \(\frac{6}{10}\) meter tall without the roof. The roof is \(\frac{15}{100}\) meter high. What is the height of the dollhouse with the roof? Choose a number from each column to complete an equation to solve.
Go Math Grade 4 Answer Key Chapter 9 Relate Fractions and Decimals img 48
\(\frac{□}{□}\) meter

Answer:
\(\frac{60}{100}\) + \(\frac{15}{100}\) = \(\frac{75}{100}\) meter

Explanation:
\(\frac{6}{10}\) + \(\frac{15}{100}\).
\(\frac{6}{10}\) = \(\frac{6 X 10}{10 X 10}\) = \(\frac{60}{100}\).
\(\frac{60}{100}\) + \(\frac{15}{100}\) = \(\frac{75}{100}\).

Common Core – New – Page No. 531

Add Fractional Parts of 10 and 100

Find the sum.

Question 1.
\(\frac{2}{10}+\frac{43}{100}\) Think: Write \(\frac{2}{10}\) as a fraction with a denominator of 100: \(\frac{2 \times 10}{10 \times 10}=\frac{20}{100}\)
Go Math Grade 4 Answer Key Chapter 9 Relate Fractions and Decimals Common Core - New img 49

Answer:
\(\frac{63}{100}\)

Explanation:
Think: Write \(\frac{2}{10}\) as a fraction with a denominator of 100: \(\frac{2 \times 10}{10 \times 10}=\frac{20}{100}\)
Go Math Grade 4 Answer Key Chapter 9 Relate Fractions and Decimals Common Core - New img 49

Question 2.
\(\frac{17}{100}+\frac{6}{10}\)
\(\frac{□}{□}\)

Answer:
\(\frac{77}{100}\)

Explanation:
\(\frac{17}{100}+\frac{6}{10}\).
\(\frac{6 \times 10}{10 \times 10}=\frac{60}{100}\)
\(\frac{17}{100}+\frac{60}{100}\) = \(\frac{77}{100}\)

Question 3.
\(\frac{9}{100}+\frac{4}{10}\)
\(\frac{□}{□}\)

Answer:
\(\frac{49}{100}\)

Explanation:
\(\frac{9}{100}+\frac{4}{10}\).
\(\frac{4 \times 10}{10 \times 10}=\frac{40}{100}\)
\(\frac{9}{100}+\frac{40}{100}\) = \(\frac{49}{100}\)

Question 4.
\(\frac{7}{10}+\frac{23}{100}\)
\(\frac{□}{□}\)

Answer:
\(\frac{93}{100}\)

Explanation:
\(\frac{7}{10}+\frac{23}{100}\).
\(\frac{7 \times 10}{10 \times 10}=\frac{70}{100}\)
\(\frac{70}{100}+\frac{23}{100}\) = \(\frac{93}{100}\)

Question 5.
$0.48 + $0.30
$ _____

Answer:
$0.78

Explanation:
Think $0.48 as \(\frac{48}{100}\).
Think $0.30 as \(\frac{30}{100}\).
\(\frac{48}{100}+\frac{30}{100}\) = \(\frac{78}{100}\) = $0.78

Question 6.
$0.25 + $0.34
$ _____

Answer:
$0.59

Explanation:
Think $0.25 as \(\frac{25}{100}\).
Think $0.34 as \(\frac{34}{100}\).
\(\frac{25}{100}+\frac{34}{100}\) = \(\frac{59}{100}\) = $0.59

Question 7.
$0.66 + $0.06
$ _____

Answer:
$0.72

Explanation:
Think $0.66 as \(\frac{66}{100}\).
Think $0.06 as \(\frac{6}{100}\).
\(\frac{66}{100}+\frac{6}{100}\) = \(\frac{72}{100}\) = $0.72

Problem Solving

Question 8.
Ned’s frog jumped \(\frac{38}{100}\) meter. Then his frog jumped \(\frac{4}{10}\) meter. How far did Ned’s frog jump in all?
\(\frac{□}{□}\)

Answer:
\(\frac{78}{100}\) meter

Explanation:
Ned’s frog jumped \(\frac{38}{100}\) meter. Then his frog jumped \(\frac{4}{10}\) meter.
So, together \(\frac{38}{100}\) + \(\frac{4}{10}\) jumped.
\(\frac{4}{10}\) = \(\frac{4 \times 10}{10 \times 10}=\frac{40}{100}\).
\(\frac{38}{100}\) + \(\frac{40}{100}\) = \(\frac{78}{100}\).

Question 9.
Keiko walks \(\frac{5}{10}\) kilometer from school to the park. Then she walks \(\frac{19}{100}\) kilometer from the park to her home. How far does Keiko walk in all?
\(\frac{□}{□}\)

Answer:
\(\frac{69}{100}\) kilometer

Explanation:
Keiko walks \(\frac{5}{10}\) kilometer from school to the park. Then she walks \(\frac{19}{100}\) kilometer from the park to her home.
Total = \(\frac{5}{10}\) + \(\frac{19}{100}\) kilometer.
\(\frac{5}{10}\) = \(\frac{5 \times 10}{10 \times 10}=\frac{50}{100}\).
\(\frac{50}{100}\) + \(\frac{19}{100}\) = \(\frac{69}{100}\).

Common Core – New – Page No. 532

Lesson Check

Question 1.
In a fish tank, \(\frac{2}{10}\) of the fish were orange and \(\frac{5}{100}\) of the fish were striped. What fraction of the fish were orange or striped?
Options:
a. \(\frac{7}{10}\)
b. \(\frac{52}{100}\)
c. \(\frac{25}{100}\)
d. \(\frac{7}{100}\)

Answer:
c. \(\frac{25}{100}\)

Explanation:
In a fish tank, \(\frac{2}{10}\) of the fish were orange and \(\frac{5}{100}\) of the fish were striped.
To find the raction of the fish were orange or striped Add \(\frac{2}{10}\) and \(\frac{5}{100}\).
\(\frac{2}{10}\) = \(\frac{2 \times 10}{10 \times 10}=\frac{20}{100}\).
\(\frac{20}{100}\) + \(\frac{5}{100}\) = \(\frac{25}{100}\).

Question 2.
Greg spends $0.45 on an eraser and $0.30 on a pen. How much money does Greg spend in all?
Options:
a. $3.45
b. $0.75
c. $0.48
d. $0.15

Answer:
b. $0.75

Explanation:
Think $0.45 as \(\frac{45}{100}\).
Think $0.30 as \(\frac{30}{100}\).
\(\frac{45}{100}+\frac{30}{100}\) = \(\frac{75}{100}\) = $0.75.

Spiral Review

Question 3.
Phillip saves $8 each month. How many months will it take him to save at least $60?
Options:
a. 6 months
b. 7 months
c. 8 months
d. 9 months

Answer:
c. 8 months

Explanation:
Phillip saves $8 each month.
To save at least $60, \(\frac{60}{8}\) = 8 months (approximately)

Question 4.
Ursula and Yi share a submarine sandwich. Ursula eats \(\frac{2}{8}\) of the sandwich. Yi eats \(\frac{3}{8}\) of the sandwich. How much of the sandwich do the two friends eat?
Options:
a. \(\frac{1}{8}\)
b. \(\frac{4}{8}\)
c. \(\frac{5}{8}\)
d. \(\frac{6}{8}\)

Answer:
c. \(\frac{5}{8}\)

Explanation:
Ursula and Yi share a submarine sandwich. Ursula eats \(\frac{2}{8}\) of the sandwich. Yi eats \(\frac{3}{8}\) of the sandwich.
Two friends eat \(\frac{2}{8}\) + \(\frac{3}{8}\) = \(\frac{5}{8}\)

Question 5.
A carpenter has a board that is 8 feet long. He cuts off two pieces. One piece is 3 \(\frac{1}{2}\) feet long and the other is 2 \(\frac{1}{3}\) feet long. How much of the board is left?
Options:
a. 2 \(\frac{1}{6}\)
b. 2 \(\frac{5}{6}\)
c. 3 \(\frac{1}{6}\)
d. 3 \(\frac{5}{6}\)

Answer:
a. 2 \(\frac{1}{6}\)

Explanation:
3 \(\frac{1}{2}\) = \(\frac{7}{2}\).
2 \(\frac{1}{3}\) = \(\frac{7}{3}\).
A carpenter has a board that is 8 feet long. He cuts off two pieces. One piece is 3 \(\frac{1}{2}\) feet long and the other is 2 \(\frac{1}{3}\) feet long.
\(\frac{7}{2}\) + \(\frac{7}{3}\) = \(\frac{7 \times 3}{2\times 3} + [latex]\frac{7 \times 2}{3\times 2} = [latex]\frac{21}{6}\) + \(\frac{14}{6}\) = \(\frac{35}{6}\) = 5\(\frac{5}{6}\).
He left 8 – 5\(\frac{5}{6}\).
7\(\frac{6}{6}\) – 5\(\frac{5}{6}\) = 2\(\frac{1}{6}\)

Question 6.
Jeff drinks \(\frac{2}{3}\) of a glass of juice. Which fraction is equivalent to \(\frac{2}{3}\)?
Options:
a. \(\frac{1}{3}\)
b. \(\frac{3}{2}\)
c. \(\frac{3}{6}\)
d. \(\frac{8}{12}\)

Answer:
d. \(\frac{8}{12}\)

Explanation:
\(\frac{8}{12}\) is divided by 4. So, \(\frac{8}{12}\) = \(\frac{2}{3}\).

Page No. 535

Question 1.
Compare 0.39 and 0.42. Write <, >, or =.
Shade the model to help.
Go Math Grade 4 Answer Key Chapter 9 Relate Fractions and Decimals img 50
0.39 ____ 0.42

Answer:
grade 4 chapter 9 Relate Fractions and Decimals Image 1 535
0.39 < 0.42

Compare. Write <, >, or =.

Question 2.
Go Math Grade 4 Answer Key Chapter 9 Relate Fractions and Decimals img 51
0.26 ____ 0.23

Answer:
0.26 > 0.23

Explanation:
grade 4 chapter 9 Relate Fractions and Decimals Image 2 535
The digits in the one’s and tenths place are the same. Compare the digits in the hundredths place. 6 > 3. So, 0.26 > 0.23.

Question 3.
Go Math Grade 4 Answer Key Chapter 9 Relate Fractions and Decimals img 52
0.7 ____ 0.54

Answer:
0.7 > 0.54

Explanation:
grade 4 chapter 9 Relate Fractions and Decimals Image 3 535
The digits in the ones place are the same. Compare the digits in the tenths place. 0.7 = 0.70. 7 > 5. So, 0.70 > 0.54.

Question 4.
Go Math Grade 4 Answer Key Chapter 9 Relate Fractions and Decimals img 53
1.15 ____ 1.3

Answer:
1.15 < 1.3

Explanation:
grade 4 chapter 9 Relate Fractions and Decimals Image 4 535
The digits in the ones place are the same. Compare the digits in the tenths place. 1 < 3. So, 1.15 < 1.3

Question 5.
Go Math Grade 4 Answer Key Chapter 9 Relate Fractions and Decimals img 54
4.5 ____ 2.89

Answer:
4.5 > 2.89

Explanation:
grade 4 chapter 9 Relate Fractions and Decimals Image 5 535
Compare one’s digits. 4 > 2 . So, 4.5 > 2.89

Compare. Write <, >, or =.

Question 6.
0.9 ____ 0.81

Answer:
0.9 > 0.81

Explanation:
0.9 is 9 tenths, which is equivalent to 90 hundredths.
0.81 is 81 hundredths.
90 hundredths > 81 hundredths. So, 0.9 > 0.81.

Question 7.
1.06 ____ 0.6

Answer:
1.06 > 0.6

Explanation:
1.06 is 106 hundredths.
0.6 is 6 tenths, which is equivalent to 60 hundredths.
106 hundredths > 60 hundredths. So, 1.06 > 0.6.

Question 8.
0.25 ____ 0.3

Answer:
0.25 < 0.3

Explanation:
0.25 is 25 hundredths.
0.3 is 3 tenths, which is equivalent to 30 hundredths.
25 hundredths < 30 hundredths. So, 0.25 < 0.3.

Question 9.
2.61 ____ 3.29

Answer:
2.61 < 3.29

Explanation:
2.61 is 261 hundredths.
3.29 is 329 hundredths.
261 hundredths < 329 hundredths. So, 2.61 < 3.29.

Reason Quantitatively Compare. Write <, >, or =.

Question 10.
0.30 ____ \(\frac{3}{10}\)

Answer:
0.30 = \(\frac{3}{10}\)

Explanation:
0.30 is 30 hundredths.
\(\frac{3}{10}\) is 3 tenths, which is equal to 30 hundredths.
30 hundredths = 30 hundredths. So, 0.30 = \(\frac{3}{10}\).

Question 11.
\(\frac{4}{100}\) ____ 0.2

Answer:
\(\frac{4}{100}\) < 0.2

Explanation:
\(\frac{4}{100}\) is 4 hundredths.
0.2 is 2 tenths, which is equal to 20 hundredths.
4 hundredths < 20 hundredths. So, \(\frac{4}{100}\) < 0.2

Question 12.
0.15 ____ \(\frac{1}{10}\)

Answer:
0.15 > \(\frac{1}{10}\)

Explanation:
0.15 is 15 hundredths.
\(\frac{1}{10}\) is 1 tenths, which is equal to 10 hundredths.
15 hundredths > 10 hundredths. So, 0.15 > \(\frac{1}{10}\).

Question 13.
\(\frac{1}{8}\) ____ 0.8

Answer:
latex]\frac{1}{8}[/latex] < 0.8

Explanation:
\(\frac{1}{8}\) = 0.25 is 25 hundredths.
0.8 is 8 tenths, which is equal to 80 hundredths.
25 hundredths < 80 hundredths. So, \(\frac{1}{8}\) < 0.8

Question 14.
Robert had $14.53 in his pocket. Ivan had $14.25 in his pocket. Matt had $14.40 in his pocket. Who had more money, Robert or Matt? Did Ivan have more money than either Robert or Matt?
________

Answer:
Robert had more money.
No, Ivan didn’t have more money than either Robert or Matt.

Explanation:
Compare Robert, Ivan, and Matt money to know who had more money.
The digits in the one’s place are the same. Compare the digits in the tenths place. 5 > 4 > 2. So, Robert had more money.

Page No. 536

Question 15.
Ricardo and Brandon ran a 1500-meter race. Ricardo finished in 4.89 minutes. Brandon finished in 4.83 minutes. What was the time of the runner who finished first?
a. What are you asked to find?–
Type below:
________

Answer:
The time of the runner who finished first.

Question 15.
b. What do you need to do to find the answer?
Type below:
________

Answer:
I have to compare the times to find the time that is less.

Question 15.
c. Solve the problem.
Type below:
________

Answer:
Use place-value chart
grade 4 chapter 9 Relate Fractions and Decimals Image 1 536
The digits of the one’s and tenths are equal. So, compare hundredths to find greater time.
9 > 3.
4.83 minutes are less than 4.89.

Question 15.
d. What was the time of the runner who finished first?
______ minutes

Answer:
4.83 minutes

Question 15.
e. Look back. Does your answer make sense? Explain.
_____

Answer:
Yes. The time of the runner who finished first is the lesser time of the two. Since 4.83, 4.89, then 4.83 minutes is the time of the runner who finished first.

Question 16.
The Venus flytrap closes in 0.3 second and the waterwheel plant closes in 0.2 second. What decimal is halfway between 0.2 and 0.3? Explain.
_____

Answer:
0.2 is 2 tenths, which is equal to the 20 hundredths.
0.3 is 3 tenths, which is equal to 30 hundredths.
The halfway between 20 hundredths and 30 hundredths is 25 hundredths.
So, the answer is 0.25.

Question 17.
For numbers 17a–17c, select True or False for the inequality.
a. 0.5 > 0.53
i. True
ii. False

Answer:
ii. False

Explanation:
0.5 is 50 hundredths.
0.53 is 53 hundredths.
50 hundredths < 53 hundredths. So, 0.5 < 0.53. So, the answer is false.

Question 17.
b. 0.35 < 0.37
i. True
ii. False

Answer:
i. True

Explanation:
0.35 is 35 hundredths.
0.37 is 37 hundredths.
35 hundredths < 37 hundredths.
0.35 < 0.37.
So, the answer is true.

Question 17. c. $1.35 > $0.35
i. True
ii. False

Answer:
i. True

Explanation:
$1.35 is 135 hundredths.
$0.35 is 35 hundredths.
135 hundredths > 35 hundredths.
$1.35 > $0.35.
So, the answer is correct.

Common Core – New – Page No. 537

Compare Decimals

Compare. Write <. >, or =.

Question 1.
Go Math Grade 4 Answer Key Chapter 9 Relate Fractions and Decimals Common Core - New img 55
Think: 3 tenths is less than 5 tenths. So, 0.35 < 0.53

Answer:
0.35 < 0.53

Explanation:
3 tenths is less than 5 tenths. So, 0.35 < 0.53

Question 2.
0.6 ______ 0.60

Answer:
0.6 = 0.60

Explanation:
0.6 is 6 tenths can write as 6 tenths and 0 hundredths. So, 0.6 = 0.60.

Question 3.
0.24 ______ 0.31

Answer:
0.24 < 0.31

Explanation:
2 tenths is less than 3 tenths. So, 0.24 < 0.31.

Question 4.
0.94 ______ 0.9

Answer:
0.94 > 0.9

Explanation:
The digits of tenths are equal. So, compare hundredths. 4 hundredths is greater than 0 hundredths. So, 0.94 > 0.9.

Question 5.
0.3 ______ 0.32

Answer:
0.3 < 0.32

Explanation:
The digits of tenths are equal. So, compare hundredths. 0 hundredths is less than 2 hundredths. So, 0.3 < 0.32.

Question 6.
0.45 ______ 0.28

Answer:
0.45 > 0.28

Explanation:
4 tenths is greater than 2 tenths. So, 0.45 > 0.28.

Question 7.
0.39 ______ 0.93

Answer:
0.39 < 0.93

Explanation:
3 tenths is less than 9 tenths. So, 0.39 < 0.93.

Use the number line to compare. Write true or false.
Go Math Grade 4 Answer Key Chapter 9 Relate Fractions and Decimals Common Core - New img 56

Question 8.
0.8 > 0.78
______

Answer:
true

Explanation:
0.78 is in between 0.7 and 0.8 that is less than 0.8. So, 0.8 > 0.78.

Question 9.
0.4 > 0.84
______

Answer:
false

Explanation:
0.4 is less than 0.84 and the left side of the number line. So, 0.4 < 0.84. The answer is false.

Question 10.
0.7 > 0.70
______

Answer:
false

Explanation:
0.7 is 7 tenths and 70 hundredths. 0.7 = 0.70. So, the answer is false.

Question 11.
0.4 > 0.04
______

Answer:
true

Explanation:
0.04 is less than 0.4 and it is left side of the 0.1 on the number line. 0.1 is less than 0.4. So, the given answer is true.

Compare. Write true or false.

Question 12.
0.09 > 0.1
______

Answer:
false

Explanation:
0 tenths is less than 1 tenths. So, 0.09 < 0.1. So, the answer is false.

Question 13.
0.24 = 0.42
______

Answer:
false

Explanation:
2 tenths is less than 4 tenths. So, 0.24 < 0.42. So, the answer is false.

Question 14.
0.17 < 0.32 ______

Answer:
true

Explanation:
1 tenths is less than 3 tenths. So, 0.17 < 0.32. So, the answer is true.

Question 15.
0.85 > 0.82
______

Answer:
true

Explanation:
The digits of tenths are equal. So, compare hundredths. 5 hundredths is greater than 2 hundredths. So, 0.85 > 0.82.

Question 16.
Kelly walks 0.7 mile to school. Mary walks 0.49 mile to school. Write an inequality using <, > or = to compare the distances they walk to school.
0.7 ______ 0.49

Answer:
0.7 > 0.49

Explanation:
7 tenths is greater than 4 tenths. So, 0.7 > 0.49.

Question 17.
Tyrone shades two decimal grids. He shades 0.03 of the squares on one grid blue. He shades 0.3 of another grid red. Which grid has the greater part shaded?
0.03 ______ 0.3

Answer:
0.03 < 0.3

Explanation:
0.03 is 3 hundredths.
0.3 is 3 tenths, which is equal to 30 hundredths.
3 hundredths < 30 hundredths. So, 0.03 < 0.3.

Common Core – New – Page No. 538

Lesson Check

Question 1.
Bob, Cal, and Pete each made a stack of baseball cards. Bob’s stack was 0.2 meter high. Cal’s stack was 0.24 meter high. Pete’s stack was 0.18 meter high.
Which statement is true?
Options:
a. 0.02 > 0.24
b. 0.24 > 0.18
c. 0.18 > 0.2
d. 0.24 = 0.2

Answer:
b. 0.24 > 0.18

Explanation:
2 tenths is greater than 1 tenth. So, 0.24 > 0.18.

Question 2.
Three classmates spent money at the school supplies store. Mark spent 0.5 dollar, Andre spent 0.45 dollar, and Raquel spent 0.52 dollar. Which
statement is true?
Options:
a. 0.45 > 0.5
b. 0.52 < 0.45
c. 0.5 = 0.52
d. 0.45 < 0.5

Answer:
d. 0.45 < 0.5

Explanation:
4 tenths is less than 5 tenth. So, 0.45 > 0.5.

Spiral Review

Question 3.
Pedro has $0.35 in his pocket. Alice has $0.40 in her pocket. How much money do Pedro and Alice have in their pockets altogether?
Options:
a. $0.05
b. $0.39
c. $0.75
d. $0.79

Answer:
c. $0.75

Explanation:
Pedro has $0.35 in his pocket. Alice has $0.40 in her pocket.
Together = $0.35 + $0.40 = $0.75.

Question 4.
The measure 62 centimeters is equivalent to \(\frac{62}{100}\) meter. What is this measure written as a decimal?
Options:
a. 62.0 meters
b. 6.2 meters
c. 0.62 meter
d. 0.6 meter

Answer:
c. 0.62 meter

Explanation:
\(\frac{62}{100}\) = 0.62 meter.

Question 5.
Joel has 24 sports trophies. Of the trophies, \(\frac{1}{8}\) are soccer trophies. How many soccer trophies does Joel have?
Options:
a. 2
b. 3
c. 4
d. 6

Answer:
b. 3

Explanation:
Joel has 24 sports trophies. Of the trophies, \(\frac{1}{8}\) are soccer trophies.
So, \(\frac{1}{8}\) X 24 = 3 soccer trophies.

Question 6.
Molly’s jump rope is 6 \(\frac{1}{3}\) feet long. Gail’s jump rope is 4 \(\frac{2}{3}\) feet long. How much longer is Molly’s jump rope?
Options:
a. 1 \(\frac{1}{3}\) feet
b. 1 \(\frac{2}{3}\) feet
c. 2 \(\frac{1}{3}\) feet
d. 2 \(\frac{2}{3}\) feet

Answer:
b. 1 \(\frac{2}{3}\) feet

Explanation:
6 \(\frac{1}{3}\) feet = \(\frac{19}{3}\) feet.
4 \(\frac{2}{3}\) feet = \(\frac{14}{3}\) feet.
\(\frac{19}{3}\) – \(\frac{14}{3}\) = \(\frac{5}{3}\) feet = b. 1 \(\frac{2}{3}\) feet.

Page No. 539

Question 1.
Select a number shown by the model. Mark all that apply.
Go Math Grade 4 Answer Key Chapter 9 Relate Fractions and Decimals img 57
Type below:
________

Answer:
1 \(\frac{4}{10}\) = \(\frac{14}{10}\) = 1.4

Explanation:
from the given image, there is one whole number and \(\frac{4}{10}\) of another model. So, 1 \(\frac{4}{10}\) = \(\frac{14}{10}\) = 1.4

Question 2.
Rick has one dollar and twenty-seven cents to buy a notebook. Which names this money amount in terms of dollars? Mark all that apply.
Options:
a. 12.7
b. 1.027
c. $1.27
d. 1.27
e. 1 \(\frac{27}{100}\)
f. \(\frac{127}{10}\)

Answer:
c. $1.27
d. 1.27
e. 1 \(\frac{27}{100}\)

Explanation:
one dollar and twenty-seven cents = 1 \(\frac{27}{100}\) = 1.27 = $1.27

Question 3.
For numbers 3a–3e, select True or False for the statement.
a. 0.9 is equivalent to 0.90.
i. True
ii. False

Answer:
i. True

Explanation:
0.9 is 9 tenths, which is equal to 90 hundredths. 0.9 = 0.90. So, the answer is true.

Question 3.
b. 0.20 is equivalent to \(\frac{2}{100}\)
i. True
ii. False

Answer:
ii. False

Explanation:
\(\frac{2}{100}\) = 0.02. So, the given answer is false.

Question 3.
c. \(\frac{80}{100}\) is equivalent to \(\frac{8}{10}\).
i. True
ii. False

Answer:
i. True

Explanation:
Divide \(\frac{80}{100}\) by 10 = \(\frac{8}{10}\). So, the answer is true.

Question 3.
d. \(\frac{6}{10}\) is equivalent to 0.60.
i. True
ii. False

Answer:
i. True

Explanation:
\(\frac{6}{10}\) is 0.6. 0.6 is 6 tenths, which is equal to 6 tenths and 0 hundredths. 0.60. So, 0.6 =0.60. The answer is true.

Question 3.
e. 0.3 is equivalent to \(\frac{3}{100}\)
i. True
ii. False

Answer:
ii. False

Explanation:
0.3 is 3 tenths, which is equal to 3 tenths and 0 hundredths. \(\frac{3}{100}\) is 0 tenths. So, the answer is false.

Page No. 540

Question 4.
After selling some old books and toys, Gwen and her brother Max had 5 one-dollar bills, 6 quarters, and 8 dimes. They agreed to divide the money equally.
Part A
Wat is the total amount of money that Gwen and Max earned?
Explain.
$ _____

Answer:
$7.30

Explanation:
After selling some old books and toys, Gwen and her brother Max had 5 one-dollar bills, 6 quarters, and 8 dimes.
5 + (6 X 25/100) + (8 X 10/100) = 5 + 150/100 + 80/100 = 5 + 230/100 = 730/100 = 7.30

Question 4.
Part B
Max said that he and Gwen cannot get equal amounts of money because 5 one-dollar bills cannot be divided evenly. Do you agree with Max?
Explain.
_____

Answer:
ii. False

Explanation:
No; they can share the 3 quarters and 4 dimes each. Then, they can change the 5 dollar bills into quarters. 1 dollar = 4 quarters. So, 5 dollars = 5 X 4 or 20 quarters. They can each get 10 quarters. So, each person has a total of 13 quarters and 4 dimes. $3.25 + $0.40 = $3.65

Question 5.
Harrison rode his bike \(\frac{6}{10}\) of a mile to the park. Shade the model. Then write the decimal to show how far Harrison rode his bike.
Go Math Grade 4 Answer Key Chapter 9 Relate Fractions and Decimals img 58
Harrison rode his bike _______ mile to the park.
_____

Answer:
grade 4 chapter 9 Relate Fractions and Decimals Image 1 540
Harrison rode his bike 0.6 mile to the park.

Explanation:
6 boxes are shaded out of 10.

Question 6.
Amaldo spent \(\frac{88}{100}\) of a dollar on a souvenir pencil from Zion National Park in Utah. What is \(\frac{88}{100}\) written as a decimal in terms of dollars?
_____

Answer:
0.88

Explanation:
Write down 88 with the decimal point 2 spaces from the right (because 100 has 2 zeros). 0.88

Question 7.
Tran has $5.82. He is saving for a video game that costs $8.95.
Tran needs _______ more to have enough money for the game.
_____

Answer:
$3.13

Explanation:
Tran has $5.82. He is saving for a video game that costs $8.95. To know more amount need to buy a video game = $8.95 – $5.82 = $3.13

Page No. 541

Question 8.
Cheyenne lives \(\frac{7}{10}\) mile from school. A fraction in hundredths equal to \(\frac{7}{10}\) is
\(\frac{□}{□}\)

Answer:
\(\frac{70}{100}\)

Explanation:
\(\frac{7}{10}\) = \(\frac{7 \times 10}{10 \times 10}\) = \(\frac{70}{100}\)

Question 9.
Write a decimal in tenths that is less than 2.42 but greater than 2.0.
Type below:
__________

Answer:
2.1, 2.2, 2.3, 2.4

Explanation:
The decimal in greater than 2.0 and below the 2.4 are 2.1, 2.2, 2.3, 2.4

Question 10.
Kylee and two of her friends are at a museum. They find two quarters and one dime on the ground.
Part A
If Kylee and her friends share the money equally, how much will each person get? Explain how you found your answer.
$ _____
Explain:
__________

Answer:
$0.20; Two quarters and one dime are equal to $0.50 + $0.10 = $0.60. Take $0.60 as 6 dimes. When 6 dimes divide equally, each person will receive 2 dimes or $0.20.

Question 10.
Part B
Kylee says that each person will receive \(\frac{2}{10}\) of the money that was found. Do you agree? Explain.
__________

Answer:
No; Each person receives $0.20, which is 2/10 of a dollar, not 2/10 of the money that was found. Since there are 3 people who share the money equally, each person will receive 1/3 of the money.

Question 11.
Shade the model to show 1 \(\frac{52}{100}\). Then write the mixed number in decimal form.
Go Math Grade 4 Answer Key Chapter 9 Relate Fractions and Decimals img 59
_____

Answer:
grade 4 chapter 9 Relate Fractions and Decimals Image 2 541
1.52

Page No. 542

Question 12.
Henry is making a recipe for biscuits. A recipe calls for \(\frac{5}{10}\) kilogram flour and \(\frac{9}{100}\) kilogram sugar.
Part A
If Henry measures correctly and combines the two amounts, how much flour and sugar will he have? Show your work.
\(\frac{□}{□}\) kilogram

Answer:
\(\frac{59}{100}\) kilogram

Explanation:
Henry is making a recipe for biscuits. A recipe calls for \(\frac{5}{10}\) kilogram flour and \(\frac{9}{100}\) kilogram sugar. So, add \(\frac{5}{10}\) kilogram flour and \(\frac{9}{100}\) kilogram flour.
\(\frac{5}{10}\) = \(\frac{5 \times 10}{10 \times 10}\) = \(\frac{50}{100}\).
\(\frac{50}{100}\) + \(\frac{9}{100}\) = \(\frac{59}{100}\).

Question 12.
Part B
How can you write your answer as a decimal?
__________ kilogram

Answer:
0.59 kilogram

Explanation:
\(\frac{59}{100}\) = 0.59

Question 13.
An orchestra has 100 musicians. \(\frac{4}{10}\) of them play string instruments—violin, viola, cello, double bass, guitar, lute, and harp. What decimal is equivalent to \(\frac{4}{10}\)?
__________

Answer:
0.4 or 0.40

Explanation:
\(\frac{4}{10}\) = 0.4 = 0.40

Question 14.
Complete the table.
Go Math Grade 4 Answer Key Chapter 9 Relate Fractions and Decimals img 60

Answer:
grade 4 chapter 9 Relate Fractions and Decimals Image 3 541

Question 15.
The point on the number line shows the number of seconds it took an athlete to run the forty-yard dash. Write the decimal that correctly names the point.
Go Math Grade 4 Answer Key Chapter 9 Relate Fractions and Decimals img 61

Answer:
\(\frac{70}{100}\)

Explanation:
The point is in between 5\(\frac{5}{10}\) and 6.0. The point after the 5\(\frac{5}{10}\) is 5\(\frac{6}{10}\) = 5.6

Page No. 543

Question 16.
Ingrid is making a toy car. The toy car is \(\frac{5}{10}\) meter high without the roof. The roof is \(\frac{18}{100}\) meter high. What is the height of the toy car with the roof? Choose a number from each column to complete an equation to solve.
Go Math Grade 4 Answer Key Chapter 9 Relate Fractions and Decimals img 62
Type below:
__________

Answer:
\(\frac{50}{100}\) + \(\frac{18}{100}\) = \(\frac{68}{100}\) meter high

Explanation:
\(\frac{5}{10}\) = \(\frac{5 \times 10}{10 \times 10}\) = \(\frac{50}{100}\).
\(\frac{50}{100}\) + \(\frac{18}{100}\) = \(\frac{68}{100}\).

Question 17.
Callie shaded the model to represent the questions she answered correctly on a test. What decimal represents the part of the model that is shaded?
Go Math Grade 4 Answer Key Chapter 9 Relate Fractions and Decimals img 63
represents _____

Answer:
0.81

Explanation:
81 boxes are shaded out of 100. So, \(\frac{81}{100}\) = 0.81

Question 18.
For numbers 18a–18f, select True or False for the inequality.
a. 0.21 < 0.27
i. True
ii. False

Answer:
i. True

Explanation:
The digits in the one’s and tenths place are the same. Compare the digits in the hundredths place. 1 < 7. So, 0.21 < 0.27. The answer is true.

Question 18. b. 0.4 > 0.45

i. True
ii. False

Answer:
ii. False

Explanation:
0.4 = 0.40
The digits in the one’s and tenths place are the same. Compare the digits in the hundredths place. 0 < 5. So, 0.4 < 0.46. The answer is false.

Question 18.
c. $3.21 > $0.2
i. True
ii. False

Answer:
i. True

Explanation:
3 ones is greater than 0’s. So, $3.21 > $0.2

Question 18.
d. 1.9 < 1.90
i. True
ii. False

Answer:
ii. False

Explanation:
1.9 = 1.90. So, the answer is false

Question 18. e. 0.41 = 0.14
i. True
ii. False

Answer:
ii. False

Explanation:
The digits in the one’s are the same. Compare the digits in the tenths place. 4 > 1. So, 0.41 > 0.14. The answer is false.

Question 18. f. 6.2 > 6.02
i. True
ii. False

Answer:
i. True

Explanation:
2 tenths is greater than 0 tenths. So, 6.2 > 6.02. The answer is true.

Question 19.
Fill in the numbers to find the sum.
Go Math Grade 4 Answer Key Chapter 9 Relate Fractions and Decimals img 64
Type below:
__________

Answer:
\(\frac{4}{10}\) + \(\frac{40}{100}\) = \(\frac{8}{10}\)

Explanation:
Let the unknown numbers are A and B.
\(\frac{4}{10}\) + \(\frac{A}{100}\) = \(\frac{8}{B}\)
If A = 40 and B = 10, then \(\frac{4}{10}\) + \(\frac{40}{100}\) = \(\frac{8}{10}\).

Page No. 544

Question 20.
Steve is measuring the growth of a tree. He drew this model to show the tree’s growth in meters. Which fraction, mixed number, or decimal does the model show? Mark all that apply.
Go Math Grade 4 Answer Key Chapter 9 Relate Fractions and Decimals img 65
Options:
a. 1.28
b. 12.8
c. 0.28
d. 2 \(\frac{8}{100}\)
e. 1 \(\frac{28}{100}\)
f. 1 \(\frac{28}{10}\)

Answer:
a. 1.28
e. 1 \(\frac{28}{100}\)

Explanation:
From the given image, it has one model of 1 whole number and other model is shades 24 boxes out of 100. So, 1 \(\frac{28}{100}\) = \(\frac{128}{100}\) = 1.28 is the answer.

Question 21.
Luke lives 0.4 kilometer from a skating rink. Mark lives 0.25 kilometer from the skating rink.
Part A
Who lives closer to the skating rink? Explain.
_____

Answer:
Mark lives closer to the skating rink

Explanation:
0.4 is 4 tenths and 0.25 is 2 tenths 5 hundredths. Compare the tenths, since
4 tenths > 2 tenths. Luke lives farther from the rink. So, Mark lives closer.

Question 21.
Part B
How can you write each distance as a fraction? Explain.
Type below:
__________

Answer:
\(\frac{4}{10}\) and \(\frac{25}{100}\)

Explanation:
0.4 is 4 tenths. So, \(\frac{4}{10}\) and 0.25 is 25 hundredths. So, \(\frac{25}{100}\).

Question 21.
Part C
Luke is walking to the skating rink to pick up a practice schedule. Then he is walking to Mark’s house. Will he walk more than a kilometer or less than a kilometer? Explain.
__________

Answer:
Less than a kilometer; \(\frac{4}{10}\) < \(\frac{5}{10}\) or \(\frac{1}{2}\) and \(\frac{25}{100}\) < \(\frac{50}{100}\) or \(\frac{1}{2}\).
\(\frac{4}{10}\) + \(\frac{25}{100}\) < \(\frac{1}{2}\) + \(\frac{1}{2}\). So, \(\frac{1}{2}\) + \(\frac{1}{2}\) = 1.
Therefore, \(\frac{4}{10}\) + \(\frac{25}{100}\) < 1.

Page No. 551

Question 1.
Draw and label \(\overline{A B}\) in the space at the right.
\(\overline{A B}\) is a __________ .
__________

Answer:
grade 4 chapter 9 review test image 1 551
\(\overline{A B}\) is a line segment.

Draw and label an example of the figure.

Question 2.
\(\underset { XY }{ \longleftrightarrow } \)
Type below:
__________

Answer:
grade 4 chapter 9 review test image 2 551
\(\underset { XY }{ \longleftrightarrow } \) is a line

Question 3.
obtuse ∠K
Type below:
__________

Answer:
grade 4 chapter 9 review test image 3 551
Angle K is greater than a right angle and less than a straight angle.

Question 4.
∠CDE
Type below:
__________

Answer:
grade 4 chapter 9 review test image 4 551
angle CDE

Use Figure M for 5 and 6.
Go Math Grade 4 Answer Key Chapter 9 Relate Fractions and Decimals img 66

Question 5.
Name a line segment.
Type below:
__________

Answer:
line segment TU

Explanation:
TU line is a straight path of points that continues without an end in both directions.

Question 6.
Name a right angle.
Type below:
__________

Answer:
Angle TUW

Explanation:
TUW is a right angle that forms a square corner.

Draw and label an example of the figure.

Question 7.
\(\overrightarrow{P Q}\)
Type below:
__________

Answer:
grade 4 chapter 9 review test image 5 551
\(\overrightarrow{P Q}\) is a ray.

Question 8.
acute ∠RST
Type below:
__________

Answer:
grade 4 chapter 9 review test image 6 551
Angle RST

Question 9.
straight ∠WXZ
Type below:
__________

Answer:
grade 4 chapter 9 review test image 7 551

Use Figure F for 10–15.
Go Math Grade 4 Answer Key Chapter 9 Relate Fractions and Decimals img 67

Question 10.
Name a ray.
Type below:
__________

Answer:
Ray K

Explanation:
K is a ray that has one endpoint and continues without an end in one direction.

Question 11.
Name an obtuse angle.
Type below:
__________

Answer:
Angle ABK

Explanation:
ABK is an obtuse angle that is greater than a right angle and less than a straight angle.

Question 12.
Name a line.
Type below:
__________

Answer:
Line AC

Explanation:
AC is a line that is a straight path of points that continues without end in
both directions.

Question 13.
Name a line segment.
Type below:
__________

Answer:
Line Segment PQ

Explanation:
PQ is a line segment that is part of a line between two endpoints.

Question 14.
Name a right angle.
Type below:
__________

Answer:
Angle PRC

Explanation:
PRC is a right angle that forms a square corner.

Question 15.
Name an acute angle.
Type below:
__________

Answer:
Angle ABJ

Explanation:
ABJ is an acute angle that is less than a right angle.

Page No. 552

Use the picture of the bridge for 16 and 17.
Go Math Grade 4 Answer Key Chapter 9 Relate Fractions and Decimals img 68

Question 16.
Classify ∠A.
_____ angle

Answer:
Right Angle

Explanation:
A is the right angle that forms a square corner.

Question 17.
Which angle appears to be obtuse?
∠ _____

Answer:
∠C

Explanation:
C is an obtuse angle that is greater than a right angle and less than a straight angle.

Question 18.
How many different angles are in Figure X?
List them.
Go Math Grade 4 Answer Key Chapter 9 Relate Fractions and Decimals img 69
Type below:
__________

Answer:
4 Angles;
Right Angle = Angle EBC;
Obtuse angle = Angle DBF;
Acute angle = Angle DBE;
Straight angle = Angle ABC.

Explanation:

Question 19.
Vanessa drew the angle at the right and named it ∠TRS. Explain why Vanessa’s name for the angle is incorrect. Write a correct name for the angle.
Go Math Grade 4 Answer Key Chapter 9 Relate Fractions and Decimals img 70
Type below:
__________

Answer:
Vanessa’s name for the angle is incorrect. Because She drew ∠TSR. The two rays R and T have the same endpoint at S called the angle. Also, the TSR is an acute angle that is less than a right angle.

Question 20.
Write the word that describes the part of Figure A.
Go Math Grade 4 Answer Key Chapter 9 Relate Fractions and Decimals img 71
Go Math Grade 4 Answer Key Chapter 9 Relate Fractions and Decimals img 72
\(\overline{B G}\) _________
\(\underset { CD }{ \longleftrightarrow } \) _________
∠FBG _________
\(\overrightarrow{B E}\) _________
∠AGD _________

Answer:
\(\overline{B G}\) Line Segment.
\(\underset { CD }{ \longleftrightarrow } \) Line.
∠FBG Right Angle.
\(\overrightarrow{B E}\) Ray.
∠AGD an acute angle.

Conclusion:

We hope the given data about Go Math Grade 4 Answer Key Chapter 9 Relate Fractions and Decimals PDF help you more during the practice sessions. If you want to ask any doubts, feel free to check HMH Go Math Grade 4 Chapter 9 Answer Key Homework Practice FL.

Go Math Grade 3 Answer Key Chapter 4 Multiplication Facts and Strategies Extra Practice

go-math-grade-3-chapter-4-multiplication-facts-and-strategies-answer-key

Go Math Grade 3 Answer Key Chapter 4 Multiplication Facts and Strategies Extra Practice helps you to improve on the concepts in it. The Extra practice of Go Math Grade 3 Chapter 4 Multiplication Facts and Strategies Answer Key provides you with an elaborate explanation making it easier for you to understand. If you have any doubts or stuck at some point you can always look up to HMH Go Math Grade 3 Multiplication Facts and Strategies Answer Key to seek assistance.

Go Math Grade 3 Chapter 4 Multiplication Facts and Strategies Extra Practice Answer Key

Go Math Grade 3 Chapter 4 Extra Practice includes all the lessons in one place. Learn the easy tricks to solve problems from 3rd Grade Go Math Chapter 4 Multiplication Facts and Strategies Extra Practice. We will not just provide you with the Answers to Go Math Grade 3 Chapter 4 Extra Practice but also the detailed description needed to understand the topic.

Common Core – Page No. 87000

Lessons 4.1–4.2

Find the product.

Question 1.
4 × 2 = ______

Answer:
4 × 2 = 8.

Explanation:
Double 2×2 to get 4×2.
2 x 2 = 4.
Double: 4 + 4 = 8.
4 x 2 = 8.

Question 2.
8 × 5 = ______

Answer:
8 x 5 = 40

Explanation:
Factor 8 is an even number. 4+ 4
5 x 4 = 20.
20 doubled is 40.
8 x 5 = 40.

Question 3.
10 × 7 = ______

Answer:
10 × 7 = 70

Explanation:
A multiple of 10 is any product that has 10 as one of its factors. So, the multiplication of any number with 10 is 10’s of that particular number. The answer is 10 × 7 = 70.

Question 4.
2 × 9 = ______

Answer:
2 x 9 = 18

Explanation:
Double the given number 6 to get the final answer. The answer is 9 + 9 = 18.
2 x 9 = 18.

Question 5.
6
× 1 0
——-
______

Answer:
6 x 10 = 60

Explanation:
Using doubles, we can find a 6 x 10 value. First, multiply the factor with half of 6. So, now we can do 3 x 10 = 30. Now, we can double the value of 3 x 10. That is 30 + 30 = 60. So, the answer for 6 x 10 = 60.

Question 6.
5
× 7
——-
______

Answer:
5 x 7 = 35.

Explanation:

Skip count by 5’s until you say 7 numbers. 5, 10, 15, 20, 25, 30, 35. Now, the count of the number is 7. So, the answer for 5 x 7 is 35.

Question 7.
2
× 1 0
——-
______

Answer:
2 x 10 = 20.

Explanation:
Double 10 to get the answer of 2 x 10.
10 + 10 = 20.
2 x 10 = 20.

Question 8.
4
× 5
——-
______

Answer:
4 x 5 =20

Explanation:
Multiply 2×5 to get the answer for 4×5. Double the answer of 2×5 to get the final answer.
2 x 5 =10.
Double: 10 + 10 = 20.
4 x 5 =20.

Lessons 4.3–4.5

Find the product.

Question 9.
6
× 2
——-
______

Answer:
6 x 2 = 12.

Explanation:
Use doubles to find the answer of 6 x 2.
Multiply 3 x 2 = 6.
Double: 6 + 6 = 12.
The answer for 6 x 2 is 12.

Question 10.
3
× 9
——-
______

Answer:
3 x 9 = 27.

Explanation:

Skip count by 3’s until you say 9 numbers. Write like 3, 6, 9, 12, 15, 18, 21, 24, 27. The answer for 3 x 9 is 27.

Question 11.
7
× 3
——-
______

Answer:
7 x 3 = 21

Explanation:
Write 7 x 3 as 3 x 7 according to the Commutative Law of Multiplication.
Skip count by 3’s until you say 7 numbers. Write like 3, 6, 9, 12, 15, 18, 21. The answer for 3 x 7 is 21. So, 7 x 3 = 21.

Question 12.
8
× 6
——-
______

Answer:
8 × 6 = 48

Explanation:
8 × 6 = (2 x 4) x 6
Use the Associative Property.
8 × 6 = 2 x (4 x 6)
Multiply. 4 × 6
8 × 6 = 2 x 24
Double the product.
8 × 6 = 24 + 24
8 × 6 = 48.

Write one way to break apart the array. Then find the product.

Question 13.
Go Math Grade 3 Answer Key Chapter 4 Multiplication Facts and Strategies Extra Practice Common Core img 1
Type below:
__________

Answer:
36

Explanation:
Draw a line to seperate the columns. Divide the columns with 4 and 5 factors.
The array is 4 x 9 = 4 x (5 + 4)
(4 x 5) + (4 x 4) = 20 + 16 = 36.

Find the product.

Question 14.
5 × 7 = ______

Answer:
5 x 7 = 35

Explanation:
Skip count by 5’s until you say 7 numbers. 5, 10, 15, 20, 25, 30, 35. Now, the count of the number is 7. So, the answer for 5 x 7 is 35.

Question 15.
2 × 6 = ______

Answer:
2 x 6 = 12

Explanation:
Double 6 to get the answer. 6 + 6 = 12.
2 x 6 = 12.

Question 16.
4 × 7 = ______

Answer:
Double 2 x 7 to find 4 x 7.
2 x 7 = 14.
Double: 14 + 14 = 28.
4 x 7 = 28.

Explanation:

Question 17.
8 × 3 = ______

Answer:
8 × 3 = 24

Explanation:
8 × 3 = (2 x 4) x 3
Use the Associative Property.
8 × 3 = 2 x (4 x 3)
Multiply. 4 × 3
8 × 3 = 2 x 12
Double the product.
8 × 3 = 12 + 12
8 × 3 = 24.

Question 18.
Abby has 5 stacks of cards with 7 cards in each stack. How many cards does she have in all?
______ cards

Answer:
35 cards

Explanation:
5 x 7 = 35.
Abby has 35 cards.

Question 18.
Noah has 3 sisters. He gave 6 balloons to each sister. How many balloons did Noah give away in all?
______ balloons

Answer:
18 balloons

Explanation:
3 x 6 = 18.
Noah gave 18 balloons to his sisters.

Common Core – Page No. 88000

Lesson 4.6

Write another way to group the factors. Then find the product.

Question 1.
(3 × 2) × 5 = ______
Explain:
__________

Answer:
3 × (2 × 5)
30

Explanation:
Using Associative Property of Multiplication, we can write (3 × 2) × 5 = 3 × (2 × 5).
Find (3 × 2) × 5. Multiply 3 x 2 = 6. Then, multiply 6 x 5 = 30.
Find 3 x (2 x 5). Multiply 2 x 5 = 10. Then, multiply 3 x 10 = 30.
So, (3 × 2) × 5 = 3 × (2 × 5). The product value is 30.

Question 2.
2 × (5 × 3) = ______
Explain:
__________

Answer:
(2 x 5) x 3
30

Explanation:
Using the Associative Property of Multiplication, we can write 2 × (5 × 3) = (2 x 5) x 3.
Find 2 × (5 × 3). Multiply 5 x 3 = 15. Then, multiply 2 x 15 = 30.
Find (2 x 5) x 3. Multiply 2 x 5 = 10. Then, multiply 10 x 3 = 30.
So, 2 × (5 × 3) = (2 x 5) x 3. The product value is 30.

Question 3.
(1 × 4) × 2 = ______
Explain:
__________

Answer:
1 x (4 x 2)

Explanation:
Using the Associative Property of Multiplication, we can write (1 × 4) × 2 = 1 x (4 x 2).
Find 1 × (4 × 2). Multiply 4 x 2 = 8. Then, multiply 1 x 8 = 8.
Find (1 x 4) x 2. Multiply 1 x 4 = 4. Then, multiply 4 x 2 = 8.
So, (1 × 4) × 2 = 1 x (4 x 2). The product value is 8.

Lesson 4.7

Is the product even or odd?
Write even or odd.
Go Math Grade 3 Answer Key Chapter 4 Multiplication Facts and Strategies Extra Practice Common Core img 2

Question 4.
6 × 6
______

Answer:
Even

Explanation:
6 x 6 = 36. The numbers end with 0, 2, 4, 6, 8 are even numbers. So, 36 is even number. The 6 x 6 is an even number.

Question 5.
2 × 3
______

Answer:
even

Explanation:
Products with 2 as a factor are even.

Question 6.
3 × 9
______

Answer:
odd

Explanation:
The product of two odd numbers is an odd number. The answer is odd. So, 3 x 9 = 27.

Lessons 4.8–4.9

Find the product.

Question 7.
8 × 2 = ______

Answer:
16

Explanation:
Factor 8 is an even number. 4+ 4
2 x 4 = 8.
8 doubled is 16.
8 x 2 = 16.

Question 8.
5 × 9 = ______

Answer:
5 × 9 = 45

Explanation:
The multiplication of 5 × 9 is calculated as Skip-count by 5’s 9 times. You can write as 5, 10, 15, 20, 25, 30, 35, 40, 45. The final answer for 5 × 9 is 45.

Question 9.
______ = 3 × 9

Answer:
27 = 3 x 9

Explanation:
Skip count by 3’s until you say 9 numbers. Write like 3, 6, 9, 12, 15, 18, 21, 24, 27. The answer for 3 x 9 is 27.

Question 10.
4 × 8 = ______

Answer:
2 × 5 = 10

Explanation:
Double 2×8 to get 4×8.
2 x 8 = 16.
16 + 16 = 32.
4 x 8 = 32.

Question 11.
______ = 9 × 4

Answer:
36

Explanation:
9 = 3 + 6
9 × 4 = (3 + 6) x 4
Multiply each addend by 4.
9 × 4 = (3 x 4) + (6 x 4)
Add the products.
9 × 4 = 12 + 24
9 × 4 = 36.

Question 12.
6 × 8 = ______

Answer:
48

Explanation:
Use doubles to find the answer of 6 x 8. Firstly, multiply 3 x 8 = 24. Then, double the value of 3 x 8. 24 + 24 = 48. The answer for 6 x 8 is 48.

Lesson 4.10

Question 13.
Leo has a total of 45¢. He has some dimes and pennies. How many different combinations of dimes and pennies could Leo have? Make a table to solve.
Leo could have ______ combinations of 45¢.

Answer:
Leo could have 4 combinations of 45¢.

Explanation:
1 dime 35 pennies
2 dimes 25 pennies
3 dimes 15 pennies
4 dimes 5 pennies.

Question 13.

Number of Dimes ____1_____ _____2____ ___3______ _____4____
Number of Pennies ____35_____ _____25____ ___15______ ______5___
Total Value 45¢ 45¢ 45¢ 45¢

Conclusion

Above listed are the Comprehensive Solutions for Grade 3 Chapter 4 Multiplication Facts and Strategies. Go Math Grade 3 Answer Key Chapter 4 Multiplication Facts and Strategies Extra Practice is a reliable source to improve on the fundamentals. Make your learning way effective taking help from this page and stand out from the crowd.

Go Math Grade 8 Answer Key Chapter 13 Volume

go-math-grade-8-chapter-13-volume-answer-key

Go Math Grade 8 Answer Key Chapter 8 Volume contains all the topics which help the students to score better marks in the exams. The main aim of providing the Go Math 8th Grade Answer Key for Chapter 13 Volume is to make the students understand the concepts in a simple way. Download Go Math Grade 8 Answer Key Chapter 13 Volume pdf. So, the students who are preparing for 3rd standard exams can check HMH Go Math Solution Key Grade 8 Chapter 13 Volume. Be the first to grab your HMH Go Math Grade 8 Answer Key and read all the questions of maths.

Download Go Math Grade 8 Chapter 13 Volume Answer Key Pdf

The topics covered in Go Math Grade 8 Answer Key Chapter 13 Volume are volume of cone, cylinders, and spheres. Get the clear cut explanations for all the questions here. Refer to Go Math Grade 8 Answer Key Chapter 13 Volume to get the highest marks in the exam. Learn and practice the maths in the best way using Go Math Grade 8 Answer Key Chapter 13 Volume. Check the topics before you start your preparation from the below sections.

Lesson 1: Volume of Cylinders

Lesson 2: Volume of Cones

Lesson 3: Volume of Spheres

Model Quiz :

Review :

Guided Practice – Volume of Cylinders – Page No. 402

Question 1.
Vocabulary Describe the bases of a cylinder.
Type below:
____________

Answer:
The ends of a cylinder are the bases of the cylinder of the two flat surfaces.

Question 2.
Figure 1 shows a view from above of inch cubes on the bottom of a cylinder. Figure 2 shows the highest stack of cubes that will fit inside the cylinder. Estimate the volume of the cylinder. Explain your reasoning.
Go Math Grade 8 Answer Key Chapter 13 Volume Lesson 1: Volume of Cylinders img 1
________ in3

Answer:

427in³

Explanation:
Number of inch cubes that fit in the base of the cylinder = 61
Number of inch cubes that fit in the length of the cylinder = 7
Volume of cylinder = basearea x height
V= 61 x 7
V = 427 cubic units.
Volume of each cube = 1 in³
Volume of cylinder = 427in³

Question 3.
Find the volume of the cylinder to the nearest tenth. Use 3.14 for π.
Go Math Grade 8 Answer Key Chapter 13 Volume Lesson 1: Volume of Cylinders img 2
The volume of the cylinder is approximately _____ m3.
________ m3

Answer:
The volume of the cylinder is approximately 1695.6 m3.
1695.6m³

Explanation:
V = πr²h
V = π . 6² . 15
V = 3.14 × 36 × 15
V = 1695.6m³

Question 4.
A Japanese odaiko is a very large drum that is made by hollowing out a section of a tree trunk. A museum in Takayama City has three odaikos of similar size carved from a single tree trunk. The largest measures about 2.7 meters in both diameter and length, and weighs about 4.5 metric tons. Using the volume formula for a cylinder, approximate the volume of the drum to the nearest tenth.
The radius of the drum is about _____ m.
The volume of the drum is about _____ m3.
The radius of the drum is about ___________ m
The volume of the drum is about ___________ m3

Answer:
The radius of the drum is about 1.35 m.
The volume of the drum is about 15.5 m3.

Explanation:
Diameter of base of drum = 2.7 m
The radius of the base of the drum = 2.7/2
R = 1.35 m
The volume of cylinder = πr²h
Height (h) = 2.7 m
Radius (R) = 1.35 m
Volume = 3.14 × (1.35) × 2.7
V = 15.4511 m³
V = 15.5 m³

ESSENTIAL QUESTION CHECK-IN

Question 5.
How do you find the volume of a cylinder? Describe which measurements of a cylinder you need to know.
Type below:
____________

Answer:
The volume of the cylinder is = πr²h

Explanation:
We need to find the radius of the base, r, and the height of the cylinder, h.
The volume of the cylinder is = πr²h

13.1 Independent Practice – Volume of Cylinders – Page No. 403

Find the volume of each figure. Round your answers to the nearest tenth if necessary. Use 3.14 for π.

Question 6.
Go Math Grade 8 Answer Key Chapter 13 Volume Lesson 1: Volume of Cylinders img 3
_________ cm3

Answer:
569.9 cm³

Explanation:
Radius of base = 11 cm
Height of cylinder = 1.5 cm
The volume of cylinder = πr²h
V = 3.14 × (11)² × 1.5
V = 569.91
V = 569.9 cm³

Question 7.
Go Math Grade 8 Answer Key Chapter 13 Volume Lesson 1: Volume of Cylinders img 4
_________ in3

Answer:
1205.8 in³

Explanation:
Radius of base = 4 in
Height of cylinder = 24 in
The volume of cylinder = πr²h
V = 3.14 × (4)² × 24
V = 1205.76
V = 1205.8 in³

Question 8.
Go Math Grade 8 Answer Key Chapter 13 Volume Lesson 1: Volume of Cylinders img 5
_________ m3

Answer:
1256 m³

Explanation:
Radius of base = 5 m
Height of cylinder = 16 m
The volume of cylinder = πr²h
V = 3.14 × (5)² × 16
V = 1256
V = 1256 m³

Question 9.
Go Math Grade 8 Answer Key Chapter 13 Volume Lesson 1: Volume of Cylinders img 6
_________ in3

Answer:
942 in³

Explanation:
Diameter of base = 10 in
Radius of base = 5 in
Height of cylinder = 12 in
The volume of cylinder = πr²h
V = 3.14 × (5)² × 12
V = 942 in³

Question 10.
A cylinder has a radius of 4 centimeters and a height of 40 centimeters.
_________ cm3

Answer:
2009.6 cm³

Explanation:
Radius of base = 4 cm
Height of cylinder = 40 cm
The volume of cylinder = πr²h
V = 3.14 × (4)² × 40
V = 2009.6 cm³

Question 11.
A cylinder has a radius of 8 meters and a height of 4 meters.
_________ m3

Answer:
803.8 m³

Explanation:
The radius of base = 8 m
Height of cylinder = 4 m
The volume of cylinder = πr²h
V = 3.14 × (8)² × 4
V = 803.84 m³
V = 803.8 m³

Round your answer to the nearest tenth, if necessary. Use 3.14 for π.

Question 12.
The cylindrical Giant Ocean Tank at the New England Aquarium in Boston is 24 feet deep and has a radius of 18.8 feet. Find the volume of the tank.
_________ ft3

Answer:
26635.2 ft³

Explanation:
Base radius of the tank = 18.8 ft
Depth of the tank in the ocean = 24 ft
The volume of cylinder = πr²h
V = 3.14 × (18.8)² × 24
V = 3.14 × 354.44 × 24
V = 26635.2384 ft³
V = 26635.2 ft³

Question 13.
A standard-size bass drum has a diameter of 22 inches and is 18 inches deep. Find the volume of this drum.
_________ in3

Answer:
6838.9 in³

Explanation:
Base diameter of drum = 22 in
Base radius of the drum = 22/2 = 11 in
Depth of the bass drum = 18 in
Volume of the bass drum = πr²h
V = 3.14 × (11)² × 18
V = 3.14 × 121 × 18
V = 6838.92 in³
V = 6838.9 in³

Question 14.
Grain is stored in cylindrical structures called silos. Find the volume of a silo with a diameter of 11.1 feet and a height of 20 feet.
_________ ft3

Answer:
1934.4 ft³

Explanation:
Base diameter of silo = 11.1 ft
Base radius of the silo = 11.1/2 = 5.55 ft
Depth of the silo = 20 ft
Volume of the silo = πr²h
V = 3.14 × (5.55)² × 18
V = 3.14 × 30.8025 × 18
V = 1934.397 ft³
V = 1934.4 ft³

Question 15.
The Frank Erwin Center, or “The Drum,” at the University of Texas in Austin can be approximated by a cylinder that is 120 meters in diameter and 30 meters in height. Find its volume.
_________ m3

Answer:
339120 m³

Explanation:
Base diameter of the drum = 120 m
Base radius of the drum = 120/2 = 60 m
Height of the drum = 30 m
Volume of the drum = πr²h
V = 3.14 × (60)² × 30
V = 3.14 × 3600 × 30
V = 339120 m³

Volume of Cylinders – Page No. 404

Question 16.
A barrel of crude oil contains about 5.61 cubic feet of oil. How many barrels of oil are contained in 1 mile (5280 feet) of a pipeline that has an inside diameter of 6 inches and is completely filled with oil? How much is “1 mile” of oil in this pipeline worth at a price of $100 per barrel?
__________ barrels
$ __________

Answer:
184.7 barrels
$18470

Explanation:
Volume of barrel = 5.61 cubic feet
Length of the pipe = 1 mile = 5280 feet
Diameter of the pipe = 6 inches = 0.5 feet
Radius of the pipe = 6/2 inches = 3 inches = 0.25 feet
Volume of oil in the pipe = πr²h = 3.14 × (0.25)² × 5280 = 1036.2 cubic feet
Number of barrels in the pipe = 1036.2/5.61 = 184.7 barrels
Cost of one barrel = $100
Cost of 184.7 barrels =184.7 × $100 = $18470

Question 17.
A pan for baking French bread is shaped like half a cylinder. It is 12 inches long and 3.5 inches in diameter. What is the volume of uncooked dough that would fill this pan?
Go Math Grade 8 Answer Key Chapter 13 Volume Lesson 1: Volume of Cylinders img 7
_________ in3

Answer:
57.697 in³

Explanation:
The length of the pan = 12 in
The diameter of the pan = 3.5 in
Radius = 3.5/2 = 1.75 in
The volume of uncooked dough = Half the volume of the full cylinder of the above dimensions.
= (πr²h)/2 = (3.14 × (1.75)² × 12)/2 = 115.395/2 = 57.697 in³

FOCUS ON HIGHER ORDER THINKING

Question 18.
Explain the Error A student said the volume of a cylinder with a 3-inch diameter is two times the volume of a cylinder with the same height and a 1.5-inch radius. What is the error?
Type below:
_______________

Answer:
The volume of the cylinder of 3 in is four times the volume of the new cylinder of radius 1.5 in

Explanation:
The volume of a cylinder is directly proportional to the square of the radius of the cylinder. The volume does not depend on the radius linearly.
Volume = πr²h
V1 = π(3)²h
V2 = π(1.5)²h
V1/V2 = (π(3)²h)/(π(1.5)²h)
V1/V2 = 4
V1 = 4V2

Question 19.
Communicate Mathematical Ideas Explain how you can find the height of a cylinder if you know the diameter and the volume. Include an example with your explanation.
Type below:
_______________

Answer:
Let the diameter be D.
Radius r = D/2
Volume = πr²h
Volume = π(D/2)²h
V = π((D)²/4)h
h = 4V/π(D)²
To find the height of a cylinder with diameter D = 2 m
Let the volume be 10 m³
h = 4V/π(D)²
h = (4 × 10)/(3.14 × 2²)
h = 3.18 m³

Question 20.
Analyze Relationships Cylinder A has a radius of 6 centimeters. Cylinder B has the same height and a radius half as long as cylinder A. What fraction of the volume of cylinder A is the volume of cylinder B? Explain.
Fraction: \(\frac{□}{□}\)

Answer:
\(\frac{VA}{4}\)

Explanation:
rA = 6 cm
rB = half of the radius of cylinder A = 3 cm
hA = hB
VA = πrA²h
VB = πrB²h
VA/VB = (πrA²h)/(πrB²h)
VA/VB = 6²/3² = 36/9 = 4
Thus VB = VA/4

Guided Practice – Volume of Cones – Page No. 408

Question 1.
The area of the base of a cylinder is 45 square inches and its height is 10 inches. A cone has the same area for its base and the same height. What is the volume of the cone?
Go Math Grade 8 Answer Key Chapter 13 Volume Lesson 2: Volume of Cones img 8
The volume of the cone is _____ in3.
_________ in3

Answer:
150 in³

Explanation:
In the question, the area of the base of the cylinder, B = 45 in²
Height of the cylinder, h = 10 inch
Volume of the cylinder, V cylinder = B × h = 45 × 10 = 450 inch³
Volume of the cone, V Cone = 1/3 V cylinder
=1/3(450 inch) = 150 inch³
So, the volume of the cone is
Vcone = 150 in³

Question 2.
A cone and a cylinder have congruent height and bases. The volume of the cone is 18 m3.What is the volume of the cylinder? Explain.
_________ m3

Answer:
54 m3

Explanation:
The volume of the cone is 18 m3.
Vcone = 1/3 Vcylinder
Vcylinder = 3Vcone
Vcylinder = 3.18
Vcylinder = 54 m3

Find the volume of each cone. Round your answer to the nearest tenth if necessary. Use 3.14 for π.

Question 3.
Go Math Grade 8 Answer Key Chapter 13 Volume Lesson 2: Volume of Cones img 9
_________ ft3

Answer:
65.94 ft³

Explanation:
the diameter of the cone is 6ft.
so, the radius of the cone is 3ft.
the height of the cone is 7ft.
the volume of the cone = 1/3 × πr²h = 1/3 × 3.14 × 3² × 7 = 65.94 ft³

Question 4.
Go Math Grade 8 Answer Key Chapter 13 Volume Lesson 2: Volume of Cones img 10
_________ in3

Answer:
113982in³

Explanation:
The radius is 33inch and the height is 100 inch
Volume of the cone = 1/3 × πr²h = 1/3 × π(33)²100 = 113982in³

Question 5.
Gretchen made a paper cone to hold a gift for a friend. The paper cone was 15 inches high and had a radius of 3 inches. Find the volume of the paper cone to the nearest tenth. Use 3.14 for π.
_________ in3

Answer:
141.3in³

Explanation:
the radius of the cone is 3inch and the height of the cone is 15inch.
Volume of the cone = 1/3 × πr²h = 1/3 × π(3)² × 15 = 141.3in³

Question 6.
A cone-shaped building is commonly used to store sand. What would be the volume of a cone-shaped building with a diameter of 50 meters and a height of 20 meters? Round your answer to the nearest tenth. Use 3.14 for π.
_________ m3

Answer:
13083.33 m³

Explanation:
The diameter of the cone is 50 meters. So, the radius of the cone is 25 meters. The height of the cone is 20 meters.
Volume of the cone = 1/3 × πr²h = 1/3 × π(25)² × 20 = 13083.33 m³

ESSENTIAL QUESTION CHECK-IN

Question 7.
How do you find the volume of a cone?
Type below:
____________

Answer:
V cone = 1/3 V cylinder
V cone = 1/3 πr²h

13.2 Independent Practice – Volume of Cones – Page No. 409

Find the volume of each cone. Round your answers to the nearest tenth if necessary. Use 3.14 for π.

Question 8.
Go Math Grade 8 Answer Key Chapter 13 Volume Lesson 2: Volume of Cones img 11
_________ mm3

Answer:
410.3 mm³

Explanation:
Radius r = 7 mm
height = 8 mm
Volume of cone = 1/3 πr²h
Volume = 1/3(3.14)(7)²(8)
Volume = 410.29 mm³
Volume = 410.3 mm³

Question 9.
Go Math Grade 8 Answer Key Chapter 13 Volume Lesson 2: Volume of Cones img 12
_________ in3

Answer:
25.1 in3

Explanation:
Radius r = 2 in
Height = 6 in
Volume of cone = 1/3 πr²h
Volume = 1/3(3.14)(2)²(6)
Volume = 25.12 in3
Volume = 25.1 in3

Question 10.
A cone has a diameter of 6 centimeters and a height of 11.5 centimeters.
_________ cm3

Answer:
108.3 cm3

Explanation:
Diameter of base = 6 cm
Radius = 6/2 cm = 3 cm
Height = 11.5 cm
Volume of cone = 1/3 πr²h
Volume = 1/3(3.14)(3)² (11.5)
Volume = 108.33 cm3
Volume = 108.3 cm3

Question 11.
A cone has a radius of 3 meters and a height of 10 meters.
_________ m3

Answer:
94.2 m3

Explanation:
Radius r = 3 m
Height = 10 m
Volume of cone = 1/3 πr²h
Volume = 1/3(3.14)(3)²(10)
Volume = 94.2 m3

Round your answers to the nearest tenth if necessary. Use 3.14 for π.

Question 12.
Antonio is making mini waffle cones. Each waffle cone is 3 inches high and has a radius of \(\frac{3}{4}\) inch. What is the volume of a waffle cone?
_________ in3

Answer:
1.8 in3

Explanation:
Radius = 3/4 in
Radius r = 0.75 in
Height = 3 in
Volume of each waffle cone = 1/3 πr²h
Volume = 1/3 (3.14) (0.75)² (3)
Volume = 1.76625 in3
Volume = 1.8 in3

Question 13.
A snack bar sells popcorn in cone-shaped containers. One container has a diameter of 8 inches and a height of 10 inches. How many cubic inches of popcorn does the container hold?
_________ in3

Answer:
167.5 in3

Explanation:
Diameter of base = 8 in
Radius = 8/2 in = 4 in
Height = 10 in
Volume of cone = 1/3 πr²h
Volume = 1/3 (3.14) (4)² (10)
Volume = 167.466 in3
Volume = 167.5 in3

Question 14.
A volcanic cone has a diameter of 300 meters and a height of 150 meters. What is the volume of the cone?
_________ m3

Answer:
3534291.7 m3

Explanation:
Diameter of base = 300 m
Radius = 300/2 m = 150 m
Height = 150 m
Volume of cone = 1/3 πr²h
Volume = 1/3 (3.14) (150)² (150)
Volume = 3534291.735 m3
Volume = 3534291.7 m3

Question 15.
Multistep Orange traffic cones come in a variety of sizes. Approximate the volume, in cubic inches, of a traffic cone that has a height of 2 feet and a diameter of 10 inches. Use 3.14 for π.
_________ in3

Answer:
628 in³

Explanation:
The radius of the cone is Diameter/2 = 10/2 = 5
The height of the cone is 2 ft = 2 . 12 = 24 in
Vcone = 1/3 πr²h
Vcone = 1/3 (3.14) (5)² (24)
Vcone = 628 in³

Find the missing measure for each cone. Round your answers to the nearest tenth if necessary. Use 3.14 for π.

Question 16.
radius = _______
height = 6 in.
volume = 100.48 in3
_________ in.

Answer:
radius = 4 in.
4 in.

Explanation:
Let radius be R.
Height = 6 in
Volume = 100.4 in
Volume of cone = 1/3 πr²h
√(3v/hπ) = R
√((3 × 100.48)/(18.84)) = R
√(301.44/18.84) = R
R = √(16)
R = 4 in

Question 17.
diameter = 6 cm
height = _______
volume = 56.52 cm3
_______ cm

Answer:
height = 6 cm
h = 6 cm

Explanation:
Let height be h
Diameter = 6 cm
Radius = 6/2 = 3 cm
Volume = 56.52 cm
Volume of cone = 1/3 πr²h
(3V/r²h) = h
(3 × 56.52)/(3² × 3.14) = h
169.56/28.26 = h
h = 6 cm

Question 18.
The diameter of a cone-shaped container is 4 inches, and its height is 6 inches. How much greater is the volume of a cylinder-shaped container with the same diameter and height? Round your answer to the nearest hundredth. Use 3.14 for π.
Type below:
____________

Answer:
The volume of the cylinder is 50.24 in³ greater than the volume of the cone.

Explanation:
The diameter of a cone, d = 4 inch
radius of a cone, r = d/2 = 4/2 = 2 inches
height of a cone, h = 6 inches.
So, volume of a cone, V cone = 1/3 πr²h
= 1/3 (3.14) (2)² (6)
= 25.12 in³
And volume of a cylinder with same diameter and height,
V cylinder = πr²h = (3.14) (2)² (6) = 75.36 in³
The volume of the cylinder is 50.24 in³ greater than the volume of the cone.

FOCUS ON HIGHER ORDER THINKING – Volume of Cones – Page No. 410

Question 19.
Alex wants to know the volume of sand in an hourglass. When all the sand is in the bottom, he stands a ruler up beside the hourglass and estimates the height of the cone of sand.
a. What else does he need to measure to find the volume of sand?
____________

Answer:
To find the volume of the sand, he needs to measure the radius of the base of the hourglass.

Question 19.
b. Make a Conjecture If the volume of sand is increasing at a constant rate, is the height increasing at a constant rate? Explain.
____________

Answer:
The volume of the cone is linearly proportional to the height of the cone. Therefore, if the volume is increasing at a constant rate, the height is also increasing at a constant rate.

Question 20.
Problem Solving The diameter of a cone is x cm, the height is 18 cm, and the volume is 301.44 cm3. What is x? Use 3.14 for π.
________ cm

Answer:
8 cm

Explanation:
V cone = 1/3 πr²h
301.44 = 1/3 . 3.14 . r² . 18
r² = 904.32/56.52
r² = 16
r = 4 cm
The diameter of the circle is twice its radius, therefore
x = 2 . r
x = 2 . 4
x = 8 cm

Question 21.
Analyze Relationships A cone has a radius of 1 foot and a height of 2 feet. How many cones of liquid would it take to fill a cylinder with a diameter of 2 feet and a height of 2 feet? Explain.
________ cones

Answer:
3 cones

Explanation:
The diameter of the base of the cylinder is 2 feet, which means that its radius is 1 foot. Its height is 2 feet. The volume of this cylinder is
V cylinder = πr²h
V cylinder = (3.14) (1)² (2)
V cylinder = 6.28
The radius of the cone is 1 foot and the height of the cone is 2 feet. The volume of the cone is:
V cone = 1/3 πr²h
V cone = 1/3 (3.14) (1)² (2)
V cone = 1/3 × 6.28
V cone = 1/3 . V cylinder
V cone = 2.09
It would take 3 cones of liquid to fill the cylinder.

Question 22.
Critique Reasoning Herb knows that the volume of a cone is one third that of a cylinder with the same base and height. He reasons that a cone with the same height as a given cylinder but 3 times the radius should therefore have the same volume as the cylinder, since \(\frac{1}{3}\) ⋅ 3 = 1. Is Herb correct? Explain.
____________

Answer:
The volume of the given cylinder is V cylinder = πr²h
The volume of the cone with the same height h as a given cylinder but 3 times the radius r is
V cone = 1/3 π(3r)²h
V cone = 3 πr²h
V cone = 3 V cylinder
As we can see, Herb is not correct. The volume of the cone is not equal to the volume of the cylinder. But it is three times the volume of the cylinder.

Guided Practice – Volume of Spheres – Page No. 414

Question 1.
Vocabulary A sphere is a three-dimensional figure with all points _____ from the center.
Type below:
____________

Answer:
A sphere is a three-dimensional figure with all points at equal distance from the center.

Question 2.
Vocabulary The _____ is the distance from the center of a sphere to a point on the sphere.
Type below:
____________

Answer:
radius

Explanation:
The radius is the distance from the center f the sphere to a point on the sphere

Find the volume of each sphere. Round your answers to the nearest tenth if necessary. Use 3.14 for π.

Question 3.
Go Math Grade 8 Answer Key Chapter 13 Volume Lesson 3: Volume of Spheres img 13
_______ in3

Answer:
4.12 in3

Explanation:
V = 4/3πr³
V = 4/3 (3.14) (1)³
V = 4.12 in3

Question 4.
Go Math Grade 8 Answer Key Chapter 13 Volume Lesson 3: Volume of Spheres img 14
_______ cm3

Answer:
4186.7 cm³

Explanation:
Diameter = 20 cm
Radius r = 20/2 = 10 cm
Volume of sphere = 4/3πr³
Volume = 4/3 (3.14) (10)³
Volume = 4186.66 cm³
Volume = 4186.7 cm³

Question 5.
A sphere has a radius of 1.5 feet.
_______ ft3

Answer:
14.1 ft³

Explanation:
Radius r = 1.5 ft
The volume of sphere = 4/3πr³
Volume = 4/3 (3.14) (1.5)³
Volume = 14.13 ft³
Volume = 14.1 ft³

Question 6.
A sphere has a diameter of 2 yards.
_______ yd3

Answer:
4.2 yd³

Explanation:
Diameter = 2 yards
Radius r = 2/2 yards
Radius r = 1 yd
Volume of sphere = 4/3πr³
Volume = 4/3 (3.14) (1)³
Volume = 4.1866 yd³
Volume = 4.2 yd³

Question 7.
A baseball has a diameter of 2.9 inches. Find the volume of the baseball. Round your answer to the nearest tenth if necessary. Use 3.14 for π.
_______ in3

Answer:
12.8 in³

Explanation:
Diameter of baseball = 2.9 in
Radius r = 2.9/2 in
Radius of baseball = 1.45 in
The volume of sphere = 4/3πr³
Volume = 4/3 (3.14) (1.45)³
Volume = 12.763 in³
Volume = 12.8 in³

Question 8.
A basketball has a radius of 4.7 inches. What is its volume to the nearest cubic inch. Use 3.14 for π.
_______ in3

Answer:
1304 in³

Explanation:
Radius of baseball = 4.7 in
The volume of sphere = 4/3πr³
Volume = 4/3 (3.14) (4.7)³
Volume = 1304.0168 in³
Volume = 1304 in³

Question 9.
A company is deciding whether to package a ball in a cubic box or a cylindrical box. In either case, the ball will touch the bottom, top, and sides.
Go Math Grade 8 Answer Key Chapter 13 Volume Go Math Grade 8 Answer Key Chapter 13 Volume img 15 img 15
a. What portion of the space inside the cylindrical box is empty? Explain.
Type below:
_______________

Answer:
The volume of the cylinder is V cylinder = πr²h
Since the ball touches the bottom, top, and sides, then the height of the cylinder will be equal to 2r.
V cylinder = πr²(2r) = 2πr³
On the other hand, the volume of the sphere is
V sphere = 4/3 πr³
The volume of the empty space inside the cylindrical box is found by subtracting the volume of the sphere from the volume of the cylinder
V cylinder – V sphere = 2πr³ – 4/3 πr³
= (2 – 4/3)πr³
= 2/3πr³

Question 9.
b. Find an expression for the volume of the cubic box.
Type below:
_______________

Answer:
The volume of a cube with side a is V cube = a³
Since the ball touches the bottom, top, and sides, then the side of the cube will be equal to 2r.
V cube = (2r)³
V cube = 8r³

Question 9.
c. About what portion of the space inside the cubic box is empty? Explain
Type below:
_______________

Answer:
The volume of the empty space inside the cubical box is found by subtracting the volume of the sphere from the volume of the cube:
V cube – V sphere = 8r³ – 4/3 πr³
= (8 – 4/3π)r³
= (8 – 4.2)r³
= 3.8r³

ESSENTIAL QUESTION CHECK-IN

Question 10.
Explain the steps you use to find the volume of a sphere.
Type below:
_______________

Answer:
Step 1: The radius of the sphere is found out.
Step 2: The volume of the sphere is 4/3 πr³; where R is the radius.
Step 3: Put the value of radius in the equation of volume.
Step 4: Calculate the volume.

13.3 Independent Practice – Volume of Spheres – Page No. 415

Find the volume of each sphere. Round your answers to the nearest tenth if necessary. Use 3.14 for π.

Question 11.
radius of 3.1 meters
_______ m3

Answer:
124.7 m³

Explanation:
The volume of the sphere with a radius of 3.1 meter is 4/3 πr³
V = 4/3 . (3.14) . (3.1)³
V = 124.7 m³

Question 12.
diameter of 18 inches
_______ in3

Answer:
3052.1 in³

Explanation:
The diameter of the sphere is 18 inches, which means that its radius is 9 inches. The volume of the sphere is
V = 4/3 πr³
V = 4/3 . (3.14) . (9)³
V = 3052.08 in³
V = 3052.1 in³

Question 13.
r = 6 in.
_______ in3

Answer:
904.3 in³

Explanation:
The volume of the sphere with a radius of 6 inches is
V = 4/3 πr³
V = 4/3 (3.14) (6)³
V = 904.32
V = 904.3 in³

Question 14.
d = 36 m
_______ m3

Answer:
24416.6 m³

Explanation:
d = 36 m
radius r = 36/2 = 18 m
Volume = 4/3 πr³
= 4/3 (3.14) (18)³
= 24416.64
Volume = 24416.6 m³

Question 15.
Go Math Grade 8 Answer Key Chapter 13 Volume Lesson 3: Volume of Spheres img 16
_______ cm3

Answer:
5572.5 cm³

Explanation:
The volume of the sphere with a radius of 11 centimeters is
V = 4/3 πr³
V = 4/3 (3.14) (11)³
V = 5572.5 cm³

Question 16.
Go Math Grade 8 Answer Key Chapter 13 Volume Lesson 3: Volume of Spheres img 17
_______ ft3

Answer:
8.2 feet³

Explanation:
The diameter of the sphere is 2.5 feet, which means that its radius is 1.25 feet. The volume of the sphere is
V = 4/3 πr³
V = 4/3 . (3.14) . (1.25)³
V = 8.2 feet³

The eggs of birds and other animals come in many different shapes and sizes. Eggs often have a shape that is nearly spherical. When this is true, you can use the formula for a sphere to find their volume.

Question 17.
The green turtle lays eggs that are approximately spherical with an average diameter of 4.5 centimeters. Each turtle lays an average of 113 eggs at one time. Find the total volume of these eggs, to the nearest cubic centimeter.
_______ cm3

Answer:
5389 cm³

Explanation:
The diameter of an egg (sphere) is 4.5 centimeters, which means that its radius is 2.25 centimeters. The volume of a single egg is
V = 4/3 πr³
V = 4/3 (3.14) (2.25)³
V = 47.68875 cm³
Therefore, the total volume of 113 eggs is
113 . V = 113(47.68875)
= 5388.82875
= 5389 cm³

Question 18.
Hummingbirds lay eggs that are nearly spherical and about 1 centimeter in diameter. Find the volume of an egg. Round your answer to the nearest tenth.
_______ cm3

Answer:
0.5 cm³

Explanation:
The diameter of an egg (sphere) is 1 centimeter, which means that its radius is 0.5 centimeters. The volume of a single egg is
V = 4/3 πr³
V = 4/3 (3.14) (0.5)³
V = 0.5 cm³

Question 19.
Fossilized spherical eggs of dinosaurs called titanosaurid sauropods were found in Patagonia. These eggs were 15 centimeters in diameter. Find the volume of an egg. Round your answer to the nearest tenth.
_______ cm3

Answer:
1766.25 cm³

Explanation:
Diameter of an egg = 15 cm
Its radius = 15/2 = 7.5 cm
Volume = 4/3 πr³
V = 4/3 (3.14) (7.5)³
V = 1766.25 cm³

Question 20.
Persevere in Problem Solving An ostrich egg has about the same volume as a sphere with a diameter of 5 inches. If the eggshell is about \(\frac{1}{12}\) inch thick, find the volume of just the shell, not including the interior of the egg. Round your answer to the nearest tenth.
_______ in3

Answer:
6.8 in³

Explanation:
Diameter including the eggshell
= 5 + (2 . 1/2)
= (5 + 1/6) in
= 5.166 in
Radius including egg shell = 5.166/2 = 2.5833 in
Volume = 4/3 πr³
Volume = 4/3 (3.14) (2.5833)³
=72.176 in³
Volume with shell = 72.2 in³
Radius excluding shell = 5/2 = 2.5 in
Volume = 4/3 (3.14) (2.5)³
= 65.416 in³
Volume (without shell) = 65.4 in³
Volume of shell = Total volume – Inner Volume
= 72.2 – 65.4
= 6.8 in³

Question 21.
Multistep Write the steps you would use to find a formula for the volume of the figure at right. Then write the formula.
Go Math Grade 8 Answer Key Chapter 13 Volume Lesson 3: Volume of Spheres img 18
Type below:
_____________

Answer:
5/3πr³

Explanation:
The radius of hemisphere = r
Radius of cylinder = r
Height of cylinder = r
Step 1: Find the formula for the volume of a hemisphere
The volume of hemisphere = 4/3 π/2 r³
= 2/3πr³
Step 2: Find the formula for the volume of a cylinder
The volume of cylinder = πr²h
=πr³
Step 3: Add both the volume expressions:
Total volume = 2/3πr³ + πr³
= 5/3πr³

Volume of Spheres – Page No. 416

Question 22.
Critical Thinking Explain what happens to the volume of a sphere if you double the radius.
Type below:
_____________

Answer:
Let Radius = r
Volume V1 = 4/3πr³
Radius = 2r
Volume V2 = 4/3π(2r)³
= 8 . 4/3πr³
= 8 V1
= 8(initial volume)
By doubling the radius of sphere we make the voulme 8 times the intial value.

Question 23.
Multistep A cylindrical can of tennis balls holds a stack of three balls so that they touch the can at the top, bottom, and sides. The radius of each ball is 1.25 inches. Find the volume inside the can that is not taken up by the three tennis balls.
Go Math Grade 8 Answer Key Chapter 13 Volume Lesson 3: Volume of Spheres img 19
_______ in3

Answer:
12.3 in³

Explanation:
Radius of the ball = 1.25 inch
Height of the cylinder = (2 × 1.25) × 3
= (2.5) × 3
= 7.5 in
radius of base of cylinder = 1.25 in.
Volume of cylinder = πr²h
= (3.14) (1.25)² (7.5)
= 36.7968
= 36.8 in³
Volume of a ball (all three) = 3 × 4/3πr³
= 4 (3.14) (1.25)³
= 24.53125 in³
= 24.5 in³
Volume of empty space = Volume of cylinder – Volume of ball
= 36.8 – 24.5 = 12.3 in³

FOCUS ON HIGHER ORDER THINKING

Question 24.
Critique Reasoning A sphere has a radius of 4 inches, and a cube-shaped box has an edge length of 7.5 inches. J.D. says the box has a greater volume, so the sphere will fit in the box. Is he correct? Explain.
_____________

Answer:
The volume of sphere = 4/3πr³
= 4/3 (3.14) (4)³
= 267.9466
= 268
The volume of cube = (7.5)³
= 421.875
=421.9
The volume of cube > Volume of a sphere
But the base of the cube has an area of (7.5 × 7.5) = 56.25 while the cross-action area of the sphere.
πr² = (3.14) (4)² = 50.24
The cross-section area of the cube is less than that of a sphere. thus J.D. is wrong and the ball (sphere) will not fit in the cube.

Question 25.
Critical Thinking Which would hold the most water: a bowl in the shape of a hemisphere with radius r, a cylindrical glass with radius r and height r, or a cone-shaped drinking cup with radius r and height r? Explain.
_____________

Answer:
The volume of a sphere with radius r is
V sphere = 4/3πr³
Therefore, the volume of a hemisphere is
V hemisphere = V sphere/2
V hemisphere = 2/3πr³
The volume of a cylinder with radius r and height r is
V cylinder = πr²h
V cylinder = πr³
The volume of a cone with radius r and height r is
V cone = 1/3πr²h
V cone = 1/3πr³
V cone < V hemisphere < V cylinder
Therefore, the cylindrical glass with radius r and height r will hold the most water.

Question 26.
Analyze Relationships Hari has models of a sphere, a cylinder, and a cone. The sphere’s diameter and the cylinder’s height are the same, 2r. The cylinder has radius r. The cone has diameter 2r and height 2r. Compare the volumes of the cone and the sphere to the volume of the cylinder.
Type below:
_____________

Answer:
Radius of sphere = 2r/2 = r
Volume of sphere = 4/3πr³
Radius of cylinder = r
Height of cylinder = 2r
volume of cylinder = πr²(2r)
V cylinder = 2πr³
Radius of cone = 2r/2 = r
Height of cone = 2r
Volume of cone = 1/3 πr²(2r)
V cone = 2/3πr³
Volume of cylinder > Volume of sphere > Volume of cone
2πr³ > 4/3πr³ > 2/3πr³

Question 27.
A spherical helium balloon that is 8 feet in diameter can lift about 17 pounds. What does the diameter of a balloon need to be to lift a person who weighs 136 pounds? Explain.
_______ feet

Answer:
Diameter of ballon = 8 ft
Weight it could lift = 17 pound
Volume = 4/3 π(8/2)³
= 4³(4π/3)
4³/x(4π/3) = 17/36
1/x = 1/8 × 3/4π × 1/48
x = 4π/3 . 4³ . 2³
x = 4/3. π . 8³
The volume of ballon which can lift 136 pounds is equal to 4/3. π . 8³
The radius of that ballon = 8ft
Diameter = 8 . 2 = 16 ft

Ready to Go On ? – Model Quiz – Page No. 417

13.1 Volume of Cylinders

Find the volume of each cylinder. Round your answers to the nearest tenth if necessary. Use 3.14 for π.

Question 1.
Go Math Grade 8 Answer Key Chapter 13 Volume Lesson 3: Model Quiz img 20
_______ ft3

Answer:
904.8 ft³

Explanation:
Radius of base = 6 ft
Height of cylinder = 8 ft
The volume of cylinder = πr²h
Volume = (3.14) (6)² (8)
Volume = 904.77 ft³
Volume = 904.8 ft³

Question 2.
A can of juice has a radius of 4 inches and a height of 7 inches. What is the volume of the can?
_______ in3

Answer:
351.7 in³

Explanation:
Radius if cylindrical can = 4 in
Height of cylindrical can = 7 in
The volume of cylinder = πr²h
Volume = (3.14) (4)² (7)
Volume = 351.68 in³
Volume = 351.7 in³

13.2 Volume of Cones

Find the volume of each cone. Round your answers to the nearest tenth if necessary. Use 3.14 for π.

Question 3.
Go Math Grade 8 Answer Key Chapter 13 Volume Lesson 3: Model Quiz img 21
_______ cm3

Answer:
565.2 cm³

Explanation:
Radius of base of cone = 6 cm
Height of cone = 15 cm
Volume of cone = 1/3πr²h
Volume = 1/3 (3.14) (4)² (7)
Volume = 565.2 cm³

Question 4.
Go Math Grade 8 Answer Key Chapter 13 Volume Lesson 3: Model Quiz img 22
_______ in3

Answer:
3014.4 in³

Explanation:
The radius of the base of cone = 12 in
Height of cone = 20 in
The volume of cone = 1/3πr²h
Volume = 1/3 (3.14) (12)² (20)
Volume = 3014.4 in³

13.3 Volume of Spheres

Find the volume of each sphere. Round your answers to the nearest tenth if necessary. Use 3.14 for π.

Question 5.
Go Math Grade 8 Answer Key Chapter 13 Volume Lesson 3: Model Quiz img 23
_______ in3

Answer:
113 in³

Explanation:
Radius of sphere = 3 ft
Volume of sphere = 4/3πr³
Volume = 4/3 (3.14) (3)³
Volume = 113.04 ft³
Volume = 113 in³

Question 6.
Go Math Grade 8 Answer Key Chapter 13 Volume Lesson 3: Model Quiz img 24
_______ cm3

Answer:
1149.8 cm³

Explanation:
Diameter = 13 cm
Radius = 13/2 cm = 6.5 cm
Volume of sphere = 4/3πr³
Volume = 4/3 (3.14) (6.5)³
Volume = 1149.7633 cm³
Volume = 1149.8 cm³

ESSENTIAL QUESTION

Question 7.
What measurements do you need to know to find the volume of a cylinder? a cone? a sphere?
Type below:
___________

Answer:
Sphere: To find the volume of the sphere, the radius is to be measured.
Cylinder: To measure the volume of the cylinder, we need to find out the base radius of the base of the cylinder along with the height of the cylinder.
Cone: To calculate the volume of the cone, we need to calculate the base radius of the base of the cone along with the height of the cone.

Explanation:
The volume of sphere = 4/3 πr³
Sphere: For finding the volume of the sphere, the radius is to be measured
The volume of Cylinder = πr²h
Cylinder: To calculate the volume of the cylinder, we need to find out the base radius of the base of the cylinder along with the height of the cylinder
The volume of Cone = 1/3 πr²h
Cone: To calculate the volume of the cone, we need to measure the base radius of the base of the cone along with the height of the cone

Selected Response – Mixed Review – Page No. 418

Question 1.
The bed of a pickup truck measures 4 feet by 8 feet. To the nearest inch, what is the length of the longest thin metal bar that will lie flat in the bed?
Options:
a. 11 ft 3 in.
b. 10 ft 0 in.
c. 8 ft 11 in.
d. 8 ft 9 in.

Answer:
d. 8 ft 9 in.

Explanation:
The length of the longest thin metal bar that will lie flat in the bed’s equal to the length of the bed’s hypotenuse. Let a = 4 and b = 8. Using the Pythagorean Theorem
a² + b² = c²
4² + 8² = c²
16 + 64 = c²
80 = c²
Rounding the length of the hypotenuse to the nearest tenth of a foot
c = 8.9 ft
Therefore, the length of the longest thin metal bar that will lie flat in the bed is 8 ft. 9 in.

Question 2.
Using 3.14 for π, what is the volume of the cylinder below to the nearest tenth?
Go Math Grade 8 Answer Key Chapter 13 Volume Lesson 3: Mixed Review img 25
OPtions:
a. 102 cubic yards
b. 347.6 cubic yards
c. 1,091.6 cubic yards
d. 4,366.4 cubic yards

Answer:
c. 1,091.6 cubic yards

Explanation:
Diameter of the base of the cylinder = 11.4 yd
Radius = 11.4/2 yd = 5.7 yd
Height = 10.7 ys
Volume of cylinder = πr²h
Volume = (3.14) (5.7)² (10.7)
Volume = 1091.599 yd³
Volume = 1091.6 yd³

Question 3.
Rhett made mini waffle cones for a birthday party. Each waffle cone was 3.5 inches high and had a radius of 0.8 inches. What is the volume of each cone to the nearest hundredth?
Options:
a. 1.70 cubic inches
b. 2.24 cubic inches
c. 2.34 cubic inches
d. 8.79 cubic inches

Answer:
c. 2.34 cubic inches

Explanation:
Height of each waffle cone = 3.5 in
Radius of base = 0.8 in
Volume of cone = 1/3 πr²h
Volume = 1/3 (3.14) (0.8)² (3.5)
Volume = 2.344533 in³
Volume = 2.34 in³

Question 4.
What is the volume of a cone that has a height of 17 meters and a base with a radius of 6 meters? Use 3.14 for π and round to the nearest tenth.
Options:
a. 204 cubic meters
b. 640.6 cubic meters
c. 2,562.2 cubic meters
d. 10,249 cubic meters

Answer:
b. 640.6 cubic meters

Explanation:
Height of the cone = 17 m
Radius of base = 6 m
Volume of cone = 1/3 πr²h
Volume = 1/3 (3.14) (6)² (17)
Volume = 640.56 m³
Volume = 640.6 m³

Question 5.
Using 3.14 for π, what is the volume of the sphere to the nearest tenth?
Go Math Grade 8 Answer Key Chapter 13 Volume Lesson 3: Mixed Review img 26
Options:
a. 4,180 cubic centimeters
b. 5,572.5 cubic centimeters
c. 33,434.7 cubic centimeters
d. 44,579.6 cubic centimeters

Answer:
b. 5,572.5 cubic centimeters

Explanation:
Diameter of the base of the sphere = 22 cm
Radius = 22/2 yd = 11 cm
Volume of sphere = 4/3 πr³
Volume = 4/3 (3.14) (11)³
Volume = 5572.4533 cm³
Volume = 5572.5 cm³

Mini-Task

Question 6.
A diagram of a deodorant container is shown. It is made up of a cylinder and half of a sphere.
Go Math Grade 8 Answer Key Chapter 13 Volume Lesson 3: Mixed Review img 27
Use 3.14 for π and round answers to the nearest tenth.
a. What is the volume of the half sphere?
_______ cm3

Answer:
8.574 cm³

Explanation:
The radius of the cylinder as well as the hemisphere = 1.6 cm
Height = 6.2 cm
the volume of the hemisphere = 2/3 πr³
the volume of the hemisphere = 2/3 (3.14) (1.6)³
the volume of the hemisphere = 8.574 cm³

Question 6.
b. What is the volume of the cylinder?
_______ cm3

Answer:
49.838 cm³

Explanation:
The volume of cylinder = πr²h
= (3.14) (1.6)² (6.2)
= 49.838 cm³

Question 6.
c. What is the volume of the whole figure?
_______ cm3

Answer:
58.4 cm³

Explanation:
Total volume = Volume of cylinder + volume of a hemisphere
Total volume = 8.574 cm³ + 49.838 cm³
Total volume = 58.4 cm³

Conclusion:

I wish the detail provided in Go Math Grade 8 Solution Key Chapter 13 Volume is helpful for you guys. In addition to the exercise problems, you can also get the explanation with answers to the review test. Check out the solutions after your practice from our Go Math 8th Grade Chapter 13 Volume. Hence Download HMH 8th Grade Go Math Answer Key Chapter 13 Volume pdf for free. Learn the concept of volume from here and compare them with real life. This will help you to improve your skills and also score good marks in the exams.

Go Math Grade 3 Answer Key Chapter 3 Understand Multiplication Extra Practice

go-math-grade-3-chapter-4-multiplication-facts-and-strategies-extra-practice-answer-key

Avail the handy resource available for your needs Go Math Grade 3 Answer Key Chapter 3 Understand Multiplication Extra Practice. Check out worked out solutions for all the problems and have an indepth knowledge of the concepts in Go Math Grade 3 Chapter 3. Get acquainted with the kind of questions using the 3rd Grade Go Math Solutions Key Ch 3 Understand Multiplication Extra Practice. Elaborate Explanation is provided in the Go Math Grade 3 Answer Key making it easier for you to understand the concepts underlying.

Go Math Grade 3 Chapter 3 Understand Multiplication Extra Practice Answer Key

The Series of Math Problems provided will help you attain great grades in your exams. Refer our Go Math Grade 3 Answer Key Chapter 3 Understand Multiplication Extra Practice and solve different questions. Practice as many times as possible so that you can attempt the exam with confidence and clear the tests with flying colors.

Common Core – Page No. 63000

Lesson 3.1

Draw equal groups. Skip count to find how many.

Question 1.
2 groups of 4 ______

Answer:
8

Explanation:
Draw 4 counters in each group.
There are 2 equal groups.
skip count by 4 until you say 2 numbers.
There are 2 equal groups with 4 counters in each group.
So, there are 8 counters in all.

Question 2.
4 groups of 3 ______

Answer:
12

Explanation:
Draw 3 counters in each group.
There are 4 equal groups.
skip count by 3 until you say 4 numbers.
There are 4 equal groups with 3 counters in each group.
So, there are 12 counters in all.

Lesson 3.2

Draw a quick picture to show the equal groups. Then write related addition and multiplication sentences.

Question 3.
2 groups of 5
______ + ______ = ______
______ × ______ = ______

Answer:
5 + 5 = 10
2 x 5 = 10

Explanation:
Addition Sentence
Draw 5 counters in each group.
There are a total of 2 groups.
Now, the addition sentence is 5 + 5 = 10.

Multiplication sentence
Draw 5 counters in each circle or group.
Since there is the same number of counters in each group, multiply counters and groups to find how many there are altogether.
2 x 5 = 10.
factor x factor = product

Question 4.
3 groups of 2
______ + ______ + ______ = ______
______ × ______ = ______

Answer:
2 + 2 + 2 = 6
3 x 2 = 6

Explanation:
Addition Sentence
Draw 2 counters in each group.
There are a total of 3 groups.
Now, the addition sentence is 2 + 2 + 2 = 6

Multiplication sentence
Draw 2 counters in each circle or group.
Since there is the same number of counters in each group, multiply groups and counters to find how many there are altogether.
3 x 2 = 6.
factor x factor = product

Lesson 3.3

1. Draw jumps on the number line to show 3 groups of 6.
Find the product.

Question 5.
Go Math Grade 3 Answer Key Chapter 3 Understand Multiplication Extra Practice Common Core img 1
3 × 6 = ______

Answer:
Chapter 12 Common Core image 1 63000
Explanation:
3 x 6 = 18

Write the multiplication sentence the number line shows.

Question 6.
Go Math Grade 3 Answer Key Chapter 3 Understand Multiplication Extra Practice Common Core img 2
______ × ______ = ______

Answer:
4 x 3 = 12

Explanation:
1 jump on the number is considered as 1 group.
There are 3 jumps on a number line. So, there are 3 groups.
The length of each jump is 4.
Begin at 0. Skip count by 3’s.
Multiply 4 x 3 = 12.

Common Core – Page No. 64000

Lesson 3.4

Question 1.
Destiny placed her hair ribbons in 3 groups of 5 on her dresser. How many hair ribbons in all does Destiny have? Draw a diagram to solve.
______ hair ribbons

Answer:
15 hair ribbons

Explanation:
3 groups of 5
Draw 3 counters (hair ribbons) in each group.
There are 5 equal groups (dresser).
We skip count by 3’s until you say 5 numbers (3,6,9,12,15)
There are 15 hair ribbons in all.

Lesson 3.5

Draw an array to find the product.

Question 2.
2 × 7 = ______

Answer:
2 x 7 = 14

Explanation:
Chapter 12 Common Core image 2 64000
2 x 7 = 14

Question 3.
2 × 6 = ______

Answer:
2 x 6 = 12

Explanation:
Chapter 12 Common Core image 4 64000
2 x 6 = 12

Lesson 3.6

Write a multiplication sentence for the model. Then use the Commutative Property of Multiplication to write a related multiplication sentence.

Question 4.
□□□□□
□□□□□
□□□□□
□□□□□
______ × ______ = ______
______ × ______ = ______

Answer:
4 x 5 = 20
5 x 4 = 20

Explanation:
The given image has 4 rows and 5 columns of square boxes. So, multiplication = 4 x 5. Using Commutative Property of Multiplication 4 x 5 = 5 x 4.

Question 5.
______ × ______ = ______
______ × ______ = ______

Answer:

Explanation:

Lesson 3.7

Find the product.

Question 6.
6 × 0 = ______

Answer:
0

Explanation:
The Zero Property of Multiplication states that the product of zero and any number is zero. So, 6 x 0 = 0.

Question 7.
5 × 1 = ______

Answer:
5

Explanation:
Use Commutative Property of Multiplication: 5 x 1 = 1 x 5.
The Identity Property of Multiplication states that the product of any number and 1 is that number. 5 x 1 = 5.

Question 8.
0 × 9 = ______

Answer:
0

Explanation:
The Zero Property of Multiplication states that the product of zero and any number is zero. So, 0 x 9 = 0.

Question 9.
1 × 8 = ______

Answer:
8

Explanation:
The Identity Property of Multiplication states that the product of any number and 1 is that number. 1 x 8 = 8.

Question 10.
1 × 4 = ______

Answer:
4

Explanation:
The Identity Property of Multiplication states that the product of any number and 1 is that number. 1 x 4 = 4.

Question 11.
9 × 1 = ______

Answer:
9

Explanation:
Use Commutative Property of Multiplication: 9 x 1 = 1 x 9.
The Identity Property of Multiplication states that the product of any number and 1 is that number. 9 x 1 = 9.

Question 12.
1 × 0 = ______

Answer:
0

Explanation:
The Zero Property of Multiplication states that the product of zero and any number is zero.
So, 1 x 0 = 0.

Question 13.
7 × 0 = ______

Answer:
0

Explanation:
The Zero Property of Multiplication states that the product of zero and any number is zero.
So, 7 x 0 = 0.

Conclusion

Go Math Grade 3 Answer Key Chapter 3 Understand Multiplication Extra Practice is intended to help the students to increase their solving ability. It is a great opportunity for students to learn the best way to solve questions. Go Math Grade 3 Chapter 3 Understand Multiplication Extra Practice Answer Key is the first priority who wants to achieve their top grades. Download HMH Go Math Grade 3 Answer Key for free.

Go Math Grade 3 Answer Key Chapter 11 Perimeter and Area

go-math-grade-3-chapter-11-perimeter-and-area-answer-key

Go Math Grade 3 Answer Key Chapter 11 Perimeter and Area helps you prepare for the exams. All the Concepts of Area and Perimeter are clearly explained in our Chapter 11 Perimeter and Area Go Math Grade 3 Answer Key. Go Math Grade 3 Perimeter and Area can be of great help during your preparation. You can get the Homework Help needed by referring to the Go Math Grade 3 Answer Key Chapter 11 Perimeter and Area.

Go Math Grade 3 Chapter 11 Perimeter and Area Answer Key

Enhance your Problem-Solving Skills using the HMH Go Math Grade 3 Ch 11 Perimeter and Area Solution Key. Cross Check the Solutions in the 3rd Grade Go Math Ch 11 Perimeter and Area Answer Key and understand where you went wrong. Get to know the Lessons in Chapter 11 with the quick links available below and solve the problems in it.

Lesson 1: Model Perimeter

Lesson 2: Find Perimeter

Lesson 3: Find Unknown Side Lengths

Lesson 4: Understand Area

Lesson 5: Measure Area

Lesson 6: Use Area Models

Mid -Chapter Checkpoint

Lesson 7: Problem Solving Area of Rectangles

Lesson 8: Area of Combined Rectangles

Lesson 9: Same Perimeter, Different Areas

Review/Test

Model Perimeter – Page No. 629

Find the perimeter of the shape. Each unit is 1 centimeter.

Question 1.
Go Math Grade 3 Answer Key Chapter 11 Perimeter and Area Model Perimeter img 1
22 centimeters

Answer:
22 centimeters

Explanation:
Model Perimeter Image 1
Each square in the grid is a 1 by 1-centimeter square. So, we have to do is add up the lengths of the dark segments right over the figure. Start the count from the box where 1 is placed. This parameter is 1, 2, 3, 4, 5, 6, 7, 8, 9, 10, 11, 12, 13, 14, 15, 16, 17, 18, 19, 20, 21, 22 centimeters long. So, it is 22 centimeters.

Question 2.
Go Math Grade 3 Answer Key Chapter 11 Perimeter and Area Model Perimeter img 2
__________ centimeters

Answer:
22 centimeters

Explanation:
Model Perimeter Image 2
Look at the length of each side. Then, add the length of each side to get the perimeter of the given shape. The lengths of the sides are 6 + 5 + 6 + 5 = 22 centimeters.

Question 3.
Go Math Grade 3 Answer Key Chapter 11 Perimeter and Area Model Perimeter img 3
__________ centimeters

Answer:
26 centimeters

Explanation:
Model perimeter Image 3
Given that each unit is 1 centimeter. Count the lengths of each box from number 1. So, this parameter is 26 centimeters long. or 6 + 5 + 2 + 2 + 2 + 2 + 2 + 5 = 26.

Question 4.
Go Math Grade 3 Answer Key Chapter 11 Perimeter and Area Model Perimeter img 4
__________ centimeters

Answer:
30 centimeters

Explanation:
Model Perimeter Image 4
Look at the length of each side. Then, add the length of each side to get the perimeter of the given shape. The lengths of the sides are 2 + 3 + 3+ 3 + 2 + 5 + 7 + 5 = 30 centimeters long.

Problem Solving

Use the drawing for 5–6. Each unit is 1 centimeter.
Go Math Grade 3 Answer Key Chapter 11 Perimeter and Area Model Perimeter img 5

Question 5.
What is the perimeter of Patrick’s shape?
__________ centimeters

Answer:
20 centimeters

Explanation:
Model Perimeter Image 5
The perimeter of Patrick’s shape = 5 + 5 + 5 + 5 =20 centimeters.

Question 6.
How much greater is the perimeter of Jillian’s shape than the perimeter of Patrick’s shape?
__________ centimeters

Answer:
2 centimeters

Explanation:
Model Perimeter Image 6
First, the perimeter of Jillian’s shape = 8 + 1 + 4 + 2 + 4 + 3 = 22 centimeters.
Difference = 22 – 20 = 2 centimeters.
The perimeter of Jillian’s shape is 2 centimeters greater than the perimeter of Patrick’s shape.

Model Perimeter – Page No. 630

Lesson Check

Question 1.
Find the perimeter of the shape.
Each unit is 1 centimeter.
Go Math Grade 3 Answer Key Chapter 11 Perimeter and Area Model Perimeter img 6
Options:
a. 14 centimeters
b. 16 centimeters
c. 18 centimeters
d. 20 centimeters

Answer:
d. 20 centimeters

Explanation:
Model Perimeter Image 7
Given that each unit is 1 centimeter. Count the lengths of each box of the dark lines from number 1. So, this parameter is 20 centimeters long.

Question 2.
Find the perimeter of the shape.
Each unit is 1 centimeter.
Go Math Grade 3 Answer Key Chapter 11 Perimeter and Area Model Perimeter img 7
Options:
a. 19 centimeters
b. 26 centimeters
c. 33 centimeters
d. 55 centimeters

Answer:
b. 26 centimeters

Explanation:
Model Perimeter Image 9
Find the length of each box and add them to find the perimeter of the given shape. The perimeter of the given shape is 8 + 4 +3 + 1 + 4 + 4 + 1 + 1 = 26 centimeters.

Spiral Review

Question 3.
Which lists the fractions in order from least to greatest?
\(\frac{2}{4}, \frac{2}{3}, \frac{2}{6}\)
Options:
a. \(\frac{2}{3}, \frac{2}{4}, \frac{2}{6}\)
b. \(\frac{2}{6}, \frac{2}{4}, \frac{2}{3}\)
c. \(\frac{2}{4}, \frac{2}{3}, \frac{2}{6}\)
d. \(\frac{2}{3}, \frac{2}{6}, \frac{2}{4}\)

Answer:
b. \(\frac{2}{6}, \frac{2}{4}, \frac{2}{3}\)

Explanation:
The numerator of the given factors is the same. So, look at the denominators to compare the size of the pieces. As the denominator gets smaller, the fraction gets larger. In the given problem, 3 is the smaller denominator compared to 4 and 6. Then, 4 is the next smaller denominator. So, the fractions in order from least to greatest are 2/6, 2/4, 2/3.

Question 4.
Kasey’s school starts at the time shown on the clock. What time does Kasey’s school start?
Go Math Grade 3 Answer Key Chapter 11 Perimeter and Area Model Perimeter img 8
Options:
a. 6:40
b. 8:06
c. 8:30
d. 9:30

Answer:
c. 8:30

Explanation:
In the figure, hour-hand is indicating number 8. So, we say it is 8 hours. The minute hand is on 6. To find the minutes, multiply 6 x 5 = 30. So, it indicates 30 minutes. The time is 8 hours 30 minutes.

Question 5.
Michael and Dex are comparing fraction strips. Which statement is NOT correct?
Options:
a. \(\frac{1}{2}<\frac{2}{2}\)
b. \(\frac{2}{3}>\frac{1}{3}\)
c. \(\frac{4}{8}<\frac{3}{8}\)
d. \(\frac{4}{6}>\frac{2}{6}\)

Answer:
c. \(\frac{4}{8}<\frac{3}{8}\)

Explanation:
The denominators of the given fractions are the same. So, look at the numerators to compare the numbers.
1 < 2 is correct.
2 > 1 is correct.
4 < 3 is not correct. So, 4/8 < 3/8 is not correct.

Question 6.
Aiden wants to find the mass of a bowling ball. Which unit should he use?
Options:
a. liter
b. inch
c. gram
d. kilogram

Answer:
d. kilogram

Explanation:
The kilogram is used to find the mass of a bowling ball.

Find Perimeter – Page No. 635

Use a ruler to find the perimeter. (Note: on mobile devices like smart phones or tablets, the measurements will not be accurate.)

Question 1.
Go Math Grade 3 Answer Key Chapter 11 Perimeter and Area Find Perimeter img 9
12 centimeters

Answer:
12 cm

Explanation:
Add the lengths of the sides measured to the perimeter. 4 + 3 + 2 + 3 = 12 cm.

Question 2.
Go Math Grade 3 Answer Key Chapter 11 Perimeter and Area Find Perimeter img 10
_________ centimeters

Answer:
13 centimeters

Explanation:
Use a centimeter ruler to measure the length of each side.
Record and add the lengths of the sides measured to the nearest centimeter.
5 cm + 1 cm + 1 cm + 2 cm + 4 cm = 13 centimeters.

Question 3.
Go Math Grade 3 Answer Key Chapter 11 Perimeter and Area Find Perimeter img 11
_________ inches

Answer:
8 inches

Explanation:
Use an inch ruler to measure the length of each side to the nearest inch.
Record and add the lengths of the sides measured to the nearest inch.
2 in + 2 in + 2 in + 2 in =8 inches.

Question 4.
Go Math Grade 3 Answer Key Chapter 11 Perimeter and Area Find Perimeter img 12
_________ inches

Answer:
6 inches

Explanation:
Use an inch ruler to measure the length of each side.
Record and add the lengths of the sides measured to the nearest inch.
2 in + 2 in + 2 in = 6 inches.

Problem Solving

Draw a picture to solve 5–6.

Question 5.
Evan has a square sticker that measures 5 inches on each side. What is the perimeter of the sticker?
_________ inches

Answer:
20 inches

Explanation:
Chapter 11 - Find Perimeter- image 33
The square has equal sides. Each side has 5 inches. So, the perimeter of the sticker is 5 x 4 = 20 inches.

Question 6.
Sophie draws a shape that has 6 sides. Each side is 3 centimeters. What is the perimeter of the shape?
_________ centimeters

Answer:
18 centimeters

Explanation:
The perimeter of the shape is = Addition of all sides.
Given that Sophie draws a shape that has 6 sides with 3 centimeters. So, the perimeter of the shape = 3 + 3 + 3 + 3 + 3 + 3 = 18 centimeters

Find Perimeter – Page No. 636

Lesson Check

Use an inch ruler for 1–2.

Question 1.
Ty cut a label the size of the shape shown. What is the perimeter, in inches, of Ty’s label?
Go Math Grade 3 Answer Key Chapter 11 Perimeter and Area Find Perimeter img 13
Options:
a. 4 inches
b. 5 inches
c. 6 inches
d. 7 inches

Answer:
c. 6 inches

Explanation:
Chapter 11 - Find perimeter - image 34
By using an inch ruler, Ty can measure the length of each side of the given shape. Now, find the lengths of each side.
1 in + 2 in + 1 in + 2 in = 6 inches.

Question 2.
Julie drew the shape shown below. What is the perimeter, in inches, of the shape?
Go Math Grade 3 Answer Key Chapter 11 Perimeter and Area Find Perimeter img 14
Options:
a. 2 inches
b. 4 inches
c. 6 inches
d. 8 inches

Answer:
d. 8 inches

Explanation:
Chapter 11 - Find perimeter - image 35
Julie can use an inch ruler to measure the length of each side of the shape. Now, find the lengths of each side. Each side has 2 inches. So, the perimeter of the shape is 2 in + 2 in + 2 in + 2 in = 8 inches.

Spiral Review

Question 3.
What is the perimeter of the shape below?
Go Math Grade 3 Answer Key Chapter 11 Perimeter and Area Find Perimeter img 15
Options:
a. 8 units
b. 10 units
c. 20 units
d. 22 units

Answer:
c. 20 units

Explanation:
Model Perimeter Image 10
From the figure, each unit is 1 centimeter. Count the lengths of each box from number 1. So, this parameter is 20 units long.

Question 4.
Vince arrives for his trumpet lesson after school at the time shown on the clock. What time does Vince arrive for his trumpet lesson?
Go Math Grade 3 Answer Key Chapter 11 Perimeter and Area Find Perimeter img 16
Options:
a. 3:26 A.M.
b. 4:26 A.M.
c. 3:26 P.M.
d. 4:26 P.M.

Answer:
c. 3:26 P.M.

Explanation:
Given that Vince arrives for his trumpet lesson after school at the time shown on the clock. As it is mentioned after school time, it is Noon. The times afternoon and before midnight are written with P.M. The hour hand is indicating number 3. So, the answer is 3:26 P.M.

Question 5.
Matthew’s small fish tank holds 12 liters. His large fish tank holds 25 liters. How many more liters does his large fish tank hold?
Options:
a. 12 liters
b. 13 liters
c. 25 liters
d. 37 liters

Answer:
b. 13 liters

Explanation:
To get the more liters, do subtraction of large fish tank liters to small fish tank liters. So, The large fish tank is 25 liters – 12 liters = 13 liters more than Matthew’s small fish tank.

Question 6.
Cecila and Sasha are comparing fraction strips. Which statement is correct?
Options:
a. \(\frac{1}{2}<\frac{1}{3}\)
b. \(\frac{1}{8}>\frac{1}{6}\)
c. \(\frac{1}{4}>\frac{1}{2}\)
d. \(\frac{1}{6}<\frac{1}{4}\)

Answer:
d. \(\frac{1}{6}<\frac{1}{4}\)

Explanation:
Cecila and Sasha are comparing fraction strips. In the given fractions, the numerators are same. So, compare the denominators to find the largest fraction. As the denominator gets smaller, the fraction gets larger.
2 < 3 wrong.
8 > 6 wrong.
4 > 2 wrong.
6 < 4 correct.
So, 1/6 < 1/4 is the correct answer.

Find Unknown Side Lengths – Page No. 641

Find the unknown side lengths.

Question 1.
Perimeter = 33 centimeters
Go Math Grade 3 Answer Key Chapter 11 Perimeter and Area Find Unknown Side Lengths img 17
5 + 8 + 7 + 4 + x = 33
24 + x = 33
x = 9
x = ____ 9 _____ centimeters

Answer:
9 centimeters

Explanation:
If I knew the length x, I would add all the side lengths to find the perimeter. Given Perimeter = 33 centimeters.
Add the lengths of the given sides. 5 + 8 + 7 + 4 + x = 33.
24 + x = 33.
x = 33 – 24 =9.
x = 9 centimeters.

Question 2.
Perimeter = 14 feet
Go Math Grade 3 Answer Key Chapter 11 Perimeter and Area Find Unknown Side Lengths img 18
r = ____ feet

Answer:
3 feet

Explanation:
Given Perimeter = 14 feet.
Add the lengths of the given sides to find r.
4 ft + 4 ft + r + r = 14 feet.
If r =1 -> 4 + 4 + 1 + 1 = 10 not equal to 14.
If r = 2 -> 4 + 4 + 2 + 2 = 12 not equal to 14.
If r = 3 -> 4 + 4 + 3 + 3 = 14 equal to 14.
So, r = 3 feet.

Question 3.
Perimeter = 37 meters
Go Math Grade 3 Answer Key Chapter 11 Perimeter and Area Find Unknown Side Lengths img 19
s = ____ meters

Answer:
11 meters

Explanation:
Given that Perimeter = 37 meters.
Add the lengths of the given sides to find s.
8 + 11 + 5 + 2 + s = 37.
26 + s = 37.
s = 37 – 26 = 11.
s = 11 meters.

Question 4.
Perimeter = 92 inches
Go Math Grade 3 Answer Key Chapter 11 Perimeter and Area Find Unknown Side Lengths img 20
t = ____ inches

Answer:
15 inches

Explanation:
Given that Perimeter = 92 inches.
Add the lengths of the given sides to find t.
7 + 23 + 12 + 12 +23 + t = 92
77 + t = 92.
s = 92 – 77 = 15.
s = 15 inches.

Problem Solving

Question 5.
Steven has a rectangular rug with a perimeter of 16 feet. The width of the rug is 5 feet. What is the length of the rug? ____ feet

Answer:
3 feet

Explanation:
Chapter 11 - FInd unknown side lengths - image 36
Given that Steven has a rectangular rug with a perimeter of 16 feet. Rectangular has 4 sides with two pairs of opposite sides that are equal in length.
Let the length will be x.
If x = 1 feet -> 5 + x + 5 + x = 5 + 1 + 5 + 1 = 12 feet not equal to 16.
If x = 2 feet -> 5 + x + 5 + x = 5 + 2 + 5 + 2 = 14 feet not equal to 16.
If x = 3 feet -> 5 + x + 5 + x = 5 + 3 + 5 + 3 = 16 feet equal to 16.
So, the length of the blanket is 3 feet.

Question 6.
Kerstin has a square tile. The perimeter of the tile is 32 inches. What is the length of each side of the tile?
____ inches

Answer:
8 inches

Explanation:
Chapter 11 - FInd unknown side lengths - image 37
A square has four sides that are equal in length.
So, 4 x s = 32
4 x 8 = 32.
So, the length of each side of the square is 8 inches.

Find Unknown Side Lengths – Page No. 642

Lesson Check

Question 1.
Jesse is putting a ribbon around a square frame. He uses 24 inches of ribbon. How long is each side of the frame? Options:
a. 4 inches
b. 5 inches
c. 6 inches
d. 8 inches

Answer:
c. 6 inches

Explanation:
Jesse is putting a ribbon around a square frame. A square has four sides that are equal in length.
So, 4 x k = 24 inches.
4 x 6 = 24 inches.
Each side of the frame 6 inches.

Question 2.
Davia draws a shape with 5 sides. Two sides are each 5 inches long. Two other sides are each 4 inches long. The perimeter of the shape is 27 inches. What is the length of the fifth side?
Options:
a. 9 inches
b. 13 inches
c. 14 inches
d. 18 inches

Answer:
a. 9 inches

Explanation:
. From the given information, Davia draws a shape with 5 sides.
2 sides = each 5 inches long.
2 sides = each 4 inches long.
Let the other side is k.
The perimeter of the shape = 5 + 5 + 4 + 4 + k = 27 inches.
18 + k = 27 inches.
k = 27 – 18 inches.
k = 9 inches.

Spiral Review

Question 3.
Which of the following represents 7 + 7 + 7 + 7?
Options:
a. 4 × 4
b. 4 × 7
c. 6 × 7
d. 7 × 7

Answer:
b. 4 × 7

Explanation:
7 + 7 + 7 + 7 = 28.
4 x 4 = 16 not equal to 28.
4 x 7 = 28 equal to 28.
7 + 7 + 7 + 7 = 4 × 7.

Question 4.
Bob bought 3 packs of model cars. He gave 4 cars to Ann. Bob has 11 cars left. How many model cars were in each pack?
Options:
a. 18
b. 11
c. 7
d. 5

Answer:
d. 5

Explanation:
If Bob has 11 cars left after he gave 4 to Ann, that means he had 15 cars in total. If Bob had 15 cars in total, and these 15 cars were divided into 3 packs, that means each pack contained 5 cars. The answer is 5 cars in each pack.

Question 5.
Randy looked at his watch when he started and finished reading. How long did Randy read?
Go Math Grade 3 Answer Key Chapter 11 Perimeter and Area Find Unknown Side Lengths img 21
Options:
a. 55 minutes
b. 45 minutes
c. 35 minutes
d. 15 minutes

Answer:
b. 45 minutes

Explanation:
Randy started reading at 4:10.
Randy finished reading at 4:55.
Subtract to find the Randy read time.
4:55 – 4:10 = 45 minutes
So, Randy read 45 minutes.

Question 6.
Which statement does the model represent?
Go Math Grade 3 Answer Key Chapter 11 Perimeter and Area Find Unknown Side Lengths img 22
Options:
a. \(\frac{4}{4}\) = 1
b. \(\frac{3}{4}\) = 1
c. \(\frac{2}{4}\) = 1
d. \(\frac{1}{4}\) = 1

Answer:
a. \(\frac{4}{4}\) = 1

Explanation:
The first image represents the whole circle divided into 4 equal parts. So, each part is \(\frac{1}{4}\).
Together, all 4 parts represent \(\frac{4}{4}\) or one whole circle.
The second image represents one whole circle.
\(\frac{4}{4}\) = 1.

Understand Area – Page No. 647

Count to find the area for the shape.

Question 1.
Go Math Grade 3 Answer Key Chapter 11 Perimeter and Area Understand Area img 23
Area = 6 square units

Answer:
6 square units

Explanation:
chapter 11 - understand area - image 1
The area is the measure of the number of unit squares needed to cover a flat surface. A unit square is a square with a side length of 1 unit. It has an area of 1 square unit (sq un). Count the number of Squares to get the answers. 1, 2, 3, 4, 5, 6.
Area = 6 square units.

Question 2.
Go Math Grade 3 Answer Key Chapter 11 Perimeter and Area Understand Area img 24
Area = ______ square units

Answer:
4 square units

Explanation:
chapter 11 - understand area - image 2
Count the number of Squares to get the answers. 1, 2, 3, 4.
Area = 4 square units.

Question 3.
Go Math Grade 3 Answer Key Chapter 11 Perimeter and Area Understand Area img 25
Area = ______ square units

Answer:
5 square units

Explanation:
chapter 11 - understand area - image 3
Count the number of Squares to get the answers. 1, 2, 3, 4, 5.
Area = 5 square units.

Question 4.
Go Math Grade 3 Answer Key Chapter 11 Perimeter and Area Understand Area img 26
Area = ______ square units

Answer:
7 square units

Explanation:
chapter 11 - understand area - image 4
Count the number of Squares to get the answers. 1, 2, 3, 4, 5, 6, 7.
Area = 7 square units.

Question 5.
Go Math Grade 3 Answer Key Chapter 11 Perimeter and Area Understand Area img 27
Area = ______ square units

Answer:
8 square units

Explanation:
chapter 11 - understand area - image 5
Count the number of Squares to get the answers. 1, 2, 3, 4, 5, 6, 7, 8.
Area = 8 square units.

Question 6.
Go Math Grade 3 Answer Key Chapter 11 Perimeter and Area Understand Area img 28
Area = ______ square units

Answer:
13 square units

Explanation:
chapter 11 - understand area - image 6
Count the number of Squares to get the answers. 1, 2, 3, 4, 5, 6, 7, 8,9, 10, 11, 12, 13.
Area = 13 square units.

Write area or perimeter for each situation.

Question 7.
carpeting a floor
_________

Answer:
Area

Question 8.
fencing a garden
_________

Answer:
2. Perimeter

Problem Solving

Use the diagram for 9–10.
Go Math Grade 3 Answer Key Chapter 11 Perimeter and Area Understand Area img 29

Answer:
12 square units

Explanation:
chapter 11 - understand area - image 7
Count the number of Squares to get the answers. 1, 2, 3, 4, 5, 6, 7, 8,9, 10, 11, 12.
Area = 12 square units.

Question 9.
Rober to is building a platform for his model railroad. What is the area of the platform?
Area = ______ square units

Answer:
12 square units

Explanation:
The area of the platform = Count the total number of Squares to get the answers. 1, 2, 3, 4, 5, 6, 7, 8,9, 10, 11, 12.
Area = 12 square units.

Question 10.
Rober to will put a border around the edges of the platform. How much border will he need?
Border = ______ units

Answer:
16 units

Explanation:
To know the edges of the platform, we need to calculate the perimeter of the given shape. So, we need to add the length of all sides. 2 + 4 + 3 + 2 + 1 + 1 + 2 + 1 = 16 units.

Understand Area – Page No. 648

Lesson Check

Question 1.
Josh used rubber bands to make the shape below on his geoboard. What is the area of the shape?
Go Math Grade 3 Answer Key Chapter 11 Perimeter and Area Understand Area img 30
Options:
a. 3 square units
b. 4 square units
c. 5 square units
d. 6 square units

Answer:
a. 3 square units

Explanation:
chapter 11 - understand area - image 8
If Josh used rubber bands to make the shape below on his geoboard, he can divide the shape into 2 smaller regular shapes.
The shape 1 marked as square = 1 square unit.
The shape 2 marked as rectangle = 2 square units.
The total area = Shape 1 + Shape 2 = 1 + 2 = 3 square units.

Question 2.
Wilma drew the shape below on dot paper. What is the area of the shape she drew?
Go Math Grade 3 Answer Key Chapter 11 Perimeter and Area Understand Area img 31
Options:
a. 4 square units
b. 5 square units
c. 6 square units
d. 7 square units

Answer:
b. 5 square units

Explanation:
chapter 11 - understand area - image 9
Wilma drew the shape below on dot paper.
The area of the shape = number of unite square boxes = 1 + 1 + 1 + 1 + 1 = 5 square units.

Spiral Review

Question 3.
Leonardo knows it is 42 days until summer break. How many weeks is it until Leonardo’s summer break? (Hint: There are 7 days in a week.)
Options:
a. 5 weeks
b. 6 weeks
c. 7 weeks
d. 8 weeks

Answer:
b. 6 weeks

Explanation:
Leonardo knows it is 42 days until summer break. There are 7 days in a week. So, by dividing 42 with 7 Leonardo can find summer break in weeks. 42/7 = 6 weeks.

Question 4.
Nan cut a submarine sandwich into 4 equal parts and ate one part. What fraction represents the part of the sandwich Nan ate?
Options:
a. \(\frac{1}{4}\)
b. \(\frac{1}{3}\)
c. \(\frac{4}{4}\)
d. \(\frac{4}{1}\)

Answer:
a. \(\frac{1}{4}\)

Explanation:
Nan cut a submarine sandwich into 4 equal parts and ate one part. So, remaining parts are 3. The fraction of the sandwich Nan ate = \(\frac{1}{4}\).

Question 5.
Wanda is eating breakfast. Which is a reasonable time for Wanda to be eating breakfast?
Options:
a. 7:45 A.M.
b. 7:45 P.M.
c. 2:15 P.M.
d. 2:15 A.M.

Answer:
a. 7:45 A.M.

Explanation:
The reasonable time for Wanda to eat breakfast is 7:45 A.M. Because breakfast will eat in the morning i.e, A.M.

Question 6.
Dick has 2 bags of dog food. Each bag contains 5 kilograms of food. How many kilograms of food does Dick have in all?
Options:
a. 3 kilograms
b. 5 kilograms
c. 7 kilograms
d. 10 kilograms

Answer:
d. 10 kilograms

Explanation:
Dick has 2 x 5 = 10 kilograms of food in total.

Measure Area – Page No. 653

Count to find the area of the shape. Each unit square is 1 square centimeter.

Question 1.
Go Math Grade 3 Answer Key Chapter 11 Perimeter and Area Measure Area img 32
Area 14 square centimeters.

Answer:
14 square centimeters

Explanation:
Model Perimeter image 11
The number of unit square in the given figure is 14. So, the area of the shape = 14 square centimeters.

Question 2.
Go Math Grade 3 Answer Key Chapter 11 Perimeter and Area Measure Area img 33
Area = ________ square centimeters

Answer:
16 square centimeters

Explanation:
Model Perimeter Image 12
The area of the shape = 16 square centimeters.

Question 3.
Go Math Grade 3 Answer Key Chapter 11 Perimeter and Area Measure Area img 34
Area = ________ square centimeters

Answer:
11 square centimeters

Explanation:
Model Perimeter Image 13
The area of the shape = 11 square centimeters.

Question 4.
Go Math Grade 3 Answer Key Chapter 11 Perimeter and Area Measure Area img 35
Area = ________ square centimeters

Answer:
22 square centimeters

Explanation:
Model Perimeter Image 14
The area of the shape = 22 square centimeters.

Problem Solving

Alan is painting his deck gray. Use the diagram at the right for 5–6. Each unit square is 1 square meter.
Go Math Grade 3 Answer Key Chapter 11 Perimeter and Area Measure Area img 36

Question 5.
What is the area of the deck that Alan has already painted gray?
_______ square meters

Answer:
16 square meters

Explanation:
Model Perimeter Image 17
The area of the deck that Alan has already painted gray is 16 square meters.

Question 6.
What is the area of the deck that Alan has left to paint?
_______ square meters

Answer:
19 square meters

Explanation:
Model Perimeter Image 18
The area of the deck that Alan has left to paint is 19 square meters.

Measure Area – Page No. 654

Lesson Check

Each unit square in the diagram is 1 square foot.
Go Math Grade 3 Answer Key Chapter 11 Perimeter and Area Measure Area img 37

Question 1.
How many square feet are shaded?
Options:
a. 19 square feet
b. 21 square feet
c. 23 square feet
d. 25 square feet

Answer:
c. 23 square feet

Explanation:
Model Perimeter Image 20

Question 2.
What is the area that has NOT been shaded?
Options:
a. 19 square feet
b. 21 square feet
c. 23 square feet
d. 25 square feet

Answer:
a. 19 square feet

Explanation:
Model Perimeter Image 21

Spiral Review

Question 3.
Sonya buys 6 packages of rolls. There are 6 rolls in each package. How many rolls does Sonya buy?
Options:
a. 42
b. 36
c. 24
d. 12

Answer:
b. 36

Explanation:
6 x 6 = 36. Sonya buys 36 rolls.

Question 4.
Charlie mixed 6 liters of juice with 2 liters of soda to make fruit punch. How many liters of fruit punch did Charlie make?
Options:
a. 3 liters
b. 4 liters
c. 8 liters
d. 12 liters

Answer:
c. 8 liters

Explanation:
6 + 2 = 8 liters. Charlie can make 8 liters of fruit punch.

Question 5.
Which drawing shows \(\frac{2}{3}\) of the circle shaded?
Options:
a. Go Math Grade 3 Answer Key Chapter 11 Perimeter and Area Measure Area img 38
b. Go Math Grade 3 Answer Key Chapter 11 Perimeter and Area Measure Area img 39
c. Go Math Grade 3 Answer Key Chapter 11 Perimeter and Area Measure Area img 40
d.Go Math Grade 3 Answer Key Chapter 11 Perimeter and Area Measure Area img 41

Answer:
d. Go Math Grade 3 Answer Key Chapter 11 Perimeter and Area Measure Area img 41

Explanation:
Option d is the correct answer. 2/3 means the whole circle has 3 parts. 2 of the parts are shaded.

Question 6.
Use the models to name a fraction that is equivalent to \(\frac{1}{2}\).
Go Math Grade 3 Answer Key Chapter 11 Perimeter and Area Measure Area img 42
Options:
a. \(\frac{2}{1}\)
b. \(\frac{2}{2}\)
c. \(\frac{2}{4}\)
d. \(\frac{4}{4}\)

Answer:
c. \(\frac{2}{4}\)

Explanation:
\(\frac{2}{4}\) = \(\frac{1}{2}\). So, the answer is \(\frac{2}{4}\)

Use Area Models – Page No. 659

Find the area of each shape. Each unit square is 1 square foot.

Question 1.
Go Math Grade 3 Answer Key Chapter 11 Perimeter and Area Use Area Models img 43

Answer:
24 square feet

Explanation:
Chapter 11- Use area Models - Image 25 (659)
There are 3 rows of 8 unit squares.
3 x 8 = 24.

Question 2.
Go Math Grade 3 Answer Key Chapter 11 Perimeter and Area Use Area Models img 44

_________ square feet

Find the area of each shape.
Each unit square is 1 square meter.

Answer:
16 square feet

Explanation:
Chapter 11 - Use area models - image 26
There are 4 rows of 4 unit squares.
4 x 4 = 16 square feet.

Question 3.
Go Math Grade 3 Answer Key Chapter 11 Perimeter and Area Use Area Models img 45
_________ square meters

Answer:
12 square meters

Explanation:
Chapter 11 - Use area models - image 27
There are 2 rows of 6 unit squares.
2 x 6 = 12 square meters.

Question 4.
Go Math Grade 3 Answer Key Chapter 11 Perimeter and Area Use Area Models img 46
_________ square meters

Answer:
24 square meters

Explanation:
Chapter 11- use area models - Image 28
There are 4 rows of 6 unit squares.
4 x 6 = 24 square meters.

Question 5.
Go Math Grade 3 Answer Key Chapter 11 Perimeter and Area Use Area Models img 47
_________ square meters

Answer:
15 square meters

Explanation:
Chapter 11 - Use area models - image 29
There are 5 rows of 3 unit squares.
5 x 3 = 15 square meters.

Problem Solving

Question 6.
Landon made a rug for the hallway. Each unit square is 1 square foot. What is the area of the rug?
Go Math Grade 3 Answer Key Chapter 11 Perimeter and Area Use Area Models img 48
_________ square feet

Answer:
20 square feet

Explanation:
chapter 11- use area models - Image 30
Count the number of unit square boxes = 20 square feet. Or, there are 2 rows of 10 unit squares.
2 x 10 = 20 square feet.

Question 7.
Eva makes a border at the top of a picture frame. Each unit square is 1 square inch. What is the area of the border? Go Math Grade 3 Answer Key Chapter 11 Perimeter and Area Use Area Models img 49
_________ square feet

Answer:
8 square feet

Explanation:
Chapter 11 - use area models - image 31
Count the number of unit square boxes = 20 square feet. Or, there are 1 rows of 8 unit squares.
1 x 8 = 8 square feet.

Use Area Models – Page No. 660

Lesson Check

Question 1.
The entrance to an office has a tiled floor. Each square tile is 1 square meter. What is the area of the floor?
Go Math Grade 3 Answer Key Chapter 11 Perimeter and Area Use Area Models img 50
Options:
a. 8 square meters
b. 9 square meters
c. 10 square meters
d. 12 square meters

Answer:
b. 9 square meters

Explanation:
Chapter 11- Use area Models-Image 23(660)
There are 9 square boxes available. So, the area of the floor = 9 square meters.

Question 2.
Ms. Burns buys a new rug. Each unit square is 1 square foot. What is the area of the rug?
Go Math Grade 3 Answer Key Chapter 11 Perimeter and Area Use Area Models img 51
Options:
a. 5 square feet
b. 7 square feet
c. 10 square feet
d. 12 square feet

Answer:
c. 10 square feet

Explanation:
Chapter 11- Use area models-Image 24(660(2))
The area of the rug = 10 square feet.

Spiral Review

Question 3.
Ann and Bill are comparing fraction strips. Which statement is correct?
Options:
a. \(\frac{3}{8}>\frac{5}{8}\)
b. \(\frac{3}{4}<\frac{1}{4}\)
c. \(\frac{3}{6}>\frac{4}{6}\)
d. \(\frac{1}{3}<\frac{2}{3}\)

Answer:
d. \(\frac{1}{3}<\frac{2}{3}\)

Explanation:
The denominator of the given fractions are same. So, compare the numerators to find the correct answer.
3 > 5 wrong.
3 < 1 wrong.
3 > 4 wrong.
1 < 2 correct.
The answer is \(\frac{1}{3}<\frac{2}{3}\).

Question 4.
Claire bought 6 packs of baseball cards. Each pack had the same number of cards. If Claire bought 48 baseball cards in all, how many cards were in each pack?
Options:
a. 54
b. 42
c. 8
d. 6

Answer:
c. 8

Explanation:
6 x k = 48. So, k = 48/6 = 8.
8 cards were in each pack.

Question 5.
Austin left for school at 7:35 A.M.. He arrived at school 15 minutes later. What time did Austin arrive at school?
Options:
a. 7:40 A.M.
b. 7:50 A.M.
c. 7:55 A.M.
d. 8:00 A.M.

Answer:
b. 7:50 A.M.

Explanation:
7:35 A.M. + 15 minutes = 7:50 A.M

Question 6.
Wyatt’s room is a rectangle with a perimeter of 40 feet. The width of the room is 8 feet. What is the length of the room?
Options:
a. 5 feet
b. 12 feet
c. 16 feet
d. 32 feet

Answer:
b. 12 feet

Explanation:
The perimeter of rectangle = 40 feet.
The width of the room is 8 feet.
8 + k + 8 + k = 40.
If k = 10 -> 8 + 10 + 8 + 10 = 36.
If k = 11 -> 8 + 11 + 8 + 11 = 38.
If k = 12 -> 8 + 12 + 8 + 12 = 40.
So, the length of the room = 12 feet.

Mid -Chapter Checkpoint – Page No. 661

Vocabulary

Choose the best term from the box.
Go Math Grade 3 Answer Key Chapter 11 Perimeter and Area Mid -Chapter Checkpoin img 52

Question 1.
The distance around a figure is the _____________ .
_____________

Answer:
Perimeter

Question 2.
The measure of the number of unit squares needed to cover a figure with no gaps or overlaps is the _____________ .
_____________

Answer:
Area

Concepts and Skills

Find the perimeter of the figure. Each unit is 1 centimeter.

Question 3.
Go Math Grade 3 Answer Key Chapter 11 Perimeter and Area Mid -Chapter Checkpoin img 53
_______ centimeters

Answer:
16 centimeters

Explanation:
Chapter 11 - Concepts and Skills -image 32. jpg (2)
5 + 2 + 2 + 1 + 2 + 1 + 1 + 2 = 16 centimeters.

Question 4.
Go Math Grade 3 Answer Key Chapter 11 Perimeter and Area Mid -Chapter Checkpoin img 54
_______ centimeters

Answer:
14 cm

Explanation:
Chapter 11 - Concepts and Skills -image 33. jpg
4 + 3 + 4 + 3 = 14 cm.

Find the unknown side lengths.

Question 5.
Perimeter = 33 centimeters
Go Math Grade 3 Answer Key Chapter 11 Perimeter and Area Mid -Chapter Checkpoin img 55
g = ______ centimeters

Answer:
3 centimeters

Explanation:
Given that Perimeter = 33 centimeters.
10 + 6 + 10 + g + 4 = 33 centimeters.
30 + g = 33 centimeters.
g= 33 – 30 centimeters.
g = 3 centimeters.

Question 6.
Perimeter = 32 feet
Go Math Grade 3 Answer Key Chapter 11 Perimeter and Area Mid -Chapter Checkpoin img 56
k = ______ feet

Answer:
4 feet

Explanation:
Perimeter = 32 feet.
12 ft + k + 12 ft + k = 32 feet.
If k = 1 then 12 + 1 + 12 + 1 = 26 feet not equal to 32 feet.
If k = 2 then 12 + 2 + 12 + 2 = 28 feet not equal to 32 feet.
If k = 3 then 12 + 3 + 12 + 3 = 30 feet not equal to 32 feet.
If k = 4 then 12 + 4 + 12 + 4 = 32 feet equal to 32 feet.
So, k = 4 feet.

Find the area of the figure. Each unit square is 1 square meter.

Question 7.
Go Math Grade 3 Answer Key Chapter 11 Perimeter and Area Mid -Chapter Checkpoin img 57
______ square meters

Answer:
14 square meters

Explanation:
Chapter 11 - Concepts and Skills -image 32

Question 8.
Go Math Grade 3 Answer Key Chapter 11 Perimeter and Area Mid -Chapter Checkpoin img 58
______ square meters

Answer:
30 square meters

Explanation:
Chapter 11 - Concepts and Skills -image 33 (2)
10 x 3 = 30 square meters.

Mid -Chapter Checkpoint – Page No. 662

Question 9.
Ramona is making a lid for her rectangular jewelry box. The jewelry box has side lengths of 6 centimeters and 4 centimeters. What is the area of the lid Ramona is making?
Go Math Grade 3 Answer Key Chapter 11 Perimeter and Area Mid -Chapter Checkpoint img 59
______ square meters

Answer:
24 square meters.

Explanation:
The area of the lid = 4 cm x 6 cm = 24 cm.
Ramona making 24 cm lid for her rectangular jewelry box.

Question 10.
Adrienne is decorating a square picture frame. She glued 36 inches of ribbon around the edge of the frame. What is the length of each side of the picture frame?
Go Math Grade 3 Answer Key Chapter 11 Perimeter and Area Mid -Chapter Checkpoint img 60
a = ______ inches

Answer:
9 inches

Explanation:
A square has the same lengths.
The length of each side 4 x a = 36 inches.
a = 36/4 = 9 inches.

Question 11.
Margo will sweep a room. A diagram of the floor that she needs to sweep is shown at the right. What is the area of the floor?
Go Math Grade 3 Answer Key Chapter 11 Perimeter and Area Mid -Chapter Checkpoint img 61
______ square units

Answer:
27 square units

Explanation:
Chapter 11 - Mid chapter checkpoint - image 38 662
27 square units

Question 12.
Jeff is making a poster for a car wash for the Campout Club. What is the perimeter of the poster?
Go Math Grade 3 Answer Key Chapter 11 Perimeter and Area Mid -Chapter Checkpoint img 62
______ feet

Answer:
8 ft

Explanation:
Perimeter = 3 + 1 + 3 + 1 = 8 ft.

Question 13.
A rectangle has two side lengths of 8 inches and two side lengths of 10 inches. What is the perimeter of the rectangle? What is the area of the rectangle?
Perimeter ______ inches
Area = ______ square inches

Answer:
Perimeter = 36 inches.
Area = 80 square inches.

Explanation:
Chapter 11 - Mid chapter checkpoint - image 39
The perimeter of the rectangle = 8 + 10 + 8 + 10 = 36.
Area = 10 x 8 = 80 square inches.

Problem Solving Area of Rectangles – Page No. 667

Use the information for 1–3.

An artist makes rectangular murals in different sizes. Below are the available sizes. Each unit square is 1 square meter.
Go Math Grade 3 Answer Key Chapter 11 Perimeter and Area Problem Solving Area of Rectangles img 63

Chapter 11 - problem solving area of rectangles-images 39

Question 1.
Complete the table to find the area of each mural.

Mural Length (in meters) Width (in meters) Area (in square meters)
A 2 1 2
B 2 2 4
C 2 __4_______ __8_______
D 2 ____8_____ ___16______

Question 2.
Find and describe a pattern of how the length changes and how the width changes for murals A through D.
Type below:
_____________

Answer:
For each mural, the width doubles and the length stays the same.

Question 3.
How do the areas of the murals change when the width changes?
Type below:
_____________

Answer:
For each mural, the area doubles.

Question 4.
Dan built a deck that is 5 feet long and 5 feet wide. He built another deck that is 5 feet long and 7 feet wide. He built a third deck that is 5 feet long and 9 feet wide. How do the areas change?
Type below:
_____________

Answer:
The area of each deck is increased by 10 square feet.

Explanation:
1st deck area = 5 x 5 = 25 feet.
2nd deck area = 5 x 7 = 35 feet.
3rd deck area = 5 x 9 = 45 feet.
The area of each deck is increased by 10 square feet.

Problem Solving Area of Rectangles – Page No. 668

Lesson Check

Question 1.
Lauren drew the designs below. Each unit square is 1 square centimeter. If the pattern continues, what will be the area of the fourth shape?
Go Math Grade 3 Answer Key Chapter 11 Perimeter and Area Problem Solving Area of Rectangles img 64
Options:
a. 10 square centimeters
b. 12 square centimeters
c. 14 square centimeters
d. 16 square centimeters

Answer:
b. 12 square centimeters

Explanation:
Chapter 11 - problem solving area of rectangles-images 40
4 x 3 = 12 cm. The area of the fourth shape is 12 square centimeters.

Question 2.
Henry built one garden that is 3 feet wide and 3 feet long. He also built a garden that is 3 feet wide and 6 feet long, and a garden that is 3 feet wide and 9 feet long. How do the areas change?
Options:
a. The areas do not change.
b. The areas double.
c. The areas increase by 3 square feet.
d. The areas increase by 9 square feet.

Answer:
d. The areas increase by 9 square feet.

Explanation:
1st garden = 3 x 3 = 9 feet.
2nd garden = 3 x 6 = 18 feet.
3rd garden = 3 x 9 = 27 feet.
The areas increase by 9 square feet.

Spiral Review

Question 3.
Joe, Jim, and Jack share 27 football cards equally. How many cards does each boy get?
Options:
a. 7
b. 8
c. 9
d. 10

Answer:
c. 9

Explanation:
x + x + x = 27.
3x = 27.
x = 27/3 = 9.
Each boy gets 9 football cards.

Question 4.
Nita uses \(\frac{1}{3}\) of a carton of 12 eggs. How many eggs does she use?
Go Math Grade 3 Answer Key Chapter 11 Perimeter and Area Problem Solving Area of Rectangles img 65
Options:
a. 3
b. 4
c. 6
d. 9

Answer:
b. 4

Explanation:
\(\frac{1}{3}\) of a carton of 12 eggs = \(\frac{1}{3}\) x 12 = 4.
Nita uses 4 eggs.

Question 5.
Brenda made 8 necklaces. Each necklace has 10 large beads. How many large beads did Brenda use to make the necklaces?
Options:
a. 80
b. 85
c. 90
d. 100

Answer:
a. 80

Explanation:
8 x 10 = 80 beads.
Brenda uses 80 beads to make the necklaces

Question 6.
Neal is tiling his kitchen floor. Each square tile is 1 square foot. Neal uses 6 rows of tiles with 9 tiles in each row. What is the area of the floor?
Options:
a. 15 square feet
b. 52 square feet
c. 54 square feet
d. 57 square feet

Answer:
c. 54 square feet

Explanation:
The area of the floor = 6 x 9 = 54 square feet.

Area of Combined Rectangles – Page No. 673

Use the Distributive Property to find the area.
Show your multiplication and addition equations.

Question 1.
Go Math Grade 3 Answer Key Chapter 11 Perimeter and Area Area of Combined Rectangles img 66

Answer:
28 Square Units

Explanation:
Chapter 11 - area of combined rectangles - image 40

Question 2.
Go Math Grade 3 Answer Key Chapter 11 Perimeter and Area Area of Combined Rectangles img 67
_______ square units

Answer:
27 square units

Explanation:
Chapter 11 - area of combined rectangles - image 41
3 x 3 = 9; 3 x 6 = 18.
9 + 18 = 27
27 square units.

Draw a line to break apart the shape into rectangles. Find the area of the shape.

Question 3.
Go Math Grade 3 Answer Key Chapter 11 Perimeter and Area Area of Combined Rectangles img 68
Type below:
_____________

Answer:
31 square units

Explanation:
Chapter 11 - area of combined rectangles - image 42
5 x 5 = 25; 2 x 3 = 6
25 + 6 = 31
31 square units

Question 4.
Go Math Grade 3 Answer Key Chapter 11 Perimeter and Area Area of Combined Rectangles img 69
Type below:
_____________

Answer:
32 square units

Explanation:
Chapter 11 - area of combined rectangles - image 43
4 x 4 = 16; 2 x 8 = 16.
16 + 16 = 32.
32 square units

Problem Solving

A diagram of Frank’s room is at right. Each unit square is 1 square foot.
Go Math Grade 3 Answer Key Chapter 11 Perimeter and Area Area of Combined Rectangles img 70

Question 5.
Draw a line to divide the shape of Frank’s room into rectangles.

Answer:
Chapter 11 - area of combined rectangles - image 45 (2)

Question 6.
What is the total area of Frank’s room?
_______ square feet

Answer:
75 square feet

Explanation:
Chapter 11 - area of combined rectangles - image 44
6 x 5 = 30; 5 x 9 = 45
30 + 45 = 75.
75 square feet

Area of Combined Rectangles – Page No. 674

Lesson Check

Question 1.
The diagram shows Ben’s backyard. Each unit square is 1 square yard. What is the area of Ben’s backyard?
Go Math Grade 3 Answer Key Chapter 11 Perimeter and Area Area of Combined Rectangles img 71
Options:
a. 12 square yards
b. 16 square yards
c. 18 square yards
d. 24 square yards

Answer:
b. 16 square yards

Explanation:
Chapter 11 - area of combined rectangles - image 45
6 x 3 = 18 square yards

Question 2.
The diagram shows a room in an art gallery. Each unit square is 1 square meter. What is the area of the room?
Go Math Grade 3 Answer Key Chapter 11 Perimeter and Area Area of Combined Rectangles img 72
Options:
a. 24 square meters
b. 30 square meters
c. 36 square meters
d. 40 square meters

Answer:
b. 30 square meters

Explanation:
Chapter 11 - area of combined rectangles - image 46
3 x 5 = 15; 3 x 5 = 15
15 + 15 = 30 square meters

Spiral Review

Question 3.
Naomi needs to solve 28 ÷ 7 = ■. What related multiplication fact can she use to find the unknown number?
Options:
a. 3 × 7 = 21
b. 4 × 7 = 28
c. 5 × 7 = 35
d. 6 × 7 = 42

Answer:
b. 4 × 7 = 28

Explanation:
28 ÷ 7 = 4. So, 4 x 7 = 28 is the answer.

Question 4.
Karen drew a triangle with side lengths 3 centimeters, 4 centimeters, and 5 centimeters. What is the perimeter of the triangle?
Options:
a. 7 centimeters
b. 9 centimeters
c. 11 centimeters
d. 12 centimeters

Answer:
d. 12 centimeters

Explanation:
Perimeter = 3 centimeters + 4 centimeters + 5 centimeters = 12 centimeters.

Question 5.
The rectangle is divided into equal parts. What is the name of the equal parts?
Go Math Grade 3 Answer Key Chapter 11 Perimeter and Area Area of Combined Rectangles img 73
Options:
a. half
b. third
c. fourth
d. sixth

Answer:
c. fourth

Question 6.
Use an inch ruler. To the nearest half inch, how long is this line segment?
Go Math Grade 3 Answer Key Chapter 11 Perimeter and Area Area of Combined Rectangles img 74
Options:
a. 1
b. 1 \(\frac{1}{2}\)
c. 2
d. 2 \(\frac{1}{2}\)

Answer:
c. 2

Explanation:
The line segment = 2 inches.

Same Perimeter, Different Areas – Page No. 679

Find the perimeter and the area.
Tell which rectangle has a greater area.

Question 1.
Go Math Grade 3 Answer Key Chapter 11 Perimeter and Area Same Perimeter, Different Areas img 75
A: Perimeter = 12 ;
Area = 9 square units

B: Perimeter = ______ units;
Area = ______ square units;
Rectangle ______ has a greater area.

Answer:
Rectangle A has a greater area.

Explanation:
Chapter 11 - same perimeter, different areas - image 1
A: Perimeter = 12 ;
Area = 9 square units
B: Perimeter = 2 + 4 + 2 + 4 = 12
Area = 2 x 4 = 8 square units
Rectangle A has a greater area.

Question 2.
Go Math Grade 3 Answer Key Chapter 11 Perimeter and Area Same Perimeter, Different Areas img 76
A: Perimeter = ______ units;
Area = ______ square units;

B: Perimeter = ______ units;
Area = ______ square units;
Rectangle ______ has a greater area.

Answer:
Rectangle A has a greater area.

Explanation:
Chapter 11 - same perimeter, different areas - image 2

A: Perimeter = 4 + 1 + 4 + 1 = 10
Area = 4 x 1 = 4 square units
B: Perimeter = 3 + 2 + 3 + 2 = 10
Area = 3 x 2 = 6 square units
Rectangle B has a greater area.

Problem Solving

Question 3.
Tara’s and Jody’s bedrooms are shaped like rectangles. Tara’s bedroom is 9 feet long and 8 feet wide. Jody’s bedroom is 7 feet long and 10 feet wide. Whose bedroom has the greater area? Explain.
_________ ‘s bedrooms is greater.

Answer:
Tara’s bedroom has a greater area than Jody’s bedroom area.

Explanation:
Tara’s bedroom area = 9 x 8 = 72 feet.
Jody’s bedroom area = 7 x 10 = 70 feet.
72 feet > 70 feet.
Tara’s bedroom has a greater area than Jody’s bedroom area.

Question 4.
Mr. Sanchez has 16 feet of fencing to put around a rectangular garden. He wants the garden to have the greatest possible area. How long should the sides of the garden be?
Width: ______ Length: ______ feet long

Answer:
All four sides should be 4 feet long.

Explanation:
Mr. Sanchez has 16 feet of fencing to put around a rectangular garden.
Perimeter = 1 + 7 + 1 + 7 = 16 feet. Area = 1 x 7 = 7 feet.
Perimeter = 2 + 6 + 2 + 6 = 16 feet. Area = 2 x 6 = 12 feet.
Perimeter = 3 + 5 + 3 + 5 = 16 feet. Area = 3 x 5 = 15 feet.
perimeter = 4 + 4 + 4 + 4 = 16 feet. Area = 4 x 4 = 16 feet.
The area is maximum when all the four sides are 4 feet long.

Same Perimeter, Different Areas – Page No. 680

Question 1.
Which shape has a perimeter of 12 units and an area of 8 square units?
Options:
a. Go Math Grade 3 Answer Key Chapter 11 Perimeter and Area Same Perimeter, Different Areas img 77
b. Go Math Grade 3 Answer Key Chapter 11 Perimeter and Area Same Perimeter, Different Areas img 78
c. Go Math Grade 3 Answer Key Chapter 11 Perimeter and Area Same Perimeter, Different Areas img 79
d. Go Math Grade 3 Answer Key Chapter 11 Perimeter and Area Same Perimeter, Different Areas img 80

Answer:
b

Explanation:
Perimeter = 2 + 4 + 2 + 4 = 12 units.
Area = 2 x 4 = 8 square units

Question 2.
All four rectangles below have the same perimeter. Which rectangle has the greatest area?
Options:
a. Go Math Grade 3 Answer Key Chapter 11 Perimeter and Area Same Perimeter, Different Areas img 81
b. Go Math Grade 3 Answer Key Chapter 11 Perimeter and Area Same Perimeter, Different Areas img 82
c. Go Math Grade 3 Answer Key Chapter 11 Perimeter and Area Same Perimeter, Different Areas img 83
d. Go Math Grade 3 Answer Key Chapter 11 Perimeter and Area Same Perimeter, Different Areas img 84

Answer:
d

Explanation:
a. Area = 2 x 10 = 20.
b. Area = 3 x 9 = 27.
c. Area = 4 x 8 = 32.
d. Area = 5 x 7 = 35.
d has the greatest area.

Spiral Review

Question 3.
Kerrie covers a table with 8 rows of square tiles. There are 7 tiles in each row. What is the area that Kerrie covers in square units?
Options:
a. 15 square units
b. 35 square units
c. 42 square units
d. 56 square units

Answer:
d. 56 square units

Explanation:
Area = 8 x 7 = 56.
Kerrie covers 56 square units area.

Question 4.
Von has a rectangular workroom with a perimeter of 26 feet. The length of the workroom is 6 feet. What is the width of Von’s workroom?
Options:
a. 7 feet
b. 13 feet
c. 20 feet
d. 26 feet

Answer:
a. 7 feet

Explanation:
Perimeter = 26 feet.
Length = 6 feet.
Given Von has a rectangular workroom. So, two lengths and two widths are the same for a rectangle.
Perimeter of a rectangle = Length + Width + Length + Width = 6 + W + 6 + W = 26 feet.
12 + 2W = 26 feet.
2W = 26 – 12 = 14.
W = 14/2 = 7 feet.

Same Perimeter, Different Areas – Page No. 685

Find the perimeter and the area. Tell which rectangle has a greater perimeter.

Question 1.
Go Math Grade 3 Answer Key Chapter 11 Perimeter and Area Same Area, Different Perimeters img 85
A: Area = 8 square units ;
Perimeter = 18 units ;

B: Area = _______ square units;
Perimeter = _______ units;
Rectangle _______ has a greater perimeter.

Answer:
Rectangle A has a greater perimeter

Explanation:
Chapter 11 - same perimeter, different areas - image 5
A: Area = 8 square units ;
Perimeter = 18 units ;
B: Area = ___8____ square units;
Perimeter = ____12___ units;
Rectangle ___A____ has a greater perimeter.

Question 2.
Go Math Grade 3 Answer Key Chapter 11 Perimeter and Area Same Area, Different Perimeters img 86
A: Area = _______ square units;
Perimeter = _______ units;

Question 2.
Go Math Grade 3 Answer Key Chapter 11 Perimeter and Area Same Area, Different Perimeters img 87
B: Area = _______ square units;
Perimeter = _______ units;
Rectangle _______ has a greater perimeter.

Answer:
Rectangle B has a greater perimeter

Explanation:
Chapter 11 - same perimeter, different areas - image 6
Chapter 11 - same perimeter, different areas - image 7
A: Area = 12 square units ;
Perimeter = 14 units ;
B: Area = ___12____ square units;
Perimeter = ____16___ units;
Rectangle ___B____ has a greater perimeter.

Question 3.
Go Math Grade 3 Answer Key Chapter 11 Perimeter and Area Same Area, Different Perimeters img 88
A: Area = _______ square units;
Perimeter = _______ units;

Question 3.
Go Math Grade 3 Answer Key Chapter 11 Perimeter and Area Same Area, Different Perimeters img 89
B: Area = _______ square units;
Perimeter = _______ units;
Rectangle _______ has a greater perimeter.

Answer:
Rectangle B has a greater perimeter

Explanation:
Chapter 11 - same perimeter, different areas - image 8
Chapter 11 - same perimeter, different areas - image 9
A: Area = 16 square units ;
Perimeter = 16 units ;
B: Area = ___16____ square units;
Perimeter = ____20___ units;
Rectangle ___B____ has a greater perimeter.

Problem Solving

Use the tile designs for 4–5.
Go Math Grade 3 Answer Key Chapter 11 Perimeter and Area Same Area, Different Perimeters img 90

Chapter 11 - same perimeter, different areas - image 10

Question 4.
Compare the areas of Design A and Design B.
The area of Design A ________ The area of Design B

Answer:
The area of Rectangle A and Rectangle B are equal.

Explanation:
A: Area = 20 square units ;
B: Area = ___20____ square units;

Question 5.
Compare the perimeters. Which design has the greater perimeter?
The perimeter of A ________ The perimeter of B

Answer:
The perimeter of A Greater than the perimeter of B

Explanation:
A: Perimeter = 24 units ;
B: Perimeter = ____18___ units;

Same Perimeter, Different Areas – Page No. 686

Lesson Check

Question 1.
Jake drew two rectangles. Which statement is true?
Go Math Grade 3 Answer Key Chapter 11 Perimeter and Area Same Area, Different Perimeters img 91
Options:
a. The perimeters are the same.
b. The area of A is greater.
c. The perimeter of A is greater.
d. The perimeter of B is greater

Answer:
d. The perimeter of B is greater

Explanation:
Chapter 11 - same perimeter, different areas - image 11
A: Area = 6 square units ;
Perimeter = 10 units ;
B: Area = ___6____ square units;
Perimeter = ____14___ units;
The perimeter of B is greater.

Question 2.
Alyssa drew two rectangles. Which statement is true?
Go Math Grade 3 Answer Key Chapter 11 Perimeter and Area Same Area, Different Perimeters img 92
Options:
a. The perimeter of B is greater.
b. The perimeter of A is greater.
c. The area of B is greater.
d. The perimeters are the same.

Answer:
b. The perimeter of A is greater.

Explanation:
Chapter 11 - same perimeter, different areas - image 12
A: Area = 18 square units ;
Perimeter = 22 units ;
B: Area = ___18____ square units;
Perimeter = ____18___ units;
The perimeter of A is greater.

Spiral Review

Question 3.
Marsha was asked to find the value of 8 – 3 x 2. She wrote a wrong answer. Which is the correct answer?
Options:
a. 22
b. 10
c. 4
d. 2

Answer:
d. 2

Explanation:
8 – (3 x 2) = 8 – 6 = 2

Question 4.
What fraction names the point on the number line?
Go Math Grade 3 Answer Key Chapter 11 Perimeter and Area Same Area, Different Perimeters img 93
Options:
a. \(\frac{1}{4}\)
b. \(\frac{2}{3}\)
c. \(\frac{3}{4}\)
d. \(\frac{3}{1}\)

Answer:
c. \(\frac{3}{4}\)

Explanation:
From the given figure, we can write the fraction name the point on the number line = \(\frac{3}{4}\)

Question 5.
Kyle drew three line segments with these lengths: \(\frac{2}{4}\) inch, \(\frac{2}{3}\) inch, and \(\frac{2}{6}\) inch. Which list orders the fractions from least to greatest?
Options:
a. \(\frac{2}{6}, \frac{2}{4}, \frac{2}{3}\)
b. \(\frac{2}{3}, \frac{2}{4}, \frac{2}{6}\)
c. \(\frac{2}{4}, \frac{2}{3}, \frac{2}{6}\)
d. \(\frac{2}{6}, \frac{2}{3}, \frac{2}{4}\)

Answer:
c. \(\frac{3}{4}\)

Explanation:
Given fractions are \(\frac{2}{4}\) inch, \(\frac{2}{3}\) inch, and \(\frac{2}{6}\) inch.
All the fractions have the same numerator. The denominators should compare to find the answer. The smaller denominator represents the larger number. So, \(\frac{2}{6}, \frac{2}{4}, \frac{2}{3}\) is the answer.

Question 6.
On Monday, \(\frac{3}{8}\) inch of snow fell. On Tuesday, \(\frac{5}{8}\) inch of snow fell. Which statement correctly compares the snow amounts?
Options:
a. \(\frac{3}{8}=\frac{5}{8}\)
b. \(\frac{3}{8}<\frac{5}{8}\)
c. \(\frac{5}{8}<\frac{3}{8}\)
d. \(\frac{3}{8}>\frac{5}{8}\)

Answer:
b. \(\frac{3}{8}<\frac{5}{8}\)

Explanation:
Given fractions are \(\frac{3}{8}\) and \(\frac{5}{8}\).
The denominators are equal for given fractions. So, compare numerators to find the correct equation.
3 < 5. So, \(\frac{3}{8}<\frac{5}{8}\) is the correct answer.

Review/Test – Page No. 687

Question 1.
Find the perimeter of each figure on the grid. Identify the figure that have a perimeter of 14 units. Mark all that apply.
Options:
a. Go Math Grade 3 Answer Key Chapter 11 Perimeter and Area Review/Test img 94
b. Go Math Grade 3 Answer Key Chapter 11 Perimeter and Area Review/Test img 95
c. Go Math Grade 3 Answer Key Chapter 11 Perimeter and Area Review/Test img 96
d. Go Math Grade 3 Answer Key Chapter 11 Perimeter and Area Review/Test img 97

Answer:
a and c have a perimeter of 14 units.

Explanation:
a. The Perimeter of A = 14 units.
b. The perimeter of B = 16 units.
c. The perimeter of C = 14 units.
d. The perimeter of D = 16 units

Question 2.
Kim wants to put trim around a picture she drew. How many centimeters of trim does Kim need for the perimeter of the picture?
Go Math Grade 3 Answer Key Chapter 11 Perimeter and Area Review/Test img 98
_________ centimeters

Answer:
24 centimeters

Explanation:
Perimeter = 6 + 6 + 6+ 6 = 24 centimeters.

Question 3.
Sophia drew this rectangle on dot paper. What is the area of the rectangle?
Go Math Grade 3 Answer Key Chapter 11 Perimeter and Area Review/Test img 99
_________ square units

Answer:
8 square units

Explanation:
Chapter 11 - review & test - image 1. jpg
8 square units

Review/Test – Page No. 688

Question 4.
The drawing shows Seth’s plan for a fort in his backyard. Each unit square is 1 square foot.
Go Math Grade 3 Answer Key Chapter 11 Perimeter and Area Review/Test img 100
Which equations can Seth use to find the area of the fort?
Mark all that apply.
Options:
a. 4 + 4 + 4 + 4 = 16
b. 7 + 4 + 7 + 4 = 22
c. 7 + 7 + 7 + 7 = 28
d. 4 × 4 = 16
e. 7 × 7 = 49
f. 4 × 7 = 28

Answer:
b and f are correct.

Explanation:
Chapter 11 - review & test - image 2. jpg
Area = 4 × 7 = 28
Perimeter = 7 + 4 + 7 + 4 = 22.

Question 5.
Which rectangle has a number of square units for its area equal to the number of units of its perimeter?
Options:
a. Go Math Grade 3 Answer Key Chapter 11 Perimeter and Area Review/Test img 101
b. Go Math Grade 3 Answer Key Chapter 11 Perimeter and Area Review/Test img 102
c. Go Math Grade 3 Answer Key Chapter 11 Perimeter and Area Review/Test img 103
d. Go Math Grade 3 Answer Key Chapter 11 Perimeter and Area Review/Test img 104

Answer:
B rectangle has a number of square units for its area equal to the number of units of its perimeter

Explanation:
a. Area = 1 x 7 = 7 units.
Perimeter = 1 + 7 + 1 + 7 = 16 units.
b. a. Area = 4 x 4 = 16 units.
Perimeter = 4 + 4 + 4 + 4 = 16 units.
c. a. Area = 2 x 6 = 12 units.
Perimeter = 2 + 6 + 2 + 6 = 16 units.
d. a. Area = 3 x 5 = 7 units.
Perimeter = 3 + 5 + 3 + 5 = 16 units.

Question 6.
Vanessa uses a ruler to draw a square. The perimeter of the square is 12 centimeters. Select a number to complete the sentence.
Go Math Grade 3 Answer Key Chapter 11 Perimeter and Area Review/Test img 105

The square has a side length of Go Math Grade 3 Answer Key Chapter 11 Perimeter and Area Review/Test img 106 centimeters.
_________ centimeters

Answer:
3 centimeters

Explanation:
Perimeter = s + s + s + s = 12 centimeters.
4s = 12 centimeters.
s = 3 centimeters.

Review/Test – Page No. 689

Question 7.
Tomas drew two rectangles on grid paper.
Circle the words that make the sentence true.
Go Math Grade 3 Answer Key Chapter 11 Perimeter and Area Review/Test img 107
Rectangle A has an area that is Go Math Grade 3 Answer Key Chapter 11 Perimeter and Area Review/Test img 108 the area of Rectangle B, and a perimeter that is Go Math Grade 3 Answer Key Chapter 11 Perimeter and Area Review/Test img 109 the perimeter of Rectangle B.
Type below:
_____________

Answer:
Rectangle A has an area that is the same as the area of Rectangle B.
A perimeter that is less than the perimeter of Rectangle B.

Explanation:
Chapter 11 - review & test - image 3. jpg
A: Area = 3 x 4 = 12 units.
Perimeter = 3 + 4 + 3 + 4 = 14 units.
B: Area = 2 x 6 = 12 units.
Perimeter = 2 + 6 + 2 + 6 = 16 units.

Question 8.
Yuji drew this figure on grid paper. What is the perimeter of the figure?
Go Math Grade 3 Answer Key Chapter 11 Perimeter and Area Review/Test img 110
________ units

Answer:
18 units

Explanation:
chapter 11 Review 689 image 1

Question 9.
What is the area of the figure shown? Each unit square is 1 square meter.
Go Math Grade 3 Answer Key Chapter 11 Perimeter and Area Review/Test img 111
________ square meters

Answer:
13 square meters

Explanation:
Chapter 11 - review & test - image 5. jpg

Review/Test – Page No. 690

Question 10.
Shawn drew a rectangle that was 2 units wide and 6 units long. Draw a different rectangle that has the same perimeter but a different area.
Go Math Grade 3 Answer Key Chapter 11 Perimeter and Area Review/Test img 112
Type below:
_____________

Answer:
Chapter 11 Review image 1 690
Area = 2 x 6 = 12 Square Units.
Area = 4 x 4 = 16 Square Units.

Question 11.
Mrs. Rios put a wallpaper border around the room shown below. She used 72 feet of wallpaper border. What is the unknown side length? Show your work.
Go Math Grade 3 Answer Key Chapter 11 Perimeter and Area Review/Test img 113
a = ________ ft

Answer:
16 feet

Explanation:
20 + a + 6 + 8 + 14 + 8 = 72 feet.
a + 56 = 72 feet.
a = 72 – 56 feet.
a = 16 feet.

Question 12.
Elizabeth has two gardens in her yard. The first garden is 8 feet long and 6 feet wide. The second garden is half the length of the first garden. The area of the second garden is twice the area of the first garden. For numbers 12a–12d, select True or False.

First garden = 8 x 6 = 48 feet.
Second garden length = Half the length of the first garden = 4 feet.
Second Garden Area = Twice the area of the first garden = 2 x 48 = 96.
Second Garden Area Width = 4 x s = 96.
s = 96/4 = 24

a. The area of the first garden is 48 square feet.
i. True
ii. False

Answer:
i. True

Question 12.
b. The area of the second garden is 24 square feet.
i. True
ii. False

Answer:
ii. False

Question 12.
c. The width of the second garden is 12 feet.
i. True
ii. False

Answer:
ii. False

Explanation:

Question 12.
d. The width of the second garden is 24 feet.
i. True
ii. False

Answer:
i. True

Review/Test – Page No. 691

Question 13.
Marcus bought some postcards. Each postcard had a perimeter of 16 inches. Which could be one of the postcards Marcus bought? Mark all that apply.
Options:
a. Go Math Grade 3 Answer Key Chapter 11 Perimeter and Area Review/Test img 114
b. Go Math Grade 3 Answer Key Chapter 11 Perimeter and Area Review/Test img 115
c. Go Math Grade 3 Answer Key Chapter 11 Perimeter and Area Review/Test img 116
d. Go Math Grade 3 Answer Key Chapter 11 Perimeter and Area Review/Test img 117

Answer:
a and d options are correct.

Explanation:
3 + 5 + 3 + 5 = 16 inches
4 + 6 + 4 + 6 = 20 inches.
5 + 10 + 5 + 10 = 30 inches.
4 + 4 + 4 + 4 = 16 inches.

Question 14.
Anthony wants to make two different rectangular flowerbeds, each with an area of 24 square feet. He will build a wooden frame around each flowerbed. The flowerbeds will have side lengths that are whole numbers.
Part A
Each unit square on the grid below is 1 square foot. Draw two possible flowerbeds. Label each with a letter.
Go Math Grade 3 Answer Key Chapter 11 Perimeter and Area Review/Test img 118
Type below:
__________

Answer:

Chapter 11 - review & test - image 8. jpg

Question 14.
Part B
Which of the flowerbeds will take more wood to the frame?
Explain how you know.
Type below:
__________

Answer:
A Perimeter is greater than B Perimeter.

Explanation:
Area = 24.
3 x 8 = 24; 4 x 6 = 24.
A Perimeter = 8 + 3 + 8 + 3 = 22 feet.
B Perimeter = 4 + 6 + 4 + 6 = 20 feet.
A Perimeter is greater than B Perimeter.

Review/Test – Page No. 692

Question 15.
Keisha draws a sketch of her living room on grid paper. Each unit square is 1 square meter. Write and solve a multiplication equation that can be used to find the area of the living room in square meters.
Go Math Grade 3 Answer Key Chapter 11 Perimeter and Area Review/Test img 119
Type below:
__________

Answer:
40 meter.

Explanation:
Chapter 11 - review & test - image 6. jpg
4 x 10 = 40 m.

Question 16.
Mr. Wicks designs houses. He uses grid paper to plan a new house design. The kitchen will have an area between 70 square feet and 85 square feet. The pantry will have an area between 4 square feet and 15 square feet. Draw and label a diagram to show what Mr. Wicks could design. Explain how to find the total area.
Go Math Grade 3 Answer Key Chapter 11 Perimeter and Area Review/Test img 120
Type below:
__________

Answer:
78 square feet

Explanation:Chapter 11 - review & test - image 7. jpg
Are of Kitchen = 8 x 9 = 72 square feet. (70 < 72 < 85)
Area of Pantry = 3 x 2 = 6 square feet. (4 < 6 < 15)
Total area of kitchen and pantry = 72 + 6 = 78 square feet.

Conclusion

Go Math Grade 3 Answer Key Chapter 11 Perimeter and Area has step-by-step solutions for all the problems making it easy for you to understand the concepts. Practice problems from the Go Math Grade 3 Answer Key Chapter 11 Perimeter and Area Extra Practice and test your preparation standards.